Gurukul NTA CUET (UG) Geography Question Bank Exam 2024 9788119962433


98 21 8MB

English Pages [389] Year 2023

Report DMCA / Copyright

DOWNLOAD PDF FILE

Table of contents :
Inner_Geography
Part-A Ch-1
Part-A Ch-2
Part-A Ch-3
Part-A Ch-4
Part-A Ch-5
Part-B Ch-1
Part-B Ch-2
Part-B Ch-3
Part-B Ch-4
Part-B Ch-5
Recommend Papers

Gurukul NTA CUET (UG) Geography Question Bank Exam 2024
 9788119962433

  • 0 0 0
  • Like this paper and download? You can publish your own PDF file online for free in a few minutes! Sign Up
File loading please wait...
Citation preview

COPYRIGHT RESERVED BY THE PUBLISHERS All rights reserved. No part of this publication may be reproduced in any form without the prior permission of the Oswal Publishers. DISCLAIMER With the ambition of providing standard academic resources, we have exercised extreme care in publishing the content. In case of any discrepancies in the matter, we request readers to excuse the unintentional lapse and not hold us liable for the same. Suggestions are always welcome. EDITION : 2023 ISBN : 978-93-95025-70-6 PRINTED AT : Upkar Print House Pvt. Ltd., Agra PUBLISHED BY

OSWAL PUBLISHERS Head Office : 1/12, Sahitya Kunj, M.G. Road, Agra – 282 002 Phone : (0562) 2527771-4

Whatsapp : +91 74550 77222 E-mail : [email protected] Website : www.oswalpublishers.com The cover of this book has been designed using resources from Freepik.com

It is a matter of immense pride for us to present our Question Bank of Geography for Section II of CUET (UG), especially prepared for students appearing for Common University Entrance Test (UG) in the upcoming years. This book is ideal for building self-confidence during exam preparation. The ‘Chapterwise’ format of the book with its exhaustive set of questions allows the students to cover every topic in an organised and systematic manner. The question bank adheres completely to the syllabus prescribed by NTA. The study material and exercises have been prepared in order to make students capable of adapting themselves to the syllabus, pattern, and standard of CUET (UG). With its simple language and style, this book is perfect for smart study. We are confident that the book will enable the candidates to develop a better understanding of the curriculum and help them organise their learning process. This book shall definitely prove to be a fruitful tool for the students and encourage them towards

scholastic excellence. The book contains a QR code given below the content page. Students can scan the QR code to solve and practice more questions and sample papers created by experts. Constructive suggestions for further improvement of the book are always appreciated. —Publisher

Part A - FUNDAMENTALS OF HUMAN GEOGRAPHY 1. Human Geography: Nature and Scope 2. Population 3. Human Activities 4. Transport, Communication and Trade 5. Human Settlement PART B - INDIA : PEOPLE AND ECONOMY 1. People 2. Human Settlements 3. Resources and Development 4. Transport and Communication 5. Geographical Perspective on Selected Issues and Problems

1. Human geography studies the relationship between ___________, the spatial distributions of human phenomena and how they come about, and the social and economic differences between different parts of the world. (a) the physical and biological worlds (b) the resting man and the stable earth (c) the natural phenomena and earth science (d) the physical and the human worlds Ans. (d) the physical and the human worlds 2. Physical and human phenomena are often described in metaphors: We often say the ‘face’ of the earth, ‘eye’ of the storm, and _________ of the river, ‘snout’ (nose) of the glacier, and so on. (a) ‘mouth (b) ‘neck’ (c) ‘limbs’ (d) ‘head’ Ans. (a) ‘mouth’ 3. Using symbols/metaphors from the human anatomy, regions, villages, and towns have been described as _____________. (a) ‘nucleus’ (b) ‘cells’ (c) ‘organisms’ (d) ‘organs’

Ans. (c) ‘organisms’ 4. Human beings interact with their physical environment with the help of ___________. (a) nature worship (b) technology (c) harmony with nature (d) none of the above Ans. (b) technology 5. The colonial period provided impetus to further explorations in order to _______________ and to obtain inventoried information. (a) explore new lands (b) conduct geographical survey (c) access the resources of the regions (d) study human societies Ans. (c) access the resources of the regions 6. Arrange the following stages of interaction with environment and the resultant developments with the passage of time, in a sequential order. (i) Human activities create cultural landscapes with imprints of their activities everywhere. (ii) In the early primitive society humans lived in harmony with nature, depending on Mother Nature for all their sustenance. (iii) They use the resources from the environment and create material things to make life comfortable. (iv) With social and cultural development, humans develop better

and efficient technology. Choose the correct option. (a) (iv), (i), (iii), (ii) (b) (iv), (i), (ii), (iii) (c) (ii), (iv), (iii), (i) (d) (iii), (i), (iv), (ii) Ans. (c) (ii), (iv), (iii), (i) 7. Arrange the following approaches in a sequential order according to their period of development, starting from the earliest. (i) Humanistic, radical and behavioural approach (ii) Spatial organisation (iii) Post-modernism approach (iv) Areal differentiation Choose the correct option. (a) (iv), (i), (ii), (iii) (b) (iv), (ii), (i), (iii) (c) (iv), (i), (iii), (ii) (d) (iii), (i), (ii), (iv), Ans. (b) (iv), (ii), (i), (iii) 8. Arrange the following features in a sequential order

according to their period of development, starting from the earliest. (i) Grand generalisations and the applicability of universal theories (ii) Imperial and trade interests initiate the discovery and exploration of new areas (iii) Discontentment with the quantitative revolution (iv) Elaborate description of all aspects of a region were undertaken Choose the correct option. (a) (i), (iv), (ii), (iii) (b) (iv), (i), (iii), (ii) (c) (ii), (iv), (iii), (i) (d) (iii), (ii), (iv), (i) Ans. (c) (ii), (iv), (iii), (i) 9. The earth comprises two major components: (a) Atoms and molecules (b) Hydrosphere and lithosphere (c) Nature and life forms, including human beings (d) Environment and natural resources Ans. (c) Nature and life forms, including human beings 10. What does geography lay emphasis on? (a) Understand all that is above and below the earth’s surface (b) The origin and the dynamic structure of the earth

(c) Explore all the domains of the earth and their interrelationship (d) The study of nature and human beings Ans. (d) The study of nature and human beings 11. Why is the dichotomy between physical and human not a valid one? (a) Nature and humans are inseparable elements (b) The biosphere has elements of the other three domains (c) Humans explore and create new resources (d) Humans are destroying the environment in the name of development Ans. (a) Nature and humans are inseparable elements 12. Symbols from which discipline are used as metaphors to describe physical and human phenomena? (a) Molecular biology (b) Human anatomy (c) Astrophysics (d) Thermodynamics Ans. (b) Human anatomy 13. Identify the metaphor used from human anatomy for a part of a glacier. (a) Neck (b) Mouth (c) Snout (d) Profile Ans. (c) Snout

14. Which of the following terms is used to explain the different layers of soil? (a) Limbs (b) Arteries (c) Eye (d) Profile Ans. (d) Profile 15. Which geographical concept is described as ‘arteries of circulation’? (a) Circulation of air/wind (b) Ocean currents (c) Networks of road, railways and waterways (d) Earth’s water cycle Ans. (c) Networks of road, railways and waterways 16. Which scholar gave this definition: Human geography is the study of “the changing relationship between the unresting man and the unstable earth”? (a) Ellen C. Semple (b) Ratzel (c) Paul Vidal de la Blache (d) Griffith Taylor Ans. (a) Ellen C. Semple 17. What helped the people of Trondheim to overcome the constraints of nature? (a) Nature worship (b) Harmony with nature

(c) Technology (d) Use of natural resources Ans. (c) Technology 18. What is meant by environmental determinism? (a) Nature permits modifications (b) Creating possibilities with the resources in the environment (c) Changing relationship between the unresting man and the unstable earth (d) Interaction between primitive human society and the strong forces of nature Ans. (d) Interaction between primitive human society and the strong forces of nature 19. Which option is an example of material culture? (a) The three domains of the earth (b) All types of natural resources (c) Houses, villages, cities, industries, etc. (d) Rocks and minerals Ans. (c) Houses, villages, cities, industries, etc. 20. Which of the following is not an approach in human geography? (a) Exploration and description (b) Areal differentiation (c) Spatial organisation (d) Quantitative revolution Ans. (d) Quantitative revolution

21. Which approach was supported by Paul Vidal de la Blache? (a) Determinism (b) Humanism (c) Possibilism (d) Welfare approach Ans. (c) Possibilism 22. Which of these scholars defined Human Geography as ‘the synthetic study of relationship between human societies and earth’s surface’? (a) Ratzel (b) Paul Vidal de la Blache (c) Ellen C. Semple (d) Griffith Taylor Ans. (a) Ratzel 23. Who was the founder of neo-determinism? (a) Carl Ritter (b) Griffith Taylor (c) Paul Vidal de la Blache (d) Sir Halford John Mackinder Ans. (b) Griffith Taylor 24. What indicates the level of cultural develop-ment of a society? (a) Pattern of settlements (b) Communication network

(c) Technology (d) Law and order Ans. (c) Technology 25. The understanding of which concepts enabled humans to conquer diseases? (a) Human anatomy (b) Cellular biology (c) Virology (d) Secrets of genetics and DNA Ans. (d) Secrets of genetics and DNA 26. In which type of societies is the physical environment looked upon as “Mother Nature”? (a) Traditional societies (b) Primitive societies (c) Developing societies (d) Rural societies Ans. (b) Primitive societies 27. Which term best describes the following statement: “Humans create possibilities with the resources obtained from the environment and their activities create a cultural landscape.”? (a) Opportunism (b) Humanisation of nature (c) Possibilism (d) Resource management Ans. (c) Possibilism

28. Which concept is also known as ‘stop and go determinism’? (a) Environmental determinism (b) Possibilism (c) Opportunism (d) Neo-determinism Ans. (d) Neo-determinism 29. Which option best explains the essence of neodeterminism? (a) A free run is permissible and there are no signals for developing economies. (b) Possibilities can be created within the limits which do not damage the environment. (c) Human beings can conquer nature by exploiting it to their advantage. (d) Increased resource consumption is permissible for growing economies. Ans. (b) Possibilities can be created within the limits which do not damage the environment. 30. Which century witnessed attempts of explorations in Europe? (a) Thirteenth century (b) Fourteenth century (c) Fifteenth century (d) Sixteenth century Ans. (c) Fifteenth century 31. What was the level of technological development when

humans were most naturalised? (a) Very low technological development (b) Medium technological development (c) Complex technological development (d) Selective technological development Ans. (a) Very low technological development 32. Which school of thought deals with the social well-being of people? (a) Radical school (b) Humanistic school (c) Behavioural school (d) Demographic school Ans. (b) Humanistic school 33. What is the nature of Human Geography? (a) Highly inter-disciplinary (b) Highly environment centered (c) Highly socio-cultural (d) Highly socio-economic Ans. (a) Highly inter-disciplinary 34. Which of the following is not an area of study in Human Geography?

(a) Settlements (b) Transport network (c) Pedagogy (d) Industries Ans. (c) Pedagogy 35. When does physical environment become “Mother Nature” for societies? (a) When human activities create cultural landscape. (b) When there is a dynamic relationship between human beings and nature. (c) When there is direct dependence of human beings on nature for their sustenance. (d) When technology is used for tapping resources in the physical environment. Ans. (c) When there is direct dependence of human beings on nature for their sustenance. 36. Which geographical concept attempts to balance nullify the ‘either’ ‘or’ dichotomy? (a) Neo-determinism (b) Spatial organisation (c) Possibilism (d) Environmental determinism Ans. (a) Neo-determinism

37. What does the analogy of traffic lights explain in the context of human geography? (a) It indicates that there are rules everywhere. (b) It explains that human beings can conquer nature by obeying it. (c) It indicates that humans should not be given total freedom. (d) It indicates that humans are basically exploitative in nature. Ans. (b) It explains that human beings can conquer nature by obeying it. 38. Of which of these fields of Human Geography, is Medical Geography a sub-field? (a) Settlement Geography (b) Population Geography (c) Urban Geography (d) Social Geography Ans. (d) Social Geography 39. With which discipline of Social Sciences is the sub-field Cultural Geography related? (a) Anthropology (b) Psychology (c) Sociology (d) History

Ans. (a) Anthropology 40. What is meant by ‘naturalised human’? (a) One who has the ability to discover natural resources and create new things. (b) One who listened to Nature, was afraid of its fury and worshiped it. (c) One who exploits nature and is at the same time impacted by it. (d) One who lived in the bosom of nature at all stages of life. Ans. (b) One who listened to Nature, was afraid of its fury and worshiped it. 41. Which concepts did the Radical School of thought explain by employing the Marxian theory? (a) Population, economy and social well-being (b) Population, poverty and death rate (c) Poverty, deprivation and social inequality (d) Housing, health and education Ans. (c) Poverty, deprivation and social inequality 42. What did the behavioural school of thought lay emphasis on? (a) Contemporary social problems and their relationship to the development of capitalism (b) The different aspects of social well-being of the people (c) Attempts to bring a balance nullifying the ‘either’ ‘or’ dichotomy (d) Lived experience and the perception of space by social

categories based on ethnicity, race, etc. Ans. (d) Lived experience and the perception of space by social categories based on ethnicity, race, etc. 43. In which approach were the grand generalisations and the applicability of universal theories to explain the human conditions questioned? (a) Post-modernism in Geography (b) Humanistic, radical and behavioural schools (c) Regional analysis (d) Exploration and description Ans. (a) Post-modernism in Geography 44. What are the broad features of the spatial organisation approach of the late 1950s to the late 1960s? (a) Elaborate description of all aspects of a region were undertaken. (b) Laws of physics were often applied to map and analyse human phenomena. (c) The grand generalisations and the applicability of universal theories to explain human conditions were questioned. (d) Discontentment with the quantitative revolution and its dehumanised manner of doing geography. Ans. (b) Laws of physics were often applied to map and analyse human phenomena. 45. During which period did the humanistic, radical and behavioural schools emerge? (a) Later Colonial period (b) Late 1950s to the late 1960s

(c) 1970s (d) 1990s Ans. (c) 1970s 46. Identify the complete set of elements of physical environment. (a) Land, water and vegetation (b) Landforms, soils, climate, water, natural vegetation and diverse flora and fauna (c) Soil, climate and vegetation (d) Natural vegetation and wildlife Ans. (b) Landforms, soils, climate, water, natural vegetation and diverse flora and fauna 47. What is naturalisation of human and humanisation of nature termed as ________? (a) Environmental determinism and possibilism respectively (b) Possibilism and Environmental determinism respectively (c) Possibilism and Neo-determinism respectively (d) Neo-determinism and Possibilism respectively Ans. (a) Environmental determinism and Possibilism respectively 48. What was the main objective of quantitative revolution? (a) To develop imperial and trade interests (b) To give an elaborate description of all aspects of a region (c) To identify the uniqueness of any region and understanding how and why it was different from others (d) To identify mappable patterns for different human activities Ans. (d) To identify mappable patterns for different human

activities. 49. What led to the emergence of humanistic, radical and behavioural schools? (a) Discontentment with the quantitative revolution and its dehumanised manner of tackling Geography (b) Understanding of regions as part of a whole (c) Discovery and exploration of new areas with imperial and trade interests (d) Application of laws of aerodynamics to develop faster planes Ans. (a) Discontentment with the quantitative revolution and its dehumanised manner of tackling Geography. 50. Consider the following statements. A. Dynamism in the relationship (between human societies and earth’s surface) is the keyword in Ratzel’s definition of Human Geography. B. Synthesis in the relationship (between human societies and earth’s surface) has been emphasized in Semple’s definition of Human Geography. Choose the correct option. (a) Only A is correct. (b) Only B is correct. (c) Both the statements are incorrect. (d) Both the statements are correct. Ans. (c) Both the statements are incorrect. 51. Consider the following statements. A. According to the concept of neo-determinism, humans cannot conquer nature by obeying it.

B. Neo-determinism conceptually attempts to bring a balance nullifying the ‘either’ ‘or’ dichotomy. Choose the correct option. (a) Only A is correct. (b) Only B is correct. (c) Both the statements are incorrect. (d) Both the statements are correct. Ans. (b) Only B is correct. 52. Consider the following statements. A. Possibilities can be created within the limits which do not damage the environment and there is no free run without accidents. B. The free run which the developed economies attempted to take resulted in the greenhouse effect, global warming, receding glaciers, and so on. Choose the correct option. (a) Only A is correct. (b) Only B is correct. (c) Both the statements are incorrect. (d) Both the statements are correct and statement B explains statement A. Ans. (d) Both the statements are correct and statement B explains statement A. 53. Match Column I (Period) with Column II (Approaches) and choose the correct option with the help of given Codes. Column I

Column II

I.

Early Colonial period

1.

Areal differentiation

II.

Later Colonial period

2.

Exploration and description

III.

1930s through the Inter-War period

3.

Spatial organisation

IV.

Late 1950s to the late 1960s

4.

Regional analysis

Codes: I II III IV (a) 2 3 4 1 (b) 3 4 2 1 (c) 2 4 1 3 (d) 1 3 2 4 Ans. (c) I – 2, II – 4, III – 1, IV – 3 54. Match Column I (Period) with Column II (Broad Features) and choose the correct option with the help of given Codes. Column I

Column II

I.

1930s through the Inter-War period

1.

Emergence of humanistic, radical and behavioural schools

II.

Late 1950s to the late 1960s

2.

Generalisations and applicability of universal theories to explain the human conditions were questioned

III.

1970s

3.

Focus on identifying the uniqueness of any region

IV.

1990s

4.

The phase of statistical tools and quantitative revolution.

Codes: I II III IV (a) 2 3 4 1 (b) 3 4 1 2 (c) 2 4 1 3 (d) 3 1 2 4 Ans. (b) I – 3, II – 4, III – 1, IV – 2 55. Match Column I (Fields of Human Geography) with Column II (Sub-fields/Interface with Sister Disciplines of Social Sciences) and choose the correct option with the help of given Codes. Column I

Column II

I.

Social Geography

1.

Geography of Tourism

II.

Political Geography

2.

Behavioural Geography

III.

Economic Geography

3.

Urban/Rural Planning

IV.

Settlement Geography

4.

Electoral Geography

Codes: I II III IV (a) 1 3 4 2 (b) 3 1 2 4 (c) 1 4 2 3 (d) 2 4 1 3

Ans. (d) I – 2, II – 4, III – 1, IV – 3 56. Match Column I (Sub-fields of Human Geography) with Column II (Interface with Sister Disciplines of Social Sciences) and choose the correct option with the help of given Codes. Column I

Column II

I.

Medical Geography

1. Resource Economics

II.

Geography of Resources

2. Women’s Studies

III.

Gender Geography

3. Welfare Economics

IV.

Geography of Social Well-being

4. Epidemiology

Codes: I II III IV (a) 4 1 2 3 (b) 3 1 2 4 (c) 1 2 4 3 (d) 2 4 1 3 Ans. (a) I – 4, II – 1, III – 2, IV – 3

1. The world at the beginning of the 21st century recorded the presence of over __________ population. (a) 5 billion (b) 5.5 billion (c) 6 billion (d) 6.6 billion Ans. (c) 6 billion 2. Approximately, __________ of the world population lives in about 10 per cent of its land area. (a) 75 percent (b) 80 percent (c) 85 percent (d) 90 percent Ans. (d) 90 percent 3. Change of population in particular area between two points of time is known as ___________. (a) growth of population (b) natural growth of population (c) positive growth of population (d) negative growth of population Ans. (a) growth of population 4. World population ___________in the eighteenth century after the Industrial Revolution. (a) exploded

(b) became constant (c) decreased (d) none of these Ans. (a) exploded 5. There is______correlation between economic development and population growth. (a) positive (b) negative (c) fluctuating (d) alternating Ans. (b) negative 6. Arrange the following regions in descending order according to their population density in 2018. (i) Africa (ii) Northern America (iii) Asia (iv) Latin America and the Caribbean Choose the correct option. (a) (i), (iv), (iii), (ii) (b) (iv), (ii), (i), (iii) (c) (ii), (iv), (iii), (i) (d) (iii), (i), (iv), (ii) Ans. (d) (iii), (i), (iv), (ii) 7. Arrange the following Asian countries in ascending order according to their population size.

(i) China (ii) Pakistan (iii) Indonesia (iv) India Choose the correct option. (a) (i), (iv), (iii), (ii) (b) (iv), (ii), (i), (iii) (c) (ii), (iii), (iv), (i) (d) (iii), (i), (iv), (ii) Ans. (c) (ii), (iii), (iv), (i) 8. Arrange the following regions in descending order according to their share in the total world population (as in 2018). (i) Northern America (ii) Africa (iii) Latin America and the Caribbean (iv) Europe Choose the correct option. (a) (ii), (iv), (iii), (i) (b) (iii), (i), (iv), (ii) (c) (ii), (iii), (iv), (i) (d) (ii), (i), (iv), (iii) Ans. (a) (ii), (iv), (iii), (i) 9. Arrange the following factors in sequential order to show the progression of impact of science and technology on the growth of population.

(i) Rapid decline in death rates throughout the world. (ii) Mechanised energy of water and wind increased agricultural and industrial production. (iii) Steam engine replaced human and animal energy. (iv) Inoculation against epidemics and communicable diseases, improvement in medical facilities and sanitation. Choose the correct option. (a) (ii), (iv), (iii), (i) (b) (iii), (ii), (iv), (i) (c) (ii), (iii), (iv), (i) (d) (i), (iii), (iv), (ii) Ans. (b) (iii), (ii), (iv), (i) 10. Arrange the following landmarks according to their occurrence on the time scale from 1700 to 2000. (i) Information and computers (ii) Sanitation and medical (iii) Industrial epoch (iv) Biotechnology Choose the correct option. (a) (ii), (iv), (i), (iii) (b) (i), (iii), (iv), (ii) (c) (ii), (iii), (iv), (i) (d) (iii), (ii), (iv), (i) Ans. (d) (iii), (ii), (iv), (i) 11. What is the real wealth of a country?

(a) Its climate (b) Its people (c) Its natural resources (d) Its flora and fauna Ans. (b) Its people 12. Why are the people of a country considered as ‘actual resources’? (a) Because they inhabit that land. (b) Because they are the citizens of that country. (c) Because they make use of the country’s resources and decide its policies. (d) Because they discover the resources within their country. Ans. (c) Because they make use of the country’s resources and decide its policies. 13. Identify the Push and Pull factors respectively. (a) Better job opportunities and Stability (b) Natural disasters and Pleasant climate (c) Pleasant climate and Better living conditions (d) Unemployment and Epidemics Ans. (b) Natural disasters and Pleasant climate 14. Who gave the following remark on the distribution of population in the world? “Asia has many places where people are few and few places where people are many.” (a) George B. Cressey

(b) Thomas Malthus (c) Ralph Waldo Emerson (d) John Graunt Ans. (a) George B. Cressey 15. How did science and technology impact population growth across the world? (a) There was constant growth in birth rates. (b) There was rapid decline in death rates. (c) The old age population was not much healthier. (d) Infant healthcare facilities were not improved. Ans. (b) There was rapid decline in death rates. 16. What has been the proportion of population increase in the 20th century? (a) Population has increased two-fold. (b) Population has increased three-fold. (c) Population has increased four-fold. (d) Population has increased five-fold. Ans. (c) Population has increased four-fold. 17. Which of these statements is correct about the world population growth? (a) It took less than a million years for the human population to attain the one million mark. (b) It took more than a million years for the human population to attain the one million mark. (c) It took only 10 years for the population to rise from 5 billion to 6 billion.

(d) It took only 12 years for the population to rise from 5 billion to 6 billion. Ans. (d) It took only 12 years for the population to rise from 5 billion to 6 billion. 18. Which of these countries will take more time to double their population? (a) Pakistan (b) India (c) Australia (d) Bangladesh Ans. (c) Australia 19. In which type of countries is the population growth moving towards explosion? (a) Developing countries (b) Developed countries (c) Least developed countries (d) Countries with large aboriginal/tribal pockets Ans. (a) Developing countries 20. Which countries does Oceania include? (a) All island countries in the Pacific Ocean (b) Australia, New Zealand and Fiji (c) The islands of Southeast Asia (d) Java, Sumatra, Borneo and Australia Ans. (b) Australia, New Zealand and Fiji 21. What were the consequences of HIV/AIDS epidemics in Africa, some parts of the Commonwealth of Independent

States and Asia? (a) Reduced death rates and average life expectancy (b) Increased death rates and reduced average life expectancy (c) Slowed down population growth (d) Both (b) and (c) Ans. (d) Both (b) and (c) 22. Which continent has the highest density of population? (a) Asia (b) Africa (c) North America (d) South America Ans. (a) Asia 23. If the area of a region X is 200 sq. km. and the population is 1,80,000 persons, what will be the population density? (a) 800 persons/sq. km. (b) 850 persons/sq. km. (c) 900 persons/sq. km. (d) 950 persons/sq. km. Ans. (c) 900 persons/sq. km. 24. Which of these are the most densely populated areas of the world? (a) Hilly areas with pleasant climate (b) River valleys (c) Desert oases (d) Coastal areas

Ans. (b) River valleys 25. In which type of areas do people prefer living? (a) Wetlands (b) Hilly areas (c) Highlands (d) Flat plains and gentle slopes Ans. (d) Flat plains and gentle slopes 26. Why are mountain and hilly areas less populated? (a) They hinder the development of transport network. (b) They do not favour agricultural and industrial development. (c) Both (a) and (b) (d) They favour agricultural and industrial development. Ans. (c) Both (a) and (b) 27. Which of these is a scarcely populated region of India? (a) Coastal plains (b) Himalayas (c) Hills of the Northeast (d) Chota Nagpur Plateau Ans. (b) Himalayas 28. Which of these places would support dense human habitation? (a) Hot or cold deserts (b) Areas with heavy rainfall (c) Areas with less rainfall

(d) Places with not much seasonal variation Ans. (d) Places with not much seasonal variation 29. Why was the Mediterranean region inhabited from early periods in history? (a) Because of its latitudinal location (b) Because of its pleasant climate (c) Because of its proximity to Europe (d) Because it was close to the important trading centres Ans. (b) Because of its pleasant climate 30. Which type of soil supports intensive agriculture and allied activities? (a) Fertile loamy soil (b) Laterite soil (c) Clayey soil (d) Peaty soil Ans. (a) Fertile loamy soil 31. Which of these states in India are thinly populated due to poor soil? (a) Madhya Pradesh (b) Assam (c) Rajasthan (d) Haryana Ans. (c) Rajasthan 32. How are mining and industries linked to population density?

(a) They generate revenue. (b) They generate employment. (c) They are fertile areas. (d) They are located in areas having a pleasant climate. Ans. (b) They generate employment 33. What has made the Katanga Zambia, a densely populated region in Africa? (a) Moderate climate (b) Fertile soil (c) Rich copper belt (d) Petroleum reserves Ans. (c) Rich copper belt 34. Why do cities grow in size? (a) People do not adopt family planning measures. (b) Families earn well and so have big families. (c) Cities have well-planned housing facilities. (d) Cities attract rural migrants because of its various pull factors. Ans. (d) Cities attract rural migrants because of its various pull factors. 35. Which of these statements is incorrect for industrial belts? (a) They have good transport facilities. (b) They provide job opportunities to factory workers. (c) There are no problems related to any type of pollution. (d) They provide job/earning opportunities to shopkeepers,

teachers doctors etc. Ans. (c) There are no problems related to any type of pollution. 36. From which type of places do people often move away? (a) Places which have religious significance (b) Places where there is social and political unrest (c) Places where there are many high-opportunity (d) Places close to forested areas Ans. (b) Places where there is social and political unrest. 37. Which of these statements is incorrect for population growth? (a) The population growth refers to the change in number of inhabitants of a territory during a specific period of time. (b) The population growth rate may be positive or negative. (c) Population growth/change in an area is an important indicator of economic development, social upliftment, etc. (d) It is always expressed in terms of absolute numbers. Ans. (d) It is always expressed in terms of absolute numbers. 38. The formula for calculating crude birth rate is: CBR = What does ‘Bi’ and ‘P’ stand for in this formula? (a) ‘Bi’ for Births during the year and ‘P’ for Population (b) ‘Bi’ for Births during the year and ‘P’ for Percentile (c) ‘Bi’ for Live Births during the year and ‘P’ for Percentile (d) ‘Bi’ for Live Births during the year and ‘P’ for Mid-year population of the area

Ans. (d) ‘Bi’ for Live Births during the year and ‘P’ for Mid-year population of the area 39. Who are known as immigrants? (a) They are migrants who move into a new place. (b) They are migrants who move out of a place. (c) Both (a) and (b) (d) None of the above Ans. (a) They are migrants who move into a new place. 40. Which of the following options states the demographic transition theory? (a) The number of people would increase faster than the food supply; any further increase would result in a population crash caused by famine, disease and war. (b) When a small annual rate is applied to a very large population, it will lead to a large population change. (c) Population of any region changes from high births and high deaths to low births and low deaths as society progresses from rural agrarian and illiterate to urban industrial and literate society. (d) None of the above Ans. (c) Population of any region changes from high births and high deaths to low births and low deaths as society progresses from rural agrarian and illiterate to urban industrial and literate society. 41. Which one of these statements is incorrect for Stage 2 of the Demographic Transition theory? (a) Fertility remains high in the beginning but it declines with time. (b) This is accompanied by reduced mortality rate.

(c) Improvements in sanitation and health conditions lead to decline in mortality. (d) The gap in the net addition to population is low. Ans. (d) The gap in the net addition to population is low. 42. Which one of these statements is incorrect for Stage 3 of the Demographic Transition theory? (a) Both fertility and mortality decline considerably. (b) The population is either unstable or grows at a fast rate. (c) The population becomes urbanised, literate and has high technical knowhow. (d) The population deliberately controls the family size. Ans. (b) The population is either unstable or grows at a fast rate. 43. Which of these countries does not belong to the Stage 1 of the demographic transition model? (a) Sri Lanka (b) Rural Bangladesh (c) Tribes in the Amazon (d) Tribes in the sub-Saharan Africa Ans. (a) Sri Lanka 44. Which of these countries does not belong to the Stage 3 of the demographic transition model? (a) Canada (b) Peru (c) Japan (d) USA Ans. (b) Peru

45. Which of these statements is incorrect? (a) In the present times, different countries are at different stages of demographic transition. (b) Demographic changes occur in stages which are collectively known as the demographic cycle. (c) Human beings are extremely flexible and are able to adjust their fertility. (d) People reproduce less to restrict their family size due to epidemics and variable food supply. Ans. (d) People reproduce less to restrict their family size due to epidemics and variable food supply. 46. Consider the following statements according to the Demographic Transition Theory Stage 1: A. High fertility and high mortality because people reproduce more to compensate for the deaths due to epidemics and variable food supply. B. Life expectancy is low and population growth is slow; most of the people are illiterate, engaged in agriculture and have low levels of technology. Choose the correct option. (a) Statement A is partially correct. (b) Statement B is partially correct. (c) Both the statements are incorrect. (d) Both the statements are correct. Ans. (d) Both the statements are correct. 47. Consider the following statements. A. A small increase in population is desirable in a growing economy.

B. Population growth beyond a certain level leads to problems of resource insufficiency. Choose the correct option. (a) Only A is correct. (b) Only B is correct. (c) Both the statements are incorrect. (d) Both the statements are correct. Ans. (d) Both the statements are correct. 48. When a small annual rate is applied to a very large population, result will __________. (a) lead to a large population change (b) lead to low population change (c) lead to constant population change (d) no change or low change in population Ans. (d) No change or low change in population 49. Consider the following statements. A. If the population decreases between two points of time it is known as negative growth of population. B. Population decreases when the birth rate falls below the death rate or people migrate to other countries. Choose the correct option. (a) Only A is correct. (b) Only B is correct.

(c) Both the statements are incorrect. (d) Both the statements are correct and statement B explains statement A. Ans. (d) Both the statements are correct and statement B explains statement A. 50. Match Column I (Regions) with Column II (Density) and choose the correct option with the help of given codes. Column I

Column II

I.

Europe

1.

20 P/Km2

II.

Africa

2.

32 P/Km2

III.

Latin America and the Caribbean

3.

34 P/Km2

IV.

Northern America

4.

43 P/Km2

V.

Asia

5.

05 P/Km2

VI.

Oceania

6.

146 P/Km2

Codes: I II III IV V VI (a) 2 3 4 5 1 6 (b) 3 4 2 1 6 5 (c) 2 4 1 5 6 3 (d) 3 1 2 6 5 4

Ans. (b) I – 3, II – 4, III – 2, IV – 1, V – 6, VI – 5 51. Match Column I (Time Period) with Column II (Factor for population growth in the different periods) and choose the correct option with the help of given codes. Column I

Column II

I.

1700 – 1800

1.

Transportation

II.

1800 – 1900

2.

Information and computers

III.

1900 – 1950

3.

Industrial epoch

IV.

1950 – 2000

4.

Sanitation and medical

Codes: I II III IV (a) 2 3 4 1 (b) 4 3 2 1 (c) 2 4 3 1 (d) 3 1 4 2 Ans. (d) I – 3, II – 1, III – 4, IV – 2 52. Match Column I (Terminology) with Column II (Definitions) and choose the correct option with the help of given codes. Column I I.

Growth of Population

Column II 1.

Population increased by the difference between births and deaths in a particular region between two points of time.

II.

Growth 2. Rate of Population

When the birth rate is more than the death rate between two points of time or when people from other countries migrate permanently to a region.

III. Natural 3. Growth of Population

Change of population in a particular area between two points of time.

IV. Positive 4. Growth of Population

Change of population expressed in percentage.

Codes: I II III IV (a) 3 4 1 2 (b) 4 3 2 1 (c) 2 4 1 3 (d) 3 1 4 2 Ans. (a) I – 3, II – 4, III – 1, IV – 2 53. Match Column I (Regions) with Column II (Factors Influencing Population Distribution) and choose the correct option with the help of given Codes. Column I

Column II

I.

Ganga plains

(i)

Mineral (copper belt)

II.

Katanga Zambia (Africa)

(ii)

Industries

III.

Mediterranean

(iii)

Landform

(iv)

Climate (Pleasant)

Region IV.

Kobe–Osaka (Japan)

I II III IV (a) 3 4 1 2 (b) 4 3 2 1 (c) 3 1 4 2 (d) 3 2 4 1 Ans. (c) I – 3, II – 1, III – 4, IV – 2 54. Match Column I (Concepts) with Column II (Formula) and choose the correct option with the help of given Codes. Column I

Column II

I.

Density of Population

1. Births – Deaths + In Migration – Out Migration

II.

Natural Growth

2.

III. Actual Growth Population IV. Crude Death Rate Codes: I II III IV (a) 3 4 2 1

of 3. 4. Births – Deaths

(b) 3 4 1 2 (c) 4 1 3 2 (d) 3 2 4 1 Ans. (b) I – 3, II – 4, III – 1, IV – 2 55. Which of the following statement is/are correct? (i) Migrants who move into a new place are called immigrants. (ii) The push factor make the place less attractive. Options: (a) Only (i) (b) Only (ii) (c) Both (i) and (ii) (d) Neither (i) nor (ii) Ans. (c) Both (i) and (ii) 56. Which one of the following is an important information given by Sex ratio in a country? (a) Status of women (b) Status of men (c) Both (a) and (b) (d) Do not provide any information, it just a number Ans. (a) Status of women 57. Shortage of housing, high cost of living, paucity of job opportunities and lack of security in cities are examples of which migration of the following settlements? (a) Discourage women to migrate from urban to rural areas. (b) Discourage women to migrate from rural to urban areas.

(c) Discourage men to migrate from urban to rural areas. (d) Discourage men to migrate from rural to urban areas. Ans. (b) Discourage women to migrate from rural to urban areas. 58. Which of the following pyramid has a narrow base and a tapered top showing low birth and death rates? (a) Declining Population (b) Ageing Population (c) Expanding Population (d) Constant Population Ans. (a) Declining Population 59. Who among the following represents the number of people of different age groups? (a) Age composition (b) Age structure (c) Constant population (d) None of the above Ans. (b) Age structure 60. Which one of the following attributes does not distinguish people? (a) Occupation (b) Age (c) Density (d) Sex Ans. (c) Density 61. Which of the following reason describes the best reason

about a deficit of males in European countries? (a) Low Birth Rate (b) High Birth Rate (c) Better Status of women and men dominated out migration (d) Better Status of men Ans. (c) Better Status of women and men dominated out migration 62. Which of the following is true about expanding population Pyramid? (a) Wide Base (b) Narrow Base (c) Broad Base (d) None of these Ans. (a) Wide Base 63. Which of the following country has the lowest Sex ratio? (a) Egypt (b) Qatar (c) Kuwait (d) Iran Ans. (b) Qatar 64. Which one of the following has caused the sex ratio of the United Arab Emirates to be low. (a) Selective migration of male working population (b) High birth rate of males (c) Low birth rate of females (d) High out-migration of females

Ans. (a) Selective migration of working population 65. Favourable sex ratio is found in how many countries? (a) 140 (b) 110 (c) 120 (d) 139 Ans. (d) 139 66. Which among the following is true about Latvia having the highest sex ratio in the world? (a) 85 males per 100 females (b) 37 males per 100 females (c) 108 males per 100 females (d) 90 males per 100 males Ans. (a) 85 males per 100 females 67. Which one of the following is the main feature of Expanding Populations ? (a) A narrow base pyramid and a tapered top showing low birth and death rates. (b) A bell-shaped pyramid and tapered towards the top. (c) A triangular shaped pyramid with a wide base and is typical of less developed countries. (d) A broad base and a tapered top showing low birth and death rates. Ans. (c) A triangular shaped pyramid with a wide base and is typical of less developed countries.

68. Which attribute does not distinguish people? (a) Age (b) Gender (c) Industries (d) Both (a) and (c) Ans. (c) Industries 69. Population ageing is the process by which the share of the older population becomes proportionally larger. This is phenomenon of which century? (a) 18th (b) 19th (c) 20th (d) 21st Ans. (c) 20th 70. Which of the following pair is correctly matched? Country

Type of Pyramid

(a)

Japan

1.

Expanding Population

(b)

Nigeria

2.

Declining Population

(c)

India

3.

Constant Population

(d)

Australia

4.

Constant Population

Ans. (d) Australia, Constant Population 71. Match the column I with Column II and choose the correct options with the help of given Codes. Column I (Country)

Column II (Sex Ratio)

I.

Latvia

1.

Males are in minority

II.

Russia

2.

Declining population

III.

Nigeria

3.

Highest Sex Ratio

IV.

Japan

4.

Expanding population

Codes: I II III IV (a) 2 1 4 3 (b) 3 1 4 2 (c) 4 3 2 1 (d) 1 3 4 2 Ans. (b) I – 3, II – 1, III – 4, IV – 2 72. Which of the following statement is not true regarding the ‘Sex Ratio’? (a) It is the ratio between the number of women and men in the population. (b) In regions where gender discrimination is less, it is unfavourable to women.

(c) More women in the population does not mean they have a better status. (d) On an average, the world population reflects a sex ratio of 102 males per 100 females. Ans. (b) In regions where gender discrimination is less, it is unfavourable to women. 73. This is an indicator of country’s socio-economic development as it reveals the standard of living, social status of females, availability of educational facilities and policies of government. Select the correct option. (a) Proportion of literate population (b) Occupational Structure (c) Rural Urban Composition (d) Rural Urban Sex Composition Ans. (a) Proportion of literate population 74. Consider the following features and choose the correct title after associating them. 1. Agriculture, forestry, fishing and mining are classified as primary activities, manufacturing as secondary, transport,communication and other services as tertiary. 2. The jobs related to research and developing ideas as quaternary activities. 3. The proportion of working population engaged in these four sectors is a good indicator of the levels of this development of a nation. Options: (a) Economic development

(b) Literacy (c) Occupational Structure (d) Selective migration of working population Ans. (c) Occupational Structure 75. Which one of the following figures represents the working age group of the population? (a) 15 to 40 years (b) 15 to 49 years (c) 15 to 50 years (d) 15 to 59 years Ans. (d) 15 to 59 years 76. The age limits that is considered for measuring the literacy rate of a country: (a) 5 years (b) 6 years (c) 7 years (d) 12 years Ans. (c) 7 years 77. Which of the following is NOT true about Rural Urban Composition? (a) The criteria for differentiating rural and urban population varies from country to country. (b) Rural areas are those where people are engaged in primary activities.

(c) Urban areas are those when majority of the working population is engaged in non-primary activities. (d) The division of population into rural and urban is based on the study. Ans. (d) The division of population into rural and urban is based on the study. 78. Age-sex structure is being represented by which diagram? (a) Line graph (b) Bar graph (c) Pie diagram (d) Pyramid Ans. (d) Pyramid 79. Which of the following is not characteristics of population in developed countries? (a) Share of older people is large (b) Population in higher age group has increased due to increased life expectancy (c) The proportion of children has declined due to reduction in birth rates (d) Level of economic development Ans. (d) Level of economic development 80. Which of the following type of population pyramid is reflected by Australia? (a) Constant

(b) Expanding (c) Declining (d) None of these Ans. (a) Constant 81. Which of the following pair is correct about world’s sex ratio ? (a) 102 males per 100 females. (b) 103 males per 100 females. (c) 104 males per 100 females. (d) 105 males per 100 females. Ans. (a) 102 males per 100 females. 82. The age-sex pyramid of Nigeria is a triangular shaped pyramid with a wide base and is typical of less developed countries. These have larger populations in lower age groups due to high birth rates. Which among the following country’s pyramid would have the same pyramid? (a) Bangladesh and Mexico (b) Bangladesh and Australia (c) Australia and Mexico (d) Denmark and Mexico Ans. (a) Bangladesh and Mexico 83. The ageing population has aged more than: (a) 45 years (b) 55 years (c 65 years

(d) 60 years Ans. (d) 60 years 84. Population composition is also called: (a) Sex composition (b) Age structure (c) Demographic structure (d) Literacy rate structure Ans. (c) Demographic structure 85. In human development, sustainability means continuity in the availability of ____________. (a) natural resources (b) raw materials (c) capital (d) opportunities Ans. (d) opportunities 86. The human development index measures ___________. (a) technological and industrial developments (b) attainments in human development (c) poverty and resource distribution (d) shortfall in human development Ans. (b) attainments in human development 87. In human development, empowerment means to have the power to ____________. (a) make choices

(b) purchase goods (c) exercise one’s veto power (d) obtain welfare benefits Ans. (a) make choices 88. Many of the countries with a high human development score are located in __________and represent the industrialised western world. (a) USA (b) Argentina (c) Europe (d) Canada Ans. (c) Europe 89. A score of _________ would be considered very high level of human development. (a) 0.550 (b) 0.699 (c) 0.983 (d) 0.549 Ans. (c) 0.983 90. Arrange these countries in descending order of their rank in the Human Development Report of 2020. (i) Switzerland (ii) Hong Kong, (China) SAR (iii) Ireland (iv) Norway

Choose the correct option. (a) (iv), (ii), (iii), (i) (b) (iv), (iii), (i), (ii) (c) (ii), (iii), (iv), (i) (d) (iii), (i), (iv), (ii) Ans. (b) (iv), (iii), (i), (ii) 91. Arrange these countries in ascending order of their rank in the Human Development Report of 2020. (i) Netherlands (ii) Denmark (iii) Australia (iv) Germany Choose the correct option. (a) (i), (iv), (iii), (ii) (b) (iii), (ii), (iv), (i) (c) (iv), (iii), (i), (ii) (d) (ii), (i), (iii), (iv) Ans. (d) (ii), (i), (iii), (iv) 92. Arrange the following statements in a sequential order to show the life of young girls who are not sent to school or are early school dropouts. (i) Many opportunities will be lost to these young girls when they grow up. (ii) These school dropouts look after their younger siblings at home and help in domestic work. (iii) Many communities do not stress on the importance of sending

its girl children to school. (iv) Most of the early school dropouts are girl children. Choose the correct option. (a) (i), (iv), (iii), (ii) (b) (iii), (ii), (iv), (i) (c) (ii), (iii), (iv), (i) (d) (iii), (iv), (ii), (i) Ans. (d) (iii), (iv), (ii), (i) 93. Arrange the following statements for a city X, as a progression of growth and development. (i) However, some facilities like housing and other basic facilities have remained the same. (ii) This increase in population shows that the city has grown. (iii) This shows that this growth in numbers has not been accompanied by development. (iv) The population of a City X has grown from one lakh to two lakhs over a period of time. Choose the correct option. (a) (iv), (ii), (i), (iii) (b) (i), (iii), (iv), (ii) (c) (ii), (iii), (iv), (i) (d) (iii), (ii), (iv), (i) Ans. (a) (iv), (ii), (i), (iii) 94. Arrange the following statements on ‘development’ in a logical sequence. (i) This means that development cannot take place unless there is

an increment or addition to the existing conditions. (ii) Yet, positive growth does not always lead to development. (iii) Development means a qualitative change which is always value positive. (iv) Development occurs when there is a positive change in quality. Choose the correct option: (a) (i), (iv), (iii), (ii) (b) (iv), (ii), (i), (iii) (c) (ii), (iv), (iii), (i) (d) (iii), (i), (ii), (iv) Ans. (d) (iii), (i), (ii), (iv) 95. Who are Dr. Mahbub-ul-Haq and Prof Amartya Sen? (a) South Asian sociologists (b) South Asian anthropologists (c) South Asian economists (d) South Asian demographers Ans. (c) South Asian economists 96. What did Nobel Laureate Prof Amartya Sen see as the main objective of development? (a) An increase in the purchasing power (b) An increase in the choices available (c) An increase in the capabilities of the people (d) An increase in freedom (or decrease in unfreedom) Ans. (d) An increase in freedom (or decrease in unfreedom) 97. What is the contribution of Dr. Mahbub-ul-Haq?

(a) Created the concept of GDP (b) Created the Human Development Index (c) Created the concept of GNH (d) Created the poverty index Ans. (b) Created the Human Development Index 98. What is the score of the high development index? (a) Above 0.6 (b) Above 0.7 (c) Above 0.8 (d) Above 0.9 Ans. (c) Above 0.8 99. Which aspect of development is not a part of the concept of Gross National Happiness? (a) Spiritual (b) Non-material (c) Qualitative (d) Material Ans. (d) Material 100. If the population of a city grows over a period of time but basic facilities like housing, provision of basic services and other characteristics remain the same, then has this growth been accompanied by development? (a) Yes (b) Only to a small extent (c) No, not at all

(d) Cannot say Ans. (c) No, not at all 101. Which idea of human development was clearly spelt out for the first time in the late eighties and early nineties? (a) The bigger the economy of the country, the more developed it was, even though this growth did not really mean much change in the lives of most people. (b) The human poverty index is related to the human development index. (c) The quality of life people enjoy in a country, the opportunities they have and freedoms they enjoy, are important aspects of development. (d) There is a link between the level of corruption or political freedom and human development in a particular region. Ans. (c) The quality of life people enjoy in a country, the opportunities they have and freedoms they enjoy, are important aspects of development. 102. What is the basic goal of human development? (a) To create conditions where people can live meaningful lives (b) To utilise natural resources for material development (c) To make policies for industrial development and employment (d) To control mortality rates Ans. (a) To create conditions where people can live meaningful lives 103. Which of the following index is more revealing than the human development index ? (a) Unemployment Index

(b) Human Poverty Index (c) Gross National Happiness (d) Infant Survival Index Ans. (b) Human Poverty Index 104. Which of these students is living a meaningful life? (a) Studying in college and following a healthy daily routine (b) Going to school and watching television most of the time (c) Doing home schooling, a foodie and lazy (d) Studying, partying and relaxing Ans. (a) Studying in college and following a healthy daily routine 105. Which of these are the key areas in human development? (a) Access to materialist product (b) Access to resources, health and education (c) Access to electricity (d) Access to jobs and financial security Ans. (b) Access to resources, health and education 106. Which concepts form the four pillars of human development? (a) Equity, sustainability, productivity and empowerment (b) Employment, education, health and housing (c) Resources, productivity, justice and education (d) Stability, employment, security and health Ans. (a) Equity, sustainability, productivity and empowerment 107. Which of these statements is not correct for equity from the perspective of human development?

(a) It refers to making equal access to opportunities available to everybody. (b) The opportunities available to people must be equal irrespective of their gender, race, income or caste. (c) It refers to social justice for the downtrodden in the society. (d) The choices of backward groups get limited by not having access to knowledge. Ans. (c) It refers to social justice for the downtrodden in the society. 108. What leads to better work efficiency from the perspective of human development? (a) Paying productivity linked incentive/bonus leads to better work efficiency (b) Providing better machines and training leads to better work efficiency (c) Efforts to improve the working and living conditions of the people (d) Efforts to increase the knowledge of the people or provide better health facilities Ans. (d) Efforts to increase the knowledge of the people or provide better health facilities 109. Which of the following ranks the country from 0 to 1 from its record in the key areas of human development? (a) Gross National Happiness (b) Infant Survival Index (c) Human Development Index (d) Unemployment Index Ans. (c) Human Development Index

110. Which indicator is chosen to assess health for ascertaining human development? (a) Mortality Rate (b) Life expectancy at birth (c) Birth Rate (d) Maternal mortality rate Ans. (b) Life expectancy at birth 111. Which of these indicator(s) represent(s) access to knowledge? (a) The number of adults who are able to read and write (b) The number of children enrolled in schools (c) Thee student enrollment and the dropout ratio (d) The adult literacy rate and the gross enrollment ratio Ans. (d) The adult literacy rate and the gross enrollment ratio 112. How is the access to resources measured? (a) In terms of purchasing power (in U.S. dollars) (b) In terms of spending power (in U.S. dollars) (c) Both (a) and (b). (d) None of the above Ans. (a) In terms of purchasing power (in U.S. dollars) 113. Which index development?

measures

(a) Unemployment rate (b) Human poverty index (c) Illiteracy rate

the

shortfall

in

human

(d) Mortality rate Ans. (b) Human poverty index 114. Which approach argues for higher government expenditure on education, health, social secondary and amenities? (a) Income Approach (b) Capability Approach (c) Welfare Approach (d) Basic Needs Approach Ans. (c) Welfare Approach 115. What are the two important indices used by the UNDP to measure human development? (a) Unemployment and income indices (b) Human development and the human poverty indices (c) Infant Survival and Human Development indices (d) Human development and unemployment indices Ans. (b) Human development and the human poverty indices 116. Which of these statements is incorrect about the human poverty index? (a) It is a non-income measure. (b) Often the human poverty index is more revealing than the human development index. (c) Often the human poverty index is less revealing than the human development index. (d) This index measures the shortfall in human development. Ans. (c) Often the human poverty index is less revealing than the

human development index. 117. Who initially proposed the basic needs approach? (a) International Labour Organisation (ILO) (b) Dr Mahbub-ul-Haq (c) Prof Amartya Sen (d) The King of Bhutan Ans. (a) International Labour Organisation (ILO) 118. What was India’s rank in Human Development Index as per Human Development Report, 2020? (a) 120th rank (b) 126th rank (c) 131st rank (d) 135th rank Ans. (c) 131st rank 119. India is lagging behind so many countries in HDI. Which of these is not the correct reason for India’s low ranking? (a) Lower life expectancy (b) Greater gender inequality (c) Increasing income inequalities (d) Increasing labour force participation of women Ans. (d) Increasing labour force participation of women 120. Which of the following options mentions the features of countries that recorded low levels (below 0.549) of human development (2020 Report)? (a) Most of these countries have been going through political

turmoil and social instability in the form of civil war, famine or a high incidence of diseases. (b) Some countries from this group were former colonies. (c) Many of these countries have emerged after the break-up of the erstwhile Soviet Union in 1990. (d) Most of these are countries which have emerged in the period after the Second World War. Ans. (a) Most of these countries have been going through political turmoil and social instability in the form of civil war, famine or a high incidence of diseases. 121. Identify the level of human development with the help of the features given in the two statements. A. Many of these countries have been rapidly improving their human development score by adopting more people-oriented policies and reducing social discrimination. B. Many in this group have faced political instability and social uprisings at some point of time in their recent history. Choose the correct option. (a) Very high level (b) High level (c) Medium level (d) Low level Ans. (c) Medium level 122. Read the statement given below. ‘Often people tend to blame low levels of human development on the culture of the people; for example, X country has lower human development because its people follow Y religion, or belong to Z community.’

Choose the correct option. (a) Such statements are misleading. (b) Largely such statements are true. (c) Such statements are based on authentic researches, hence true. (d) Such results are based on ethnographic studies, might be true. Ans. (a) Such statements are misleading. 123. What are the six basic needs identified and proposed by the ILO? (a) Health, employment

education,

food,

water

supply,

security,

and

(b) Medical aid, education, food security, water supply, fresh air, and housing (c) Medical aid, minimum wages, education, food, water supply, and housing (d) Health, education, food, water supply, sanitation, and housing Ans. (d) Health, education, food, water supply, sanitation, and housing 124. Which of these factors is not a hindrance to ‘equity’? (a) Freedom (b) Gender (c) Race (d) Caste Ans. (a) Freedom 125. Which country ranks first in the world in Human Development Index?

(a) Canada (b) Norway (c) Iceland (d) Australia Ans. (b) Norway 126. Which state has the highest HDI? (a) Madhya Pradesh (b) Gujarat (c) Kerala (d) Uttar Pradesh Ans. (c) Kerala 127. Which of these is not a key area on which Human Development Index of a country is determined? (a) Health (b) Happiness (c) Education (d) Access to resources Ans. (b) Happiness 128. Which of these statements is incorrect about Welfare Approach? (a) This approach looks at human beings as beneficiaries or targets of all development activities. (b) People are not participants in development but only passive recipients. (c) People are participants in development and are active recipients.

(d) The government is responsible for increasing levels of human development by maximising expenditure on welfare. Ans. (c) People are participants in development and are active recipients. 129. Which of these statements is incorrect for ascertaining the level of human development? (a) The closer a score is to 1, the greater is the level of human development. (b) A score of 0.983 would be considered very high level of human development. (c) A score of 0.268 would mean a very low level of human development. (d) A score of 0.650 would mean high level of human development. Ans. (d) A score of 0.650 would mean high level of human development. 130. Consider the following statements. A. Places with low levels of human development tend to spend more on defence rather than social sectors. B. These countries tend to be located in areas of political instability and have not been able to initiate accelerated economic development. Choose the correct option. (a) Statement A is correct but statement B is incorrect. (b) Statement B is correct but statement A is incorrect. (c) Both the statements are correct but statement B does not explain statement A. (d) Both the statements are correct and statement B explains

statement A. Ans. (d) Both the statements are correct and statement B explains statement A. 131. Consider the following statements. A. Empowerment means to have the power to make choices and it comes from increasing freedom and capability. B. Good governance and people-oriented policies are required to empower people. Choose the correct option. (a) Statement A is only partially correct. (b) Statement B is only partially correct. (c) Both the statements are correct but statement B does not explain statement A. (d) Both the statements are correct and statement B explains statement A. Ans. (d) Both the statements are correct and statement B explains statement A. 132. Consider the following statements. A. To have sustainable human development, each generation must have the same opportunities. B. All environmental, financial and human resources must be used keeping in mind the future. Choose the correct option. (a) Only A is correct. (b) Only B is correct. (c) Both the statements are partially correct. (d) Both the statements are correct and statement B explains

statement A. Ans. (d) Both the statements are correct and statement B explains statement A. 133. Consider the following statements. A. Very often, people do not have the capability and freedom to make even basic choices due to their inability to acquire knowledge, their material poverty, social discrimination, inefficiency of institutions and other reasons. B. This prevents them from leading healthy lives, being able to get educated or to have the means to live a decent life. Choose the correct option. (a) Only A is correct. (b) Only B is correct. (c) Both the statements are correct. (d) Both the statements are correct and statement B explains statement A. Ans. (d) Both the statements are correct and statement B explains statement A. 134. Consider the following statements. A. In India, a large number of women and persons belonging to socially and economically backward groups drop out of school. B. The choices of these groups get limited by not having access to knowledge. Choose the correct option. (a) Only A is correct. (b) Only B is correct. (c) Both the statements are only partially correct.

(d) Both the statements are correct and statement B explains statement A. Ans. (d) Both the statements are correct and statement B explains statement A. 135. Match Column I (Level of Human Development) with Column II (Number of Countries) and choose the correct option with the help of given codes. Column I

Column II

I.

Very High

1.

53 Countries

II.

High

2.

36 Countries

III.

Medium

3.

66 Countries

IV.

Low

4.

37 Countries

Codes: I II III IV (a) 2 3 4 1 (b) 3 1 4 2 (c) 2 4 1 3 (d) 3 1 2 4 Ans. (c) I-3, II-1, III-4, IV-2 136. Match Column I (Number of Countries) with Column II (Score in Development Index) and choose the correct option with the help of given codes. Column I I.

Below 0.549

Column II 1.

37 Countries

II.

Between 0.550 up to 0.699

2.

66 Countries

III.

Between 0.700 up to 0.799

3.

33 Countries

IV.

Above 0.800

4.

53 Countries

Codes: I II III IV (a) 1 3 4 2 (b) 4 3 2 1 (c) 3 1 4 2 (d) 3 1 2 4 Ans. (b) I-3, II-1, III-4, IV-2 137. Match Column I (Term) with Column II (Meaning) and choose the correct option with the help of given codes. Column I

Column II

I.

Development 1. Making equal access to opportunities available to everybody

II.

Sustainability 2. Ranks countries based on their performance in the key areas of health, education and access to resources

III. Equity

3.

IV. HDI

4. A qualitative change which is always value positive

Codes: I II III IV

Continuity in the availability of opportunities

(a) 3 4 1 2 (b) 4 3 1 2 (c) 2 4 1 3 (d) 3 1 4 2 Ans. (d) I-4, II-3, III-1, IV-2 138. Match Column I (Approaches to Human Development) with Column II (Features) and choose the correct option with the help of given codes. Column I

Column II

I.

Income Approach

1.

Was initially proposed by the International Labour Organisation (ILO).

II.

Welfare

2.

Associated with Prof. Amartya Sen

Approach III.

Basic Needs Approach

3.

One of the oldest approaches to human development

IV.

Capability

4.

Looks at human beings as beneficiaries or targets of all development activities

Approach Codes: I II III IV (a) 3 4 2 1 (b) 3 1 4 2 (c) 4 1 3 2 (d) 3 4 1 2

Ans. (b) I-3, II-4, III-1, IV-2 139. To understand why a particular region keeps reporting low or high levels of human development it is important to look at ______. (a) Pattern of people expenditure on social section (b) Pattern of people, how they earn (c) Pattern of corporate social responsibility (d) Pattern of government expenditure on the social sector Ans. (d) Pattern of government expenditure on the social sector

1. Primitive people living in ______________ survived on hunting. (a) very cold climate (b) extremely hot climate (c) very cold and extremely hot climates (d) wet and moderate climates Ans. (c) very cold and extremely hot climates 2. The seasonal migration of people in search of pastures over vast areas is known as ________________. (a) transhumance (b) livestock rearing (c) animal grazing (d) ranching Ans. (a) transhumance 3. In ______________vegetation is usually cleared by fire. (a) intensive subsistence agriculture (b) primitive subsistence agriculture (c) plantation agriculture (d) commercial grain cultivation

Ans. (b) primitive subsistence agriculture 4. Intensive subsistence agriculture is largely found in _______________regions of monsoon Asia. (a) sparsely populated (b) thinly populated (c) densely populated (d) unevenly populated Ans. (c) densely populated 5. Mixed farming is found in the _________ parts of the world. (a) less developed (b) underdeveloped (c) developing (d) highly developed Ans. (d) highly developed 6. Arrange the following primary activities starting from the most primitive to the most modern. (i) Nomadic herding (ii) Commercial livestock rearing (iii) Domestication of animals (iv) Hunting and gathering Choose the correct option:

(a) (i), (iv), (iii), (ii) (b) (iv), (iii), (i), (ii) (c) (ii), (iv), (iii), (i) (d) (iii), (i), (iv), (ii) Ans. (b) (iv), (iii), (i), (ii) 7. Arrange the following activities in a logical sequence. (i) Do a simple processing of the collected plants and parts. (ii) Gatherers collect valuable plants or various parts of plants. (iii) Sell the products in the market. (iv) Do sorting of collected items. Choose the correct option: (a) (iv), (ii), (i), (iii) (b) (i), (iv), (iii), (ii) (c) (ii), (iii), (iv), (i) (d) (ii), (iv), (i), (iii) Ans. (d) (ii), (iv), (i), (iii) 8. Arrange the steps followed in ‘slash and burn agriculture’. (i) The farmers move to a new patch of land and clear it by slashing and burning (ii) Vegetation is cut, cleared and set on fire (iii) Ashes add to soil fertility and the cleared patch of land is cultivated. (iv) After harvesting the crop for a few years, the field is abandoned.

Choose the correct option: (a) (iii), (i), (iv), (ii) (b) (ii), (iv), (iii), (i) (c) (ii), (iii), (iv), (i) (d) (ii), (i), (iv), (iii) Ans. (c) (ii), (iii), (iv), (i) 9. Arrange the following steps in a sequential order to show the formation and functioning of a co-operative farming society. (i) The co-operative society helps farmers to procure all the important inputs of farming. (ii) The co-operative society helps farmers in processing of quality products and selling the products at the most favourable terms. (iii) They form a co-operative society by pooling in their resources voluntarily. (iv) A group of farmers come together for more efficient and profitable farming. Choose the correct option: (a) (iv), (iii), (i), (ii) (b) (iii), (ii), (i), (iv) (c) (ii), (iii), (iv), (i) (d) (iii), (i), (iv), (ii) Ans. (a) (iv), (iii), (i), (ii) 10. Arrange the sequence of the steps involved in shaft mining. (i) The minerals then go through different stages of processing and refining.

(ii) Minerals are extracted and transported to the surface through these passages using specially designed lifts. (iii) When the ore lies deep below the surface, underground mining/shaft method is used. (iv) Vertical shafts have to be sunk, from where underground galleries radiate to reach the minerals. Choose the correct option: (a) (iii), (iv), (i), (ii) (b) (iii), (iv), (ii), (i) (c) (ii), (iii), (iv), (i) (d) (iii), (ii), (iv), (i) Ans. (b) (iii), (iv), (ii), (i) 11. Which of the following are the oldest known economic activities? (a) Fishing and gathering (b) Farming and gathering (c) Gathering and hunting (d) Fishing and pottery Ans. (c) Gathering and hunting 12. In which type of climatic conditions is gathering practiced? (a) Monsoon type of climate (b) Harsh climatic conditions (c) Hot and wet climate (d) Continental climate Ans. (b) Harsh climatic conditions

13. Which of these statements is not true for gathering and hunting? (a) It often involves primitive societies. (b) It requires a small amount of capital investment. (c) It operates at very low level of technology. (d) The yield per person is fairly moderate and some surplus food is produced. Ans. (d) The yield per person is fairly moderate and some surplus food is produced. 14. Which factor plays the most important role in the selection of animals for domestication? (a) Economic factors (b) Geographical factors (c) Cultural factors (d) Need-based factors Ans. (b) Geographical factors 15. Why has the number of pastoral nomads been decreasing and the areas operated by them shrinking? (a) Due to the impact of modernisation on the nomadic people and changes in lifestyle (b) Due to new settlement plans by different countries and imposition of political boundaries (c) Due to climatic changes, increasing temperatures and decreasing rainfall (d) Due to the twin processes of industrialisation and urbanisation Ans. (b) Due to new settlement plans by different countries and imposition of political boundaries

16. Which of these statements is not correct for commercial livestock rearing? (a) It is a specialised activity in which more than one type of animal is reared. (b) Emphasis is on breeding, genetic improvement, and health care of the animals. (c) Products are processed and packed scientifically and exported to different world markets. (d) Important countries where this is practiced are New Zealand, Australia, Argentina, Uruguay and United States of America. Ans. (a) It is a specialised activity in which more than one type of animal is reared. 17. Which of these statements is not correct about intensive subsistence agriculture, dominated by wet paddy cultivation? (a) Land holdings are very small. (b) Farm yard manure is used to maintain the fertility of the soil. (c) The yield per unit area is low but per labour productivity is high. (d) Use of machinery is limited and most operations are done by manual labour. Ans. (c) The yield per unit area is low but per labour productivity is high. 18. Which of these statements is not correct about intensive subsistence agriculture, dominated by crops other than paddy? (a) Wheat, soyabean, barley and sorghum are grown in northern China, Manchuria, North Korea and North Japan. (b) In India, wheat is grown in the eastern parts of the IndoGangetic plains.

(c) Millets are grown in the dry parts of western and southern India. (d) Irrigation is often used in this type of agriculture. Ans. (b) In India, wheat is grown in the eastern parts of the IndoGangetic plains. 19. Who introduced plantations as a profit-oriented large scale production system? (a) North Americans (b) South-east Asians (c) South Americans (d) Europeans Ans. (d) Europeans 20. Following name of crops are given and choose them plantation crops. (i) Tea (ii) Rubber (iii) Cotton (iv) Mangoes (a) (i) and (ii) only (b) (i) and (iii) only (c) (i), (iii) and (iv) (d) (i), (ii) and (iii) Ans. (d) (i), (ii) and (iii) 21. Which of these statements is not a characteristic/ feature of plantation agriculture? (a) Managerial and technical support (b) Multiple crop specialisation on a single plantation (c) Large estates and large capital investment (d) Cheap labour, and a good system of transportation

Ans. (b) Multiple crop specialisation on a single plantation 22. Who established cocoa and coffee plantations in West Africa? (a) French (b) British (c) Spanish (d) Americans Ans. (a) French 23. Who set up large tea gardens in India and Sri Lanka, and rubber plantations in Malaysia? (a) Dutch (b) Spanish (c) British (d) French Ans. (c) British 24. Where did the Spanish and Americans invest heavily? (a) In rubber plantations in Malaysia (b) In cocoa plantations in Brazil (c) In banana plantations in West Indies (d) In coconut and sugarcane plantations in the Philippines Ans. (d) In coconut and sugarcane plantations in the Philippines 25. Who are still managing some coffee fazendas in Brazil? (a) Europeans (b) Dutch (c) Spanish

(d) French Ans. (a) Europeans 26. Where is commercial grain cultivation practised? (a) In the coastal regions of semi-arid lands of the mid-latitudes (b) In the interior parts of semi-arid lands of the high-latitudes (c) In the interior parts of semi-arid lands of the mid-latitudes (d) In the interior parts of arid lands of the mid-latitudes Ans. (c) In the interior parts of semi-arid lands of the mid-latitudes 27. Which of these is an incorrect statement about extensive commercial grain cultivation? (a) There is high yield per acre but average yield per person. (b) Wheat is the principal crop although other crops are also grown. (c) The size of the farm is very large. (d) Entire operations of cultivation are mechanised. Ans. (a) There is high yield per acre but average yield per person. 28. Which of these options is a mismatch in its name and region/country? (a) The Steppes of Eurasia (b) The Prairies of Canada and America (c) The Velds of Argentina (d) The Downs of Australia Ans. (c) The Velds of Argentina 29. Which of the following crops is associated with mixed farming?

(a) Rice (b) Bajra (c) Rye (d) Pulses Ans. (c) Rye 30. How is soil fertility maintained in mixed farming? (a) Use of mulching method to retain moisture in soil (b) Use of high-yielding variety seeds (c) Use of sprinklers for irrigation to control soil erosion (d) Practising crop rotation and inter-cropping methods Ans. (d) Practising crop rotation and inter-cropping methods 31. Which of these statements is not correct for mixed farming? (a) Mixed farms are large in size. (b) Equal emphasis is laid on crop cultivation and animal husbandry. (c) There is high capital expenditure on farm machinery. (d) There is extensive use of chemical fertilisers and green manures. Ans. (a) Mixed farms are large in size. 32. Why is dairy farming practised mainly near urban and industrial centres? (a) Labour is easily available.

(b) There are facilities of refrigeration. (c) There is neighbourhood market for fresh milk and milk products. (d) Pasteurisation and other preservation processes are available in urban centres. Ans. (c) There is neighbourhood market for fresh milk and milk products. 33. Which is the largest region of commercial dairy farming in the world? (a) Canada (b) North Western Europe (c) South Eastern Australia (d) New Zealand and Tasmania Ans. (b) North Western Europe 34. What is the speciality of Mediterranean agriculture? (a) Grapes (b) Mango (c) Banana (d) All of these Ans. (a) Grapes 35. Which of these statements is incorrect for Mediterranean agriculture?

(a) Best quality wines are produced from high quality grapes. (b) The inferior grapes are dried into raisins and currants. (c) Valuable crops such as fruits and vegetables are grown in winters. (d) Winter fruits and vegetables are not in much demand. Ans. (d) Winter fruits and vegetables are not in much demand. 36. Which flower is a speciality of Netherlands? (a) Tulip (b) Rose (c) Orchid (d) Hollyhock Ans. (a) Tulip 37. Which of these statements is true for market gardening and horticulture? (a) It is both labour and capital intensive. (b) It requires greenhouses and artificial heating in colder regions. (c) There is much emphasis on irrigation, fertilisers and insecticides. (d) All of the above Ans. (d) All of the above 38. In which of the following parts of the world is market gardening and horticulture most developed?

(a) Europe, Argentina, Australia and north eastern United States of America (b) North-west Europe, north eastern United States of America and the Mediterranean regions (c) New Zealand, north-west Europe and Mediterranean regions (d) North-west Europe, north eastern United States of America, Canada and Japan Ans. (b) North-west Europe, north eastern United States of America and the Mediterranean regions 39. What is truck farming? (a) Farming in which trucks are the main means of transport (b) Farming in which all trucks ply round the clock (c) Farming in which farmers specialise in vegetables only (a) Farming in which trucks are used as mobile shops Ans. (c) Farming in which farmers specialise in vegetables only 40. What is meant by factory farming? (a) Machinery manufacturing for market gardening and horticulture (b) Providing specialised transport services for market gardening (c) Livestock, particularly poultry and cattle rearing, done in stalls and pens (d) Manufacturing poultry feed Ans. (c) Livestock, particularly poultry and cattle rearing, done in

stalls and pens 41. What is co-operative farming? (a) A group of experts who help farmers to increase their crop productivity (b) A rural bank that provides loan to farmers (c) An advisory group that guides the farmers on soil fertility (d) A group of farmers form a co-operative society by pooling in their resources voluntarily Ans. (d) A group of farmers form a co-operative society by pooling in their resources voluntarily 42. Which of these statements is not true for surface mining or open-cast mining? (a) It is the easiest and the cheapest way of mining minerals that occur close to the surface. (b) Overhead costs such as safety precautions and equipment is relatively low in this method. (c) The output is moderate and the process is slow. (d) The output is both large and rapid. Ans. (c) The output is moderate and the process is slow. 43. Why are developed economies moving away from mining, processing and refining stages of production? (a) Due to risks involved in mining (b) Due to high labour costs (c) Labour force of developing countries is now striving for higher standard of living. (d) Both (b) and (c) Ans. (d) Both (b) and (c)

44. Which of these statements on transhumance is incorrect? (a) In mountain regions, such as Himalayas, Gaddis and Bhotiyas migrate from plains to the mountains in summers and to the plains from the high altitude pastures in winters. (b) In mountain regions, such as Himalayas, Gujjars and Bakarwals migrate from plains to the mountains in winters and to the plains from the high altitude pastures in summers. (c) In the tundra regions, the nomadic herders move from south to north in summers and from north to south in winters. (d) Movement in search of pastures is undertaken either over vast horizontal distances or vertically from one elevation to another in the mountainous regions. Ans. (b) In mountain regions, such as Himalayas, Gujjars and Bakarwals migrate from plains to the mountains in winters and to the plains from the high altitude pastures in summers. 45. Consider the following statements. Gathering is practised in: A. High latitude zones which include northern Canada, northern Eurasia and southern Chile. B. Low latitude zones such as the Amazon Basin, tropical Africa, Northern fringe of Australia and the interior parts of South-east Asia. Choose the correct option: (a) Statement A is partially correct. (b) Statement B is partially correct. (c) Both the statements are incorrect. (d) Both the statements are completely correct. Ans. (d) Both the statements are completely correct.

46. Consider the following statements. Pastoral nomadism is associated with which of the following region? A. The core region extends from the Atlantic shores of North Africa, eastwards across the Arabian peninsula into Mongolia and Central China. B. The second region extends over the tundra region of Eurasia. C. The third region is in the southern hemisphere where there are small areas in South-west Africa and on the island of Madagascar. Choose the correct option: (a) Only statement A is correct. (b) Only statement B is correct. (c) All the statements are correct. (d) Only statements A and B are correct. Ans. (c) All the statement are correct. 47. Consider the following statements. A. The early hunters used primitive tools made of stones, twigs or arrows. B. The number of animals killed was limited. . Choose the correct option: (a) Only A is correct. (b) Only B is correct. (c) Both the statements are incorrect. (d) Both the statements are correct and statement B explains statement A. Ans. (d) Both the statements are correct and statement B explains

statement A. 48. Consider the following statements. A. The people in the coastal areas still catch fish though fishing has experienced modernisation due to technological progress. B. Many species, now have become extinct or endangered due to illegal hunting. Choose the correct option: (a) Only A is correct. (b) Only B is correct. (c) Both the statements are incorrect. (d) Both the statements are correct. Ans. (d) Both the statements are correct. 49. Consider the following statements. A. Dairy farming highly labour intensive. B. It involves rigorous care in feeding and milching and there is no off season during the whole year. Choose the correct option: (a) Only A is correct. (b) Only B is correct. (c) Both the statements are incorrect. (d) Both the statements are correct and statement B explains statement A. Ans. (d) Both the statements are correct and statement B explains statement A. 50. Match Column I (Specific name for shifting cultivation) with Column II (Region/Country) and choose the correct

option with the help of given codes. Column I

Column II

I.

Jhuming

1.

Indonesia and Malaysia

II.

Milpa

2.

Brazil

III.

Ladang

3.

Vietnam

IV.

Roca

4.

North eastern states of India

V.

Ray

5.

Central America and Mexico

Codes: I II III IV V (a) 2 3 4 5 1 (b) 3 4 2 1 5 (c) 4 5 1 2 3 (d) 3 1 2 4 5 Ans. (c) I – 4, II – 5, III – 1, IV – 2, V – 3 51. Match Column I (Regions) with Column II (Livestock Animals) and choose the correct option with the help of given codes. Column I

Column II

I.

Tropical Africa

1. Reindeer

II.

Tibet and Andes

2. Sheep, goats and camel

(Mountainous

areas) III. Arctic and Sub-Arctic areas

3. Cattle

IV. Sahara and Asiatic deserts

4. Yak and llamas

Codes: I II III IV (a) 2 3 4 1 (b) 3 4 1 2 (c) 2 4 3 1 (d) 3 1 4 2 Ans. (b) I – 3, II – 4, III – 1, IV – 2 52. Match Column I (Types of Agriculture) with Column II (Features) and choose the correct option with the help of given codes. Column I I.

Subsistence

Column II 1.

Practised in the interior parts of semiarid lands of the mid-latitudes on very large farms

2.

Moderate size farms found in the highly developed parts of the world, involving high capital expenditure on farm machinery, fertiliser, etc.

3.

Farming areas consume nearly all of the products grown locally

agriculture II.

Plantation agriculture

III.

Extensive commercial

grain cultivation IV. Mixed

4.

farming

Large estates, large capital investment, scientific methods of cultivation, single crop specialisation, etc.

Codes: I II III IV (a) 3 4 1 2 (b) 4 3 2 1 (c) 2 4 1 3 (d) 3 1 4 2 Ans. (a) I – 3, II – 4, III – 1, IV – 2 53. Match Column I (Types of Agriculture) with Column II (Regions where practiced) and choose the correct option with the help of given Codes. Column I I.

Primitive

Column II 1.

Malaysia, West Indies, Indonesia, India, etc.

2.

Eurasian Steppes, the Canadian and American Prairies, the Pampas of Argentina, the Velds of South Africa, etc.

3.

Practised by many tribes in the tropics, especially in Africa, south and central America and south east Asia.

subsistence agriculture II.

Intensive subsistence agriculture

III.

Plantation agriculture

IV.

Extensive

4.

commercial

Europe, Eastern North America, parts of Eurasia, etc.

grain cultivation V.

Mixed farming

North-western

5.

Largely found in the densely populated regions of monsoon Asia

Codes: I II III IV V (a) 3 4 2 1 5 (b) 4 3 2 5 1 (c) 3 5 1 2 4 (d) 3 1 4 2 5 Ans. (c) I – 3, II – 5, III – 1, IV – 2, V – 4 54. Match column I (Types of Agriculture) with column II (Main crops) and choose the correct option with the help of given codes. Column I I.

Intensive

Column II 1.

Wheat (main crop) and other crops such as corn, barley, oats and rye

2.

Wet paddy (rice)and other crops such as wheat, soyabean, barley and sorghum

3.

Wheat, barley, oats, rye, maize, fodder and root crops

subsistence agriculture II.

Plantation agriculture

III.

Extensive commercial

grain cultivation IV.

Mixed Farming

4.

Tea, coffee, cocoa, rubber, cotton, oil palm, sugarcane, bananas, etc.

Codes: I II III IV (a) 4 3 2 1 (b) 3 4 1 2 (c) 4 1 3 2 (d) 2 4 1 3 Ans. (d) I – 2, II – 4, III – 1, IV – 3 55. Secondary activities add value to natural resources by _______ raw materials into valuable products. (a) refining (b) transforming (c) manufacturing (d) developing Ans. (b) transforming 56. Most of the _______ countries still ‘manufacture’ using primitive methods. (a) developed (b) third World (c) developing (d) highly Developed Ans. (b) third World

57. Major concentrations of modern manufacturing that have flourished in a few places, cover less than _______ of the world’s land area. (a) 25 per cent (b) 15 per cent (c) 10 per cent (d) 5 per cent Ans. (c) 10 per cent 58. Earlier coal was the main source of energy, today _______ and petroleum are also important sources of energy for many industries. (a) wind energy (b) hydroelectricity (c) tidal energy (d) solar energy Ans. (b) hydroelectricity 59. Governments adopt _______ to promote ‘balanced’ economic development and hence set up industries in particular areas. (a) ‘national policies’ (b) ‘subsidised policies’ (c) ‘import policies’ (d) ‘regional policies’ Ans. (d) ‘regional policies’ 60. Countries like India, _______ have developed labour intensive small-scale manufacturing in order to provide employment to their population.

(a) New Zealand, Indonesia and Brazil (b) China, Indonesia and Brazil (c) United States of America, Indonesia and China (d) China, Indonesia and Germany Ans. (d) China, Indonesia and Germany 61. Assembly-line production involves _______ . (a) less capital (b) small capital (c) large capital (d) foreign exchange Ans. (c) large capital 62. Drying, fermenting and pickling are some ancient _______ techniques. (a) brewing (b) canning (c) flavouring (d) preserving Ans. (d) preserving 63. Wooden furniture industry is an example of _______ industries. (a) chemical based (b) agro based (c) forest based (d) mineral based Ans. (c) forest based

64. A pulp mill is associated with _______ . (a) paper industry (b) furniture industry (c) lac industry (d) plastic industry Ans. (a) paper industry 65. Ivory is obtained from _______ . (a) rhino horns (b) stag antlers (c) elephant’s tusks (d) reindeer antlers Ans. (c) elephant’s tusks 66. The industry whose products are used to make other goods by using them as raw materials are called _______ . (a) secondary industries (b) basic industries (c) foot loose industries (d) household industries Ans. (b) basic industries 67. Industries producing breads and biscuits, tea, soaps and toiletries, paper for writing, televisions, etc. are _______ . (a) secondary industries (b) basic industries (c) consumer goods industries (d) traditional industries

Ans. (c) consumer 68. Public Sector Industries are owned and managed by _______ . (a) individual investors (b) joint stock companies (c) co-operatives (d) government Ans. (d) government 69. In capitalist countries, industries are generally owned _________. (a) privately (b) by the government (c) by co-operatives (d) by joint stock companies Ans. (a) privately 70. Arrange the following steps involved in iron and steel industry as a process starting from the beginning to the end. i. Pig iron is converted into steel by adding strengthening materials like manganese. ii. Iron is extracted from iron ore by smelting in a blast furnace. iii. Carbon (coke) and limestone is added during smelting. iv. The molten iron is cooled and moulded to form pig iron. Choose the correct option. (a) (ii), (i), (iv), (iii) (b) (iv), (ii), (i), (iii) (c) (ii), (iii), (iv), (i)

(d) (iii), (i), (iv), (ii) Ans. (c) (ii), (iii), (iv), (i) 71. Arrange the following activities of the cotton textile industry, in a logical sequence. i. Finishing of fabric ii. Weaving of fabric iii. Cultivating cotton iv. Spinning of yarn Choose the correct option. (a) (iv), (ii), (i), (iii) (b) (iii), (ii), (iv), (i) (c) (ii), (iii), (iv), (i) (d) (iii), (iv), (ii), (i) Ans. (d) (iii), (iv), (ii), (i) 72. Choose the correct statement: A. The developed regions of Europe, North America, Japan and Australia Provide large global Markets. B. The density populated region of south and south-east Asia also provide large markets. (a) Only Statement A is correct (b) Only statement B is correct (c) Both statement A and B are correct (d) Neither A is Correct, Nor B is correct Ans. (c) Both statement A and B are correct 73. Which of these industries have a global market?

(a) Aircraft and arms manufacturing (b) Electronics and timber (c) Heavy machinery and fertiliser (d) Ceramic and cotton Ans. (a) Aircraft and arms manufacturing 74. Which one of these is a vital factor for agro-processing and dairy products industry to be located closer to the source of the raw material? (a) Quantity (b) Cost/value (c) Perishability (d) Durability Ans. (c) Perishability 75. What is the consequence of increased mechanisation, automation and flexibility of industrial processes? (a) No dependence on labourers (b) Heavy dependence on cheap labour (c) Increased dependence on labourers (d) Reduced dependence on labourers Ans. (d) Reduced dependence on labourers 76. Identify the feature of ‘Foot Loose Industries’. (a) They can be located only in very specific and limited places. (b) They employ a large labour force. (c) They are not dependent on any specific raw material. (d) They do produce in large quantity.

Ans. (c) They are not dependent on any specific raw material. 77. Which key factor of these is a responsible for the concentration of industries in Western Europe and eastern North America? (a) A highly developed transport system (b) Rich deposits of essential minerals (c) Availability of hydroelectricity (d) Availability of cheap labour Ans. (a) A highly developed transport system 78. Which of these is an important need for industries for the exchange and management of information? (a) Transportation (b) Communication (c) Road network (d) Railway network Ans. (b) Communication 79. Why capital and transportation do not wield much influence in cottage manufacturing? (a) They have medium commercial importance and locally devised tools. (b) They have high commercial significance but locally devised tools. (c) They have low commercial significance and mostly locally devised tools. (d) They have fairly high commercial significance and locally devised tools. Ans. (c) They have low commercial significance and mostly locally

devised tools. 80. Which one of these products does not belong to the category of cottage industry? (a) Wooden figurines (b) Clay and stone pottery (c) Bamboo furniture (d) Steel utensils Ans. (d) Steel utensils 81. Which of these statements is not correct for small scale manufacturing? (a) Local raw material is used. (b) Simple power-driven machines and semi-skilled labour used. (c) It provides employment and raises local purchasing power. (d) Only manually operated machines are used in order to provide employment to people. Ans. (d) Only manually operated machines are used in order to provide employment to people. 82. Which type of manufacturing involves enormous energy? (a) Small scale (b) Medium scale (c) Large scale (d) Electronics Ans. (c) Large scale 83. Which one of these industries does not use non-ferrous metallic minerals as raw material? (a) Aluminium

(b) Copper (c) Iron and steel (d) Jewellery Ans. (c) Iron and steel 84. To which type of industry do ivory products belong? (a) Agro based industry (b) Mineral based industry (c) Animal based industry (d) Forest based industry Ans. (c) Animal based industry 85. Which one of these statements is not correct about traditional large-scale Industrial regions? (a) These are based on heavy industry. (b) These are often located far away from coal-fields. (c) These are engaged in metal smelting, heavy engineering, chemical manufacture or textile production. (d) These industries are now known as smokestack industries. Ans. (b) These are often located far away from coal-fields. 86. Why is the iron and steel industry also called a heavy industry? (a) This is because it uses large quantities of bulky raw materials (b) This is because its products are also heavy (c) Both (a) and (b) are correct (d) Both (a) and (b) are incorrect Ans. (c) Both (a) and (b) are correct

87. Which one of the followings is a major hazards of traditional large-scale industrial regions? (a) Congested housing (b) Environmental pollution (c) Outmigration (d) Ability for self reliance Ans. (b) Environmental pollution 88. In which country is the Ruhr coal-field? (a) France (b) United Kingdom (c) Germany (d) U.S.A. Ans. (c) Germany 89. The Ruhr region is responsible for _________ per cent of Germany’s total steel production. (a) 50 (b) 60 (c) 70 (d) 80 Ans. (d) 80 90. By which process is iron ore extracted in the blast furnace? (a) Combustion Process (b) Melting Process (c) Casting Process

(d) Smelting Process Ans. (d) Smelting Process 91. The molten iron when cooled and moulded, is converted into ________. (a) Pig iron (b) Wrought iron (c) Cast iron (d) Pure iron Ans. (a) Pig iron 92. What is added to the iron ore during the process of smelting? (a) Carbon and manganese (b) Carbon and limestone (c) Carbon and sulphur (d) Coke and coal Ans. (b) Carbon and limestone 93. What is the key factor for the location of mini steel mills? (a) Access to mines (b) Proximity to ports (c) Good network (d) Access to markets Ans. (d) Access to markets 94. Which of these is not correct for mini steel mills? (a) They are less expensive to build. (b) They are less expensive to operate.

(c) Scrap metal is not an essential requirement. (d) They can be located near markets. Ans. (c) Scrap metal is not an essential requirement. 95. Which of these regions produce the maximum iron and steel in the U.S.A.? (a) Appalachian Region, Great Lake Region, Atlantic Coast Region (b) Appalachian Region, Alabama Region, Great Lake Region (c) Canadian Shield Region, Alabama Region, Great Lake Region (d) Appalachian Region, Great Lake Region, Pacific Coast Region Ans. (a) Appalachian Region, Great Lake Region, Atlantic Coast Region 96. Which iron and steel centre is now losing ground and is now called the `rust bowl’ of U.S.A.? (a) Cleveland (b) Duluth (c) Pittsburgh (d) Morrisville Ans. (c) Pittsburgh 97. In which country are the important steel centres: Scunthorpe, Port Talbot, Birmingham and Sheffield? (a) Canada (b) Germany (c) United Kingdom (d) Russia Ans. (c) United Kingdom

98. In which country are the important steel centres: Moscow, St. Petersburgh, Lipetsk and Tula? (a) France (b) Ukraine (c) Germany (d) Russia Ans. (d) Russia 99. Which of these statements is not correct for the powerloom industry? (a) The volume of production increases (b) It introduces machines (c) It becomes less labour intensive (d) It becomes more labour intensive Ans. (d) It becomes more labour intensive 100. Which one of the following countries accounts for nearly half of the world’s cotton imports? (a) Egypt (b) Europe (c) India (d) U.S.A. Ans. (b) Europe 101. Which type of fibre has given a stiff competition to the cotton textile across the world? (a) Woollen fibre (b) Jute fibre (c) Silk fibre

(d) Synthetic fibres Ans. (d) Synthetic fibres 102. For what are the Silicon Valley near San Francisco and the Silicon Forest near Seattle examples of? (a) Metropolis (b) Technology parks (c) Technopolies (d) Electronic agglomerations Ans. (c) Technopolies 103. Which of these features of traditional industrial regions is incorrect? (a) High proportion of employment (b) High-density housing, often of inferior type, and poor services (c) Medium-density housing and fairly good services (d) Unattractive environment such as waste heaps Ans. (c) Medium-density housing and fairly good services 104. What is the chief feature of the latest generation of hightech manufacturing activities? (a) Bulk production at a low cost but controlled quality (b) Computer-aided design (CAD) and manu-facturing (c) Both (a) and (b) are correct (d) Both (a) and (b) are incorrect Ans. (c) Both (a) and (b) are correct 105. Consider the following statements. Large scale manufacturing involves:

A. A large market, various raw materials, enormous energy, specialised workers, advanced technology, assembly-line mass production and large capital. B. This kind of manufacturing developed in the last 100 years, in the United Kingdom, the eastern U.S.A. and Europe. Choose the correct option. (a) Both the statements are correct. (b) Both the statements are incorrect. (c) Statement A is correct and Statement B is wrong. (d) Statement A is wrong and Statement B is correct. Ans. (c) Statement A is correct and Statement B is wrong. 106. Consider the following statements on the system of large scale manufacturing and choose the correct option: The world’s major industrial regions may be grouped under two broad types, namely: A. Traditional large-scale industrial regions are thickly clustered in a few, but highly developed countries B. High-technology medium scale industrial regions are located in the least developed countries the world Choose the correct option. (a) Only Statement A is correct. (b) Only Statement B is correct. (c) Both the statements are correct. (d) None of the statements are correct. Ans. (a) Only Statement A is correct. 107. Consider the following statements : A. Public Sector Industries are owned and managed by

governments. B. Private Sector dominates the market in capitalist countries. Choose the correct option. (a) Only A is correct. (b) Only B is correct. (c) Both the statements are correct. (d) Both the statements are incorrect. Ans. (c) Both the statements are correct. 108. Match Column (A) Types of Industries with Column (B) Characteristics and choose the correct option with the help of given Codes. COLUMN (A)

COLUMN (B)

I.

Cottage manufacturing

1.

Simple power-driven machines and semi-skilled labour

II.

Small scale manufacturing

2.

Various raw materials, enormous energy, and specialised workers

III.

Large scale manufacturing

3.

Use local raw materials and simple tools

IV.

Cotton textile Industry

4.

It has three sub-sectors, i.e., handloom powerloom and mill sectors

Codes: I II III IV (a) 3 1 2 4 (b) 3 2 1 4

(c) 2 3 1 4 (d) 4 2 3 1 Ans. (a) I – 3, II – 1, III – 2, IV—4 109. Match Column (A) Types of Industries with Column (B) (Feature) and choose the correct option with the help of given Codes. COLUMN (A)

COLUMN (B)

I.

Public Sector Industries

1.

Managed by joint stock companies or sometimes the private and public sectors together establish and manage the industries

II.

Private Sector Industries

2.

Owned and managed by governments

III.

Joint Sector Industries

3.

Owned by individual investors

IV.

Iron and Steel Industry

4.

It forms the base for other industries

CODES : I II III IV (a) 3 2 1 4 (b) 1 3 2 4 (c) 2 3 1 4 (d) 1 4 3 2

Ans. (c) I – 2, II – 3, III – 1, IV-4 110. Match Column (A) with Column (B) and choose the correct option with the help of the given Codes : COLUMN (A)

COLUMN (B)

I.

Agro based

1.

Jewellery

II.

Mineral based

2.

Furniture

III.

Chemical based

3.

Spices

IV.

Forest based

4.

Leather

V.

Animal based

5.

Plastic

Codes: I II III IV V (a) 2 3 4 5 1 (b) 3 1 5 2 4 (c) 4 5 1 2 3 (d) 3 1 2 4 5 Ans. (b) I–3, II–1, III–5, IV–2, V–4 111. Match the countries Column (A) with Industrial towns Column (B) and choose the correct option with the help of the given Codes : COLUMN (A) I.

U.S.A

COLUMN (B) 1.

Duisburg, Dortmund, Dusseldorf and Essen

II.

U.K.

2.

Nagasaki and Tokyo-Yokohama

III.

Germany

3.

Pittsburgh, Chicago, Lorain and Sparrows Point

IV.

France

4.

Scunthorpe, Port Talbot, Birmingham and Sheffield

V.

Japan

5.

Le Creusot and St. Ettienne

Codes: I II III IV V (a) 3 4 2 1 5 (b) 4 3 2 5 1 (c) 3 4 1 5 2 (d) 3 1 4 2 5 Ans. (c) I–3, II–4, III–1, IV–5, V–2 112. Match the industrial towns Column (A) with their correct Indian State Column (B) and choose the correct option with the help of the given Codes : COLUMN (A)

COLUMN (B)

I.

Jamshedpur

1.

Andhra Pradesh

II.

Salem

2.

Karnataka

III.

Vishakhapatnam

3.

West Bengal

IV.

Bhadravati

4.

Jharkhand

V.

Durgapur

5.

Tamil Nadu

Codes: I II III IV V (a) 4 3 2 5 1 (b) 3 4 5 1 2 (c) 5 4 1 3 2 (d) 4 5 1 2 3 Ans. (d) I–4, II–5, III–1, IV–2, V–3 113. Which one of the following statements is wrong? [NCERT] (a) Cheap water transport has facilitated the jute mill industry along the Hugli. (b) Sugar, cotton textiles and vegetable oils are footloose industries. (c) The development of hydro-electricity and petroleum reduced, to a great extent, the importance of coal energy as a locational factor for industry. (d) Port towns in India have attracted industries. Ans. (b) Sugar, cotton textiles and vegetable oils are footloose industries. 114. In which one of the following types of economy are the factors of production owned individually? [NCERT] (a) Capitalist (b) Mixed (c) Socialist (d) None of these

Ans. (a) Capitalist 115. Which one of the following types of industries produces raw materials for other industries? [NCERT] (a) Cottage industries (b) Small-scale industries (c) Basic industries (d) Footloose industries Ans. (c) Basic industries 116. There are many professionals who provide their _________against payment of their fee. (a) advice (b) valuable time (c) suggestions (d) services Ans. (d) services 117. Tertiary _________.

activities

include

both

production

and

(a) exchange (b) distribution (c) sale (d) quality control Ans. (a) exchange 118. In tertiary activities, production involves the provision of services that are_________.

(a) essential (b) urgent (c) consumed (d) highly skilled Ans. (c) consumed 119. The towns and cities where buying and selling take place are known as_________. (a) markets (b) metropolis (c) trading centres (d) urban centres Ans. (c) trading centres 120. Business activity concerned with the sale of goods directly to the consumers is known as _________. (a) warehousing (b) retail trading (c) periodic selling (d) customer dealing Ans. (b) retail trading 121. Modern society requires speedy and efficient _________to assist in the production, distribution and consumption of goods. (a) highways (b) communication systems

(c) heavy vehicles (d) transport systems Ans. (d) transport systems 122. Every road that joins two nodes is called a_________. (a) route (b) link (c) node (d) network Ans. (b) link 123. Demand for transport is influenced by the_________. (a) level of industrial development (b) level of road and rail development (c) level of town planning (d) size of population Ans. (d) size of population 124. Radio and television help to relay news and pictures to vast audiences around the world, hence they are termed as_________. (a) easy media (b) world-wide (c) mass media (d) global media Ans. (c) mass media 125. When medical treatment is combined with international

tourism activity, it lends itself to what is commonly known as_________. (a) urgent tourism (b) medical tourism (c) health tourism (d) hospital tourism Ans. (b) medical tourism 126. The_________places around the Mediterranean Coast and the West Coast of India are some of the popular tourist destinations in the world. (a) continental (b) colder (c) warmer (d) hotter Ans. (c) warmer 126. Medical tourism brings abundant benefits to developing countries like_________. (a) India, Bangladesh, Myanmar (b) China, Sri Lanka and Singapore (c) Pakistan, Indonesia and India (d) India, Thailand, Singapore and Malaysia Ans. (d) India, Thailand, Singapore and Malaysia 127. Quinary activities are services which are often referred to as_________professions. (a) ‘white collar’

(b) ‘gold collar’ (c) ‘blue collar’ (d) ‘red collar’ Ans. (b) ‘gold collar’ 128. When developing countries lag behind developed countries in providing ICT access and benefits to its citizens, it is known as_________. (a) technological lag (b) developmental divide (c) progress lacuna (d) digital divide Ans. (d) digital divide 129. Arrange these statements in a sequence to indicate the development of trade from the earliest to the latest. i. Emergence of trading centres and large-scale money exchange ii. Trading centres and money-exchange on international level iii. Barter trade at the local level iv. Emergence of local markets and buying goods with money Choose the correct option. (a) (i), (iv), (iii), (ii) (b) (iv), (ii), (i), (iii) (c) (ii), (iii), (iv), (i) (d) (iii), (iv), (i), (ii) Ans. (d) (iii), (iv), (i), (ii)

130. Arrange the following activities on the basis of the skill set, professional knowledge and expertise required for them, starting from the lowest to the highest. i. Tertiary ii. Secondary iii. Primary iv. Quinary v. Quaternary Choose the correct option. (a) (iv), (iii), (i), (ii),(v) (b) (iii), (ii), (i), (v), (iv) (c) (v), (ii), (iv), (i), (iii) (d) (iii), (v), (i), (iv), (ii) Ans. (b) (iii), (ii), (i), (v), (iv) 131. Which of the following occupation does not require theoretical knowledge and skills? (a) Doctor (b) Teacher (c) Lawyer (d) Vendor Ans. (d) Vendor 132. In which sector did the large proportion of people work in the initial stages of economic development? (a) Primary sector

(b) Secondary sector (c) Intermediary sector (d) Tertiary sector Ans. (a) Primary sector 133. For exchanging goods how do the distance between places get overcome. (a) Availability of markets (b) Availability of road and rail network (c) With the help of heavy load vehicles (d) With the help of trade, transport and communication facilities Ans. (d) With the help of trade, transport and communication facilities 134. Which of the following statement is correct definition of Trade? (a) Transporting finished goods to the markets (b) Buying and selling of items produced else-where (c) Large-scale production of goods for profits (d) Import and export Ans. (b) Buying and selling of items produced else-where 135. Which of the following statements is not correct about Rural Marketing centres? (a) They are significant centres for making available goods and services which are most frequently demanded by rural folk. (b) These form local collecting and distributing centres. (c) Most of these have mandis (wholesale markets) and also

retailing areas. (d) Here personal and professional services are well-developed. Ans. (d) Here personal and professional services are welldeveloped. 136. What are ‘mandis’? (a) Local transport hub (b) Rural wholesale markets (c) Local raw material markets (d) Rural retailing areas Ans. (b) Rural wholesale markets 137. Where does most of the retail trading take place? (a) In weekly markets (b) In ‘mandis’ (c) In kiosks (d) In fixed establishments or stores Ans. (d) In fixed establishments or stores 138. Which of the following is not an example of non-store retail trading? (a) Departmental stores (b) Automatic vending machines (c) Street peddling (d) Handcarts Ans. (a) Departmental stores 139. Which of the following statements is not a feature of

chain stores? (a) Chain stores are able to purchase merchandise most economically (b) They cannot direct the goods to be manu-factured to their specification. (c) They employ highly skilled specialists in many executive tasks. (d) They have the ability to experiment in one store and apply the results to many. Ans. (b) They cannot direct the goods to be manu-factured to their specification. 140. Which of these statements is not a feature of wholesale trading? (a) Wholesale trading constitutes bulk business through numerous intermediary merchants and supply houses. (b) Wholesale trading constitutes bulk business through retail stores. (c) Wholesalers often extend credit to retail stores. (d) The credit to retail stores is often to such an extent that the retailer operates very largely on the wholesaler’s capital. Ans. (b) Wholesale trading constitutes bulk business through retail stores. 141. The lines drawn on a map to join places equal in terms of the time taken to reach them? (a) Isotherm (b) Isoline (c) Isobar

(d) Isochrone Ans. (d) Isochrone 142. A name given to the meeting point of two or more routes —a point of origin, a point of destination or any sizeable town along a route is? (a) Network (b) Node (c) Main road (d) Link Ans. (b) Node 143. What do routes depend on? (a) Location of cities, towns and villages (b) Location of industrial centres and raw materials (c) Nature of the landscape and type of climate, (d) All of the above Ans. (d) All of the above 144. What do communication services involve? (a) Words and messages (b) Routes and transport (c) None of the above (d) Both (a) and (c) Ans. (d) Both (a) and (c) 145. Which type of sector do Mumbai’s dabbawala (tiffin) service belong to?

(a) Unorganised (b) Organised (c) Semi-organised (d) None of the above Ans. (a) Unorganised 146. Which industry has become the world’s single largest tertiary activity in total registered jobs and total revenue (GDP)? (a) Textile (b) Tourism (c) Iron and steel (d) Precious and semi-precious stones Ans. (b) Tourism 147. The type of towns that attract tourists for their monuments, heritage sites and cultural activities are: (a) Religious towns (b) Industrial towns (c) Port towns (d) Historical towns Ans. (d) Historical towns 148. Which of these statements is not true about transport in relation to tourism? (a) The opening-up of tourist areas has been a result of improvement in transport facilities. (b) Travel is easier by car in areas of better road systems.

(c) There has hardly been any expansion in air transport in recent years and this has affected tourism. (d) The popularity of package holidays has reduced the holiday costs and increased tourism. Ans. (c) There has hardly been any expansion in air transport in recent years and this has affected tourism. 149. What does a CEO of an MNC in Copenhagen, at New York and a medical transcriptionist at Bengaluru have in common? (a) All these people work in primary sector related to plantation agriculture. (b) All these people work in quinary activities as policy makers. (c) All these people work in a segment of the service sector that is knowledge oriented. (d) All these people work in a segment of communication technology. Ans. (c) All these people work in a segment of the service sector that is knowledge oriented. 150. Which of the following statements is not correct about the Quaternary Sector? (a) The Quaternary Sector along with the Tertiary sector has replaced most of the primary and secondary employment as the basis for economic growth. (b) Quaternary activities cannot be outsourced. (c) They are not tied to resources, affected by the environment, or necessarily localised by market. (d) Personnel working in office buildings, elementary schools and university class-rooms, hospitals and the office of doctors theatres,

accounting and brokerage firms all belong to this category of services. Ans. (b) Quaternary activities cannot be outsourced. 151. With which of the following processes is the term ‘OffShoring’ associated? (a) Export (b) Transport (c) Outsourcing (d) Shipping Ans. (c) Outsourcing 152. What does KPO stand for? (a) Key Placement Officer (b) Key Players Organisation (c) Knowledge Process Output (d) Knowledge Processing Outsourcing Ans. (d) Knowledge Processing Outsourcing 153. Which of the following is not a feature of KPO? (a) It helps in setting up new companies. (b) Research and Development (R) and (D) activities (c) It is information driven knowledge out-sourcing. (d) Enables companies to create additional business opportunities. Ans. (a) It helps in setting up new companies 154. ‘‘Outsourcing has resulted in the opening up of a large number of call centres in certain countries.’’

Choose the correct set of countries from the options given below. (a) India, China, Western Europe, Portugal, Philippines and Costa Rica (b) India, China, Argentina, Israel, Philippines and UAE (c) India, China, Eastern Europe, Israel, Philippines and Costa Rica (d) India, China, Eastern Europe, Israel, Australia and Costa Rica Ans. (c) India, China, Eastern Europe, Israel, Philippines and Costa Rica 155. What is wholesale trading? (a) It constitutes business through retail stores. (b) It constitutes maximum business with minimum effort (c) It constitutes business through online marketing. (d) It constitutes bulk business through numerous intermediary merchants and supply houses. Ans. (d) It constitutes bulk business intermediary merchants and supply houses.

through

numerous

156. Consider the statement below. ‘Certain developments, such as mobile telephony and satellites, have made communications independent of transport.’ (a) The above statement is partially correct. (b) The above statement is totally incorrect. (c) The above statement is totally correct.

(d) The above statement is partially incorrect. Ans. (c) The above statement is totally correct. 157. Which of the following statements is not true about migrant workers? (a) They come to the cities for employment. (b) They are unskilled. (c) They are employed in domestic services. (d) This segment of workers is organised to some extent. Ans. (d) This segment of workers is organised to some extent. 158. Which of these statements is not true about Information and Communication Technology? (a) Opportunities emerging from the Information and Communication Technology based development is unevenly distributed across the globe. (b) Digital divides do not exist within countries. (c) Digital divides exist within countries. (d) Developed countries in general have surged forward in providing ICT access and benefits to its citizens. Ans. (b) Digital divides do not exist within countries. 159. Which of the following statements is correct for urban marketing centres? A. Urban marketing centres have more widely specialised urban services. B. Urban marketing centres provide ordinary goods and services as well as many of the specialised goods and services required by people.

Choose the correct option. (a) Both the statements are correct. (b) Both the statements are incorrect. (c) Statement A is correct and Statement B is incorrect. (d) Statement A is incorrect and Statement B is correct. Ans. (a) Both the statements are correct. 160. Consider the following statements. A. Most people from hot regions expect to have warm, sunny weather for beach holidays. B. People taking winter holidays have specific climatic requirements, either higher temperatures than their own homelands, or snow covered areas suitable for skiing. Choose the correct option. (a) Both the statements are correct. (b) Both the statements are incorrect. (c) Statement A is incorrect and Statement B is correct. (d) Statement A is correct and Statement B is incorrect. Ans. (c) Statement A is incorrect and Statement B is correct. 161. Consider the following statements. A. In a developed economy, the majority of workers get employment in tertiary activity and a moderate proportion is employed in the secondary sector. B. In a developed economy, the majority of workers get employment in tertiary activity and a large proportion is employed in the primary sector.

Choose the correct option. (a) Both the statements are correct. (b) Both the statements are incorrect. (c) Statement A is correct and Statement B is incorrect. (d) Statement A is incorrect and Statement B is correct. Ans. (c) Statement A is correct and Statement B is incorrect. 162. Consider the following statements. A. Rural marketing centres cater to nearby settlements. B. These centres are quasi-urban centres which serve as trading centres of the most rudimentary type. Choose the correct option. (a) Only Statement A is correct. (b) Only Statement B is correct. (c) Both the statements are incorrect. (d) Both the statements are correct. Ans. (d) Both the statements are correct. 163. Which of the following statements is/are correct about medical tourism? A. India has emerged as the leading country of medical tourism in the world. B. India offers world class medical services at reasonable price comparing with developed nations like England, USA and Canada. C. The number of patients visited India is small when compared to the millions of surgeries performed each year in the U.S. healthcare system.

Choose the correct option. (a) Only Statement A is correct. (b) Only Statement B is correct. (c) Only statements A and C are correct. (d) All the statements are correct. Ans. (d) All the statements are correct. 164. Match the Column (A) (Types of Market) with the Column (B) (features) and choose the correct option with the help of the given Codes. COLUMN (A)

COLUMN (B)

I.

Rural marketing centres

1.

Weekly markets

II.

Periodic markets

2.

Specialised markets, e.g markets for labour, housing, etc.

III.

Urban marketing centres

3.

Mandis (wholesale markets)

Codes: I II III (a) 3 1 2 (b) 3 2 1 (c) 2 3 1 (d) 1 2 3

Ans. (a) I–3, II–1, III–2 165. Match the Column (A) (Transport Distance Terms) with the Column (B) (Definitions) and choose the correct option with the help of the given codes. COLUMN (A)

COLUMN (B)

I.

Km distance

1.

Time taken to travel on a particular route

II.

Time distance

2.

Expense of travelling on a route

III.

Cost distance

3.

Actual distance of route length

Codes: I II III (a) 2 3 1 (b) 3 1 2 (c) 3 2 1 (d) 1 2 3 Ans. (b) I–3, II–1, III–2 166. Match the Column (A) (Commercial Activities) with the Column (B) (Explanation) and choose the correct option with the help of the given codes. COLUMN (A) I.

Quaternary activities

COLUMN (B) 1.

They include both production and exchange.

II.

Quinary activities

2.

They centre around research, development and may be seen as an advanced form of services involving specialised knowledge and technical skills.

III.

Tertiary activities

3.

It means giving work to an outside agency to improve efficiency and to reduce costs.

IV.

Outsourcing or contracting out

4.

They focus on the creation, rearrangement and interpretation of new and existing ideas; data interpretation and the use and evaluation of new technologies.

Codes: I II III IV (a) 3 4 2 1 (b) 4 3 2 1 (c) 3 4 1 2 (d) 2 4 1 3 Ans. (d) I–2, II–4, III–1, IV–3 167. Match the Column (A) (Colours of the Collar) with the Column (B) (Nature of Work) and choose the correct option with the help of the given codes. COLUMN (A)

COLUMN (B)

I.

Red

1.

Manual labour

II.

Gold

2.

Elderly individuals working beyond the age

of retirement as well as those occupations incorporating elements of both blue and white-collar work. III.

White

3.

Job that is traditionally considered to be women’s work.

IV.

Grey

4.

Government workers of all types.

V.

Blue

5.

Professionals (educated workers) and managerial or administrative workers.

VI.

Pink

6.

Highly paid skilled senior business executives, government officials, research scientists, financial and legal consultants, etc.

Codes: I II III IV V VI (a) 4 3 2 5 1 6 (b) 4 6 5 2 1 3 (c) 5 4 1 3 6 2 (d) 2 6 1 3 5 4 Ans. (b) I–4, II–6, III–5, IV–2, V–1, VI–3 168. Which one of the following is a tertiary activity? [NCERT] (a) Farming (b) Trading (c) Weaving (d) Hunting

Ans. (b) Trading 169. Which one of the following activities is NOT a secondary sector activity? [NCERT] (a) Iron Smelting (b) Catching Fish (c) Making Garments (d) Basket Weaving Ans. (b) Catching Fish 170. Which one of the following sectors provides most of the employment in Delhi, Mumbai, Chennai and Kolkata? [NCERT] (a) Primary (b) Secondary (c) Quaternary (d) Service Ans. (d) Service 171. Jobs that involve high degrees and level of innovations are known as: [NCERT] (a) Secondary activities (b) Quaternary activities (c) Quinary activities (d) Primary activities

Ans. (c) Quinary activities

1. Pipelines carry materials like petroleum, natural gas, and ores in___________form. (a) semi-solid (b) liquidified (c) gaseous (d) semi-liquid Ans. (b) liquidified 2. Assured and speedy transportation, along with efficient communication, promote___________. (a) cooperation and unity among scattered people (b) unity among scattered people in urban areas (c) cooperation among scattered people in rural areas (d) socialisation among scattered people Ans. (a) cooperation and unity among scattered people 3. International movement of goods by water is handled by___________. (a) defence guards (b) international traders (c) coast Guards (d) mariners Ans. (c) coast Guards 4. Long ago, in rural parts of North India, brides were carried___________.

(a) in a haudah on elephant/camel back (b) in a palanquin palki/doli by four persons called kahars (c) in a carriage drawn by horses (d) in a cart drawn by bullocks Ans. (b) in a palanquin palki/doli by four parsons called kahars 5. The revolution in transport came about only after the invention of the steam engine in the___________. (a) fifteenth century (b) sixteenth century (c) seventeenth century (d) eighteenth century Ans. (d) eighteenth century 6. During heavy rains and floods, to protect rail-tracks from being unmotorable and seriously damaged; on _______ and the efficient maintenance of railway transport service, is an effective solution. (a) high embankment of rail-tracks (b) latest technology (c) elevated floodplains (d) elevated mounds Ans. (a) high embankment of rail-tracks 7. The highest road density and the highest number of vehicles are registered in___________as compared to Western Europe. (a) South America (b) North America

(c) Africa (d) Asia Ans. (b) North America 8. Europe has a large number of vehicles and a welldeveloped highway network but these highways face a lot of competition from __________. (a) Railways and roadways (b) Railways and airways (c) Railways and waterways (d) Airways and waterways Ans. (c) Railways and waterways 9. In Russia, a dense highway network is developed in the industrialised region___________of the Urals with Moscow as the hub. (a) east (b) west (c) north (d) south Ans. (b) west 10. New Zealand’s railways are mainly in the North Island to serve the___________. (a) industrial areas (b) mining areas (c) farming areas (d) urban areas

Ans. (c) farming areas 11. In Asia, rail network is the most dense in the thickly populated areas of___________. (a) Japan, China and India (b) India, Bangladesh and Myanmar (c) India, Pakistan and China (d) India, China and Bangladesh Ans. (a) Japan, China and India 12. The Union and Pacific Railway connects New York on the Atlantic Coast to ___________on the Pacific Coast. (a) San Francisco (b) Los Angeles (c) Vancouver (d) California Ans. (a) San Francisco 13. There is a proposal to build a Trans–Asiatic Railway linking Istanbul with___________via Iran, Pakistan, India, Bangladesh and Myanmar. (a) Bangkok (b) Dhaka (c) Sri Lanka (d) Beijing Ans. (a) Bangkok 14. The Volga-Moscow Canal connects the Caspian Sea with the Moscow region and the Volga-Don Canal with

the___________. (a) Caspian Sea (b) Red Sea (c) Black Sea (d) Mediterranean Sea Ans. (c) Black Sea 15. The Rhine Waterways connects the industrial areas of Switzerland, Germany, France, Belgium and the Netherlands with the___________. (a) North Pacific Sea Route (b) South Pacific Sea Route (c) North Atlantic Sea Route (d) The Mediterranean–Indian Ocean Sea Route Ans. (c) North Atlantic Sea Route 16. In Russia, the Volga Waterway provides a navigable waterway of 11,200 km and drains into the___________. (a) Black Sea (b) Aral Sea (c) Dead Sea (d) Caspian Sea Ans. (d) Caspian Sea 17. The Mississippi-Ohio waterway connects the interior part of the U.S.A. with the___________in the south. (a) Gulf of Bothnia (b) Gulf of Mexico

(c) Bay of Biscay (d) Gulf of California Ans. (b) Gulf of Mexico 18. Sequence the following carriers of load from the earliest to the modern. i. Carts and wagons ii. Palanquin iii. Steam engine iv. Humans Choose the correct option. (a) (i), (iv), (iii), (ii) (b) (iv), (ii), (i), (iii) (c) (ii), (iii), (iv), (i) (d) (iii), (i), (iv), (ii) Ans. (b) (iv), (ii), (i), (iii) 19. Which of the follwoing statement is/are correct? A. Several place (Nodes) joined together by series of routes (links) to form a pattern. B. Road transport is cheaper and faster over short distance and for door-to-door services. (a) Only A is correct (b) Only B is correct (c) Both A and B are correct (d) Neither A nor B are correct Ans. (c) Both A and B are correct

20. What does the significance of a mode of transport depend on? (a) On the type of goods and services to be transported, costs of transport and the mode available (b) On the type of goods and services, place and connectivity (c) On the type of goods and services to be transported (d) On the costs of transport and the mode available Ans. (a) On the type of goods and services to be transported, costs of transport and the mode available 21. Which mode of transport is most suited for large volumes of bulky materials over long Inland distances within a country? (a) Roadways (b) Railways (c) Airways (d) Waterways Ans. (b) Railways 22. Which invention made the use of carts and wagons important? (a) CNG (b) Fuel (c) Battery (d) Wheels Ans. (d) Wheels 23. Which of these are the great freight carriers? (a) Container trucks and railways

(b) Trucks, boats and aircrafts (c) Railways, airways, cruise ships, pipelines (d) Railways, ocean vessels, barges, boats and motor trucks and pipelines Ans. (d) Railways, ocean vessels, barges, boats and motor trucks and pipelines 24. The Trans-Continental Stuart Highway runs between: [NCERT] (a) Darwin and Melbourne (b) Edmonton and Anchorage (c) Vancouver and St. John’s City (d) Chengdu and Lhasa Ans. (a) Darwin and Melbourne 25. Why is freight transport by road gaining importance? (a) Because it offers door-to-door service (b) Because it takes less time (c) Because it is cheaper (d) Because it is not impacted by bad weather conditions Ans. (a) Because it offers door-to-door service 26. At what time of the day do city roads suffer from chronic traffic congestion? (a) Morning at office going time (b) Evening at office closure time (c) Both (a) and (b) (d) None of the above

Ans. (c) Both (a) and (b) 27. Which one of these statements is/are incorrect about highways? (a) They are constructed in a manner for unobstructed vehicular movement. (b) These are 80 m wide, with separate traffic lanes, bridges, flyovers and dual carriageways. (c) These are built to facilitate uninterrupted traffic flow. (d) Even in the undeveloped countries, every city and port town is linked through highways. Ans. (d) Even in the undeveloped countries, every city and port town is linked through highways. 28. Which two cities are connected by NH 7, the longest National Highway in India? (a) Jaipur and Kolkata (b) Mumbai and Chennai (c) Varanasi and Kanya Kumari (d) Delhi and Bengaluru Ans. (c) Varanasi and Kanya Kumari 29. Which of these statements is/are correct about the importance of border roads? (a) Integrating people in remote areas with major cities (b) Providing defence to the country borders (c) Transporting goods to border villages and military camps. (d) All of the above Ans. (d) All of the above

30. What is the width of the standard railway gauge? (a) 1.34 metres (b) 1.40 metres (c) 1.44 metres (d) 1.50 metres Ans. (c) 1.44 metres 31. Which country has the highest density of railway network? [NCERT] (a) Brazil (b) U.S.A (c) Canada (d) Russia Ans. (b) U.S.A 32. The Big Trunk Route runs through: [NCERT] (a) The Mediterranean – Indian ocean (b) The North Atlantic Ocean (c) The South Atlantic Ocean (d) The North Pacific Ocean Ans. (b) The North Atlantic Ocean 33. Which of these options is not a feature of rail network in South America? (a) The rail network is most dense in two regions, namely, the Pampas of Argentina and the coffee growing region of Brazil.

(b) The two regions (Argentina and Brazil) account for 40 per cent of South America’s total route length. (c) Among the remaining countries, Chile has a considerable route length linking coastal centres with the mining sites in the interior. (d) Peru, Bolivia, Ecuador, Colombia and Venezuela have long single-track rail-lines from ports to the interior with several interconnecting links. Ans. (d) Peru, Bolivia, Ecuador, Colombia and Venezuela have long single-track rail-lines from ports to the interior with several inter-connecting links. 34. The Big Inch pipeline transports: [NCERT] (a) Milk (b) Liquid petroleum gas (LPG) (c) Water (d) Petroleum Ans. (d) Petroleum 35. Which of these statements is/are not correct about the Trans-Siberian Railway? (a) It is a major rail route of Russia which runs from St. Petersburg in the west to Vladivostok on the Pacific Coast. (b) It is the most important route in Asia and the longest (9,332 km) double-tracked and electrified trans–continental railway in the world. (c) It runs across the Altai Mountains. (d) Chita is an important agro-centre and Irkutsk, a fur centre. Ans. (c) It runs across the Altai Mountains.

36. Which of these statements is/are not correct about the Southern Atlantic Sea Route? (a) This is another important sea route across the Atlantic Ocean, which connects West European and West African countries, with Brazil, Argentina and Uruguay in South America. (b) The traffic is far less on this route because of the limited development and population in South America and Africa. (c) Only southeastern Brazil and Plata estuary and parts of South Africa have large-scale industries. (d) There is much greater traffic on the route between Rio de Janeiro and Cape Town because both South America and Africa have similar products and resources. Ans. (d) There is much greater traffic on the route between Rio de Janeiro and Cape Town because both South America and Africa have similar products and resources. 37. Which of these statements is/are not correct about the South Pacific Sea Route? (a) This sea route connects Western Europe and North America with Australia, New Zealand and the scattered Pacific islands via the Panama Canal. (b) This route is also used for reaching Hong Kong, Philippines and Indonesia. (c) The distance covered between Panama and Sydney is 2,000 km. (d) Honolulu is an important port on this route. Ans. (c) The distance covered between Panama and Sydney is 2,000 km. 38. Which one pair of the following places is linked by Channel Tunnel?

[NCERT] (a) London – Berlin (b) Paris – London (c) Berlin – Paris (d) Barcelona – Berlin Ans. (b) Paris – London 39. Which city among the following Headquarters of EU? (a) London (b) Paris (c) Brussels (d) Berlin Ans. (c) Brussels 40. Arrange the following religion Blocks in a sequential order according to their develop-ment. Choose the correct option. 1. European Union 1943 2. Association of South East Asian Nations 1943 3. Latin American Integration Association 1994 4. South Asian Free Trade Agreement Options: (a) 2 1 3 4 (b) 4 1 3 2 (c) 2 3 1 4 (d) 3 2 4 1 Ans. (a) 2 1 3 4

40. Aden, Honolulu and Singapore are good examples of which type of the following ports ? (a) Packet Station (b) Ports of Call (c) Out Ports (d) Inland Ports Ans. (b) Ports of Call 41. Which of the following ports have strategic importance and serve warships and have repair workshops for them? (a) Packet Station (b) Oil Ports (c) Entrepot Ports (d) Naval Ports Ans. (d) Naval Ports 42. Basis of International trade can be based on: (i) Difference in national resource (ii) Population factors (iii) Stage of economic development (iv) Extent of foreign investment (v) Transport Choose the correct option. (a) (i), (ii), (iii) only (b) (i), (ii), (iv) only (c) (i), (ii), (v), Only (d) (i), (ii), (iii), (iv), (v)

Ans. (d) (i), (ii), (iii), (iv), (v) 43. Which of the following column is not matched correctly? COUNTRY YEAR OF ABOLISHING Slave Trade (a) USA 1809 (b) Denmark 1792 (c) Great Britain 1807 (d) United States 1809 Ans. (a) United States 1809 44. Which one of the following is not included in SAARC? (a) India (b) Pakistan (c) Nepal (d) Iran Ans. (d) Iran 45. Which year among the following GATT was formed? (a) 1924 (b) 1914 (c) 1954 (d) 1948 Ans. (d) 1948 46. The word salary comes from Salarium which means payment by salt. This is taken from which language? (a) Latin (b) German

(c) French (d) English Ans. (a) Latin 47. How many countries are there in OPEC? (a) 15 (b) 14 (c) 13 (d) 10 Ans. (c) 13 48. Which one of the following organisations helped in reducing tariff ? (a) NAFT (b) LAIA (c) EU (d) WTO Ans. (d) WTO 49. Which of the following country is not a member of South Asian Free Trade Agreement ? (a) Bangladesh (b) Maldives (c) Bhutan (d) Algeria Ans. (d) Algeria 50. Which of the following pairs is matched correctly? Column I

Column II

(Ports)

(Types of Cargo Handled)

(a) Industrial Ports

1.

Parent Port

(b) Commercial Ports

2.

Grain, sugar, ore, oil,

(c) Comprehensive Ports

3.

Bulk and general cargo in large volumes

(d) Out Port

4.

Cargo-packaged products and manufactured good.

Ans. (c) Comprehensive 3. Bulk and general cargo Ports in large volumes 51. Match the column I with Column II and choose the correct options with the help of given Codes. Column I (Ports)

Column II (Types of Cargo Handled)

I.

Oil Ports

1.

Warships and repair workshops

II.

Naval Ports

2.

Processing and shipping of oil

III.

Entrepot Ports

3.

Ferry ports

IV.

Packet Station

4.

Collection centres

V.

Ports of Call

5.

Located away from the sea coast

VI.

Inland Ports

6.

Commercial ports

Codes: I II III IV V VI

(a) 1 2 3 4 5 6 (b) 4 2 1 3 6 5 (c) 6 5 4 3 2 1 (d) 2 1 4 3 6 5 Ans. (d) I – 2, II – 1, III – 4, IV – 3, V – 6, VI – 5 52. Which one of the following is the main feature of Regional Trade Blocs? (a) It is detrimental to nations of it leads to dependence on other countries, uneven levels of development, exploitation and commercial rivalry leading to wars. (b) It affects many aspects of life from the environment to health and well-being of the people around the world. (c) As countries compete to trade more, production and the use of natural resources spiral up, resources get used up faster than they can be replenished. (d) It has come up in order to encourage trade between countries with geographical proximity, similarity and complementarities in trading items and to curb restrictions on trade of the developing world. Ans. (d) It has come up in order to encourage trade between countries with geographical proximity, similarity and complementarities in trading items and to curb restrictions on trade of the developing world. 53. Which of these trade blocs has India as an associate member? (a) ASEAN (b) SAFTA (c) OPEC

(d) OECD Ans. (b) SAFTA 54. Which of the following is the correct meaning of Latin word ‘Salarium’ ? (a) Non-payment of salary (b) Payment by salt (c) Trade in Salt by Sea (d) Barter of Salt for Silver Ans. (b) Payment by salt 55. In which year among the following General Agreement for Tariffs and Trade was trans-formed into the World Trade Organisation ? (a) 1st January 1990 (b) 1st January 1985 (c) 1st January 1995 (d) 2nd January 1995 Ans. (c) 1st January 1995 56. Which of the following attributes are not related to Balance of Trade? (a) It is the volume of goods and services imported as well as exported by a country to other countries. (b) If the value of imports is more than the value of a country’s exports, the country has negative or unfavourable balance of trade. (c) If the value of exports is more than the value of imports, then the country has a positive or favourable balance of trade. (d) All of the above

Ans. (d) All of the above 57. Which among the following was the initial form of trade in primitive societies ? (a) Barter system, (b) Use of flintstones, obsidian, cowrie shells, tiger’s paws and whale’s teeth. (c) Use of skins, furs, cattle, rice, peppercorns, salt and small tools. (d) Use of copper, silver and gold. Ans. (a) Barter system, 58. Which one of the following is a Inland port? (a) Mumbai (b) Kochi (c) Kolkata (d) Karachi Ans. (c) Kolkata 59. Which of the following statement is/are correct? (i) Most of the world great part are classified as comprehensive port. (ii) Maracaibo in Venezuela is a oil ports. Option (a) Only (i) (b) Only (ii) (c) Both (i) and (ii) (d) None of the above Ans. (c) Both (i) and (ii)

60. Ports City (i) Esskhira 1. Lebanon (ii) Tripoli 2. Tunisia (iii) Dover 3. France (iv) Ealais 4. England (a) (i)–1, (ii)–2, (iii)–3, (iv)–4 (b) (i)–4, (ii)–3, (iii)–2, (iv)–1 (c) (i)–2, (ii)–1, (iii)–4, (iv)–3 (d) (i)–2, (ii)–3, (iii)–4, (iv)–1 Ans. (c) (i)–2, (ii)–1, (iii)–4, (iv)–3 61. Which of the following is a good example of Out Port? (a) Manchester (b) Rhine (c) Kolkata (d) Piraeus Ans. (d) Piraeus 62. These ports are collection centres where the goods are brought from different countries for export: (a) Entrepot Ports (b) Packet Station (c) Ferry ports (d) Ports of Call Ans. (a) Entrepot Ports 63. The chief gateways of the world of international trade are the ______ and _____.

(a) harbours, ports (b) trade, commerce (c) harbour, land route (d) land route, airways Ans. (a) harbours, ports 64. Choose the countries connected by Silk route: (a) China and Rome (b) China and India (c) Japan and Sri Lanka (d) Africa and China Ans. (a) China and Rome 65. Which of the following is characteristic of a Port’s hinterland?

the

most

important

(a) The ports provide facilities of docking, loading, unloading and the storage facilities for cargo. (b) The port authorities make arrangements for maintaining navigable channels, arranging tugs and barges and providing labour and managerial services. (c) The importance of a port is judged by the size of cargo and the number of ships handled. (d) The quantity of cargo handled by a port is an indicator of the level of development of its hinterland. Ans. (d) The quantity of cargo handled by a port is an indicator of the level of development of its hinterland. 66. Which one of the following South American nation is a part of OPEC? (a) Brazil

(b) Chile (c) Venezuela (d) Peru Ans. (c) Venezuela 67. Which of the following transported on the Silk Route?

commodities

the

traders

(a) Chinese silk, Gold and precious metals (b) Chinese silk, Roman wool and Gold (c) Gold, Roman wool and precious metals (d) Chinese silk, Roman wool and precious metals Ans. (d) Chinese silk, Roman wool and precious metals 68. Why size of population is considered as Basis of International Trade Choose the correct answer (a) They have large volume of internal trade but little external trade (b) They have rich mineral resource base (c) They have different types of flora and fauna (d) They have more Foreign investment Ans. (a) They have large volume of internal trade but little external trade 69. International trade has three very important aspects. These are : (a) Transportation, sectoral composition and direction of trade. (b) Volume, transportation and direction of trade. (c) Volume, sectoral composition and trans-portation. (d) Volume, sectoral composition and direction of trade.

Ans. (d) Volume, sectoral composition and direction of trade. 70. A ________ balance would mean that the country spends more on buying goods than it can earn by selling its goods. Choose the correct option. (a) Negative (b) Positive (c) Favourable (d) None of the above Ans. (a) Negative 71. Which one of the following is the areas of cooperation of European Union ? (a) Accelerate economic growth, cultural development, peace and regional stability (b) Single market with single currency (c) Coordinate and unify petroleum policies. (d) Reduce tariffs on interregional trade Ans. (b) Single market with single currency

1. Which one of the following settlement pattern is found along road, railway line, river etc.? (a) Rectangular (b) Linear (c) Triangular (d) Star-like Ans. (b) Linear 2. Arrange the following urban centers in a sequential order according to their size and the services available and functions rendered. Choose the correct option. 1. City 2. Million City 3. Conurbation 4. Megalopolis Options: (a) 1 4 2 3 (b) 4 1 3 2 (c) 1 2 3 4

(d) 3 2 4 1 Ans. (c) 1 2 3 4 3. Which of the following is a new trend of people moving away from congested urban areas to cleaner areas outside the city in search of a better quality of living? (a) Urban Settlement (b) Urbanisation (c) Sub Urbanisation (d) Rural Settlement Ans. (c) Sub Urbanisation 4. Which of the diseases is most common in villages ? (a) Water borne diseases (b) HIV/AIDS (c) Diabetes (d) Depression Ans. (a) Water borne diseases 5. Which of the following is not a pattern of human settlement? (a) Linear (b) Circular (c) Star-shaped

(d) Hamleted Ans. (d) Hamleted 6. Which one of the following types of economic activities dominates in all rural settlements? (a) Primary (b) Tertiary (c) Secondary (d) Quaternary Ans. (a) Primary 7. Which of the following best describes ‘urbanisation’? (a) Increase in the proportion population of a country who live in urban areas. (b) Its cause is rural-urban migration. (c) During the late 1990s some 20 to 30 million people were leaving the countryside every year and moving into towns and cities. (d) Developed countries experienced rapid urbanisation during the nineteenth century. Ans. (a) Increase in the proportion population of a country who live in urban areas. 8. Which of the following regions has the oldest urban settlement? (a) Indus Valley

(b) Huang-Ho valley (c) Nile Valley (d) Mesopotamia Ans. (d) Mesopotamia 9. How many of the following cities in India have attained the million status in 2011? (a) 44 (b) 48 (c) 53 (d) 56 Ans. (c) 53 10. Which of the following is the functional classification of an urban settlement? (a) Industrial (b) Mining Town (c) Administrative (d) All of these Ans. (d) All of these 11. According to year 2017, which of the following depicts the percentage of World’s Population Living in Urban Areas? (a) 54%

(b) 37% (c) 48% (d) 30% Ans. (a) 54 % 12. Which of the following town act as administrative city? (a) London (b) Chandigarh (c) Canberra (d) All of these Ans. (d) All of these 13. Which were the first million cities in the world? (a) Mumbai (b) London (c) Singapore (d) Tokyo Ans. (b) London 14. Which of the following settlements extend on both sides of a river where there is a bridge or a ferry? (a) Rectangular pattern (b) Circular pattern

(c) Star like pattern (d) Double village Ans. (d) Double village 15. Which of the following pair is not correctly matched: City Type of Urban Center (a) Washington D.C. Administrative Town (b) Manchester Trading and commercial Center (c) Pittsburgh Industrial (d) Singapore Cultural Ans. (d) Singapore Cultural 16. Which of the following town was planned in 1912 by American landscape architect, Walter Burley Griffin? (a) Pittsburgh (b) Canberra (c) Addis Ababa (d) Broken Hill Ans. (b) Canberra 17. Which of the following towns is an Agricultural market town? (a) Winnipeg (b) Frankfurt (c) Amsterdam (d) Manchester

Ans. (a) Winnipeg 18. Which of the following is a good example of health and recreation city ? (a) Miami (b) Mughal Sarai (c) Jerusalem (d) Mecca Ans. (a) Miami 19. Who coined the word CONURBATION? (a) Patrick Geddes (b) Amartyasec (c) Jene Gottman (d) Ratzel Ans. (a) Patrick Geddes 20. Which of the following place is planned settlement? (a) Villagisation in Ethiopia (b) Canal colonies in Indira Gandhi canal (c) Both (a) and (b) (d) Every town in India Ans. (c) Both (a) and (b)

21. Which of the following area people in south east Asia chose to live near suited for wet rice cultivation? (a) Low lying river valleys and coastal plains (b) Low lying river valleys and plains (c) High lying river valleys and coastal plains (d) High lying river valleys and plains Ans. (a) Low lying river valleys and coastal plains 22. Which of the following place, where tropical countries people build their houses to protect themselves from flood, insects and animal pests? (a) Upland (b) Stilts near marshy lands (c) Low lying river basins (d) Terraces and levees Ans. (b) Stilts near marshy lands 23. Choose the pattern when village is planned in such a way that the central part remains open and is used for keeping the animals to protect them from wild animals. (a) Circular pattern (b) Star like pattern (c) Cross-shaped (d) Cruciform settlements Ans. (a) Circular pattern

24. Which one of the following is not an Administrative Towns? (a) Canberra (b) Beijing (c) Addis Ababa (d) Winnipeg Ans. (d) Winnipeg 25. When was Addis Ababa was established? (a) 1870 (b) 1878 (c) 1875 (d) 1900 Ans. (b) 1878 26. In the Asia Pacific countries, what per cent of the urban population lives in squatter settlements. Select the correct option. (a) 40% (b) 50% (c) 60% (d) 70% Ans. (c) 60% 27. A ‘healthy city’s characteristics like- A ’Clean’ and ‘Safe’ environment, meeting the ‘Basic Needs’ of ‘All’ its inhabitants

and involving the ‘Community’ in local government. Above mentioned important characteristic of city is given by which of the following organisation? (a) UNDP (b) WHO (c) UNICEF (d) NDMC Ans. (b) WHO 28. Which of the following problem associated with urban settlement? (a) Economic Problems (b) Socio-cultural Problems (c) Environmental Problems (d) All of the above Ans. (d) All of the above 29. The term ``megalopolis'' designates the largest type of urban conurbation. The concept was first proposed by: (a) Patrick Geddes (b) Amartya Sen (c) Jean Gottman (d) Ratzel

Ans. (c) Jean Gottman 30. Which of the following is the population of a megacity? (a) More than 10 million (b) More than 30 million (c) More than 20 million (d) More than 5 million Ans. (a) More than 10 million 31. Which among the following shaped settlements emerge as the places where two roads converge on the third one and houses are built along these roads? (a) T-shaped (b) Y-shaped (c) Cross-shaped (d) Cruciform settlements Ans. (b) Y-shaped

1. India’s population as per 2011 census: (a) 3182 Million (b) 1210 Million (c) 1120 Million (d) 3000 Million Ans. (b) 1210 Million 2. Which of the Indian state has the highest rural population? (a) Goa (b) Punjab (c) Bihar (d) Tamil Nadu Ans. (c) Bihar 3. Which of the following is the most significant religious minority in India? (a) Sikh (b) Buddhist (c) Muslim (d) Christian Ans. (c) Muslim 4. Complete the following flow chart by filling the blank column:

(a) Birth rates (b) Crude birth rates (c) Death rates (d) None of the above Ans. (b) Crude birth rates 5. When was the first significant modification introduced in the census by bringing in two additional components – place of birth and duration of residence? (a) 1921 (b) 1941 (c) 1961 (d) 1981 Ans. (c) 1961 6. What is India’s life expectancy according to 2011 census for male? (a) 55 Years (b) 60 years (c) 66 years (d) 64.6 years Ans. (d) 64.6 years

7. Which of these language families is predominant in India? (a) Sino-Tibetan (b) Dravidian (c) Austric (d) Indo-European Ans. (d) Indo-European (Map Based Questions) Study the map carefully and answer the questions that follow.

8. Which state of India has the highest density of population? (a) Bihar (b) HP (c) UP (d) MP Ans. (a) Bihar 9. Which state of India has the lowest density of population? (a) Bihar (b) Arunachal Pradesh (c) Madhya Pradesh

(d) Haryana Ans. (b) Arunachal Pradesh 10. Which state of India has the highest % of the rural population? (a) Uttar Pradesh (b) Madhya Pradesh (c) Haryana (d) Jammu and Kashmir Ans. (a) Uttar Pradesh 11. Which one of the following states has the lowest literacy rate? (a) Bihar (b) MP (c) Gujarat (d) Kerala Ans. (a) Bihar 12. Which of the following states had out-migration? (a) Uttar Pradesh (b) Mumbai (c) Both (a) and (b) (d) West Bengal Ans. (a) Uttar Pradesh 13. Which one of the factors given below is responsible for the migration of people in India? I. High population pressure on land

II. Availability of regular work III. For higher wages (a) I and III are responsible factors (b) Only I factor (c) Only II and III are responsible (d) Only III factor Ans. (c) Only II and III are responsible 14. Which of the statements given below is/are true? I. Due to poverty, unskilled migrants move from rural to urban. II. Due to the absence of family members, unskilled migrants move from rural to urban. III. Due to humiliation, unskilled migrants move from rural to urban. (a) Only I statement is true (b) Both II and III are true (c) Only II is true (d) All three are true Ans. (a) Only I statement is true 15. Who wrote these lines? “Sar Zamin-E-Hind Par Aqwam-e-Alam ke Firaque Kafile Baste Gaye, Hindostan Banta Gya” (a) Faiz (b) Gorakhpuri (c) Josh (d) Akbar Ans. (b) Gorakhpuri

16. Consider the following statements. The work participation rate is high among the scheduled tribes because: I. They live in harsh conditions such as inaccessible, hilly, and forested regions, where they work whole day to earn their bread. II. The tribal community is not an open society where males, females, and children enjoy equal status and participation in work. Choose the correct option: (a) Both the statements are correct (b) Only I is true. (c) Only II is true. (d) Both statements are wrong. Ans. (b) Only I is true. 17. Match the following consequences of international migration in India: Column A I

Positive consequences

II

Negative

Column B (a) Intermixing of people from diverse cultures (b) Cross-National

consequences III

Social consequences

Options : (a) I-b, II-c, III-a (b) I-a, II-c, III-b

networks (c)

Loss of significant number of highly skilled labour

(c) I-c, II-a, III-b (d) I-b, II-a, III-c Ans. (a) I-b, II-c, III-a 18. Consider the following statements and choose the correct answer from the given options. 1. In India, the work participation rate tends to be higher in the areas of lower levels of economic development. 2. The number of manual workers are needed to perform the subsistence or near subsistence economic activities. Options: (a) Only 1 is correct (b) Only 2 is correct (c) Both the statements are incorrect (d) Both the statements are correct and statement 2 correctly explains the statement 1. Ans. (d) Both the statements are correct and statement 2 correctly explains the statement 1. 19. Which of the group given below is correct? I. Main workers – works at least 183 days in a year II. Marginal workers – who work less than 100 days in a year III. Non-worker – who only works for his livelihood (a) Only I is true. (b) Only II is true. (c) Both II and III are true. (d) All three are true. Ans. (a) Only I is true.

20. How many Scheduled languages are there in our Constitution? (a) 20 (b) 21 (c) 22 (d) 23 Ans. (c) 22 21. Match the following: Column A

Column B

I.

Low degree of urbanization

II.

Less population in comparison (b) Mizoram to plain area

III.

Union territory having the lowest density

(a)

(c)

Andaman and Nicobar Island

Remote and hilly areas

(a) I-(a), II-(b), III-(c) (b) I-(c), II-(a), III-(b) (c) I-(a), II-(c), III-(b) (d) I-(b), II-(c), III-(a) Ans. (d) I-(b), II-(c), III-(a) 22. How many phases of population growth are there? (a) 1 (b) 2 (c) 3 (d) 4

Ans. (d) 4 23. The average density of population of India in 2011 is_______ persons per km. (a) 216 (b) 382 (c) 300 (d) 314 Ans. (b) 382 24. What is the average sex ratio in India according to census 2011 ? (a) 940 (b) 840 (c) 490 (d) 740 Ans. (a) 940 25. According to the 2011 census, which of the following districts has the highest population density? (a) North East Delhi (b) Spiti (c) Lahaul (d) All of the above Ans. (a) North East Delhi 26. Which of the group given below is true? I. Urban population – part of the population lives in towns and cities II. Urban population – engaged in tertiary and secondary

occupations III. Rural population – advanced infrastructure facilities IV. Urban population – lower population growth (a) Both III and IV are true. (b) Both I and II are true. (c) Only I is true. (d) Only III is true. Ans. (b) Both I and II are true. 27. The annual growth rate of India’s population is: (a) 17.7% (b) 2% (c) 2.5% (d) 1.64% Ans. (d) 1.64% (Map Based Questions) Study the map carefully and answer the questions that follow.

28. Choose the Union Territory with lowest density of population in India.

(a) NCT of Delhi (b) Lakshadweep (c) Andaman and Nicobar Islands (d) Puducherry Ans. (c) Andaman and Nicobar Islands 29. Which of the following Indian states has the low rate of population growth during 1991-2001? (a) Karnataka (b) Gujarat (c) Maharashtra (d) Madhya Pradesh Ans. (a) Karnataka 30. Identify the state which attracts most migrants from other states. (a) Uttar Pradesh (b) Rajasthan (c) Gujarat (d) Maharashtra Ans. (d) Maharashtra 31. Choose the state where high out-migration has brought serious imbalances in age and sex composition. (a) Bihar (b) Uttarakhand (c) Maharashtra (d) Tamil Nadu

Ans. (b) Uttarakhand 32. Which of the following states has very high percentage of rural population? (a) Sikkim (b) Himachal Pradesh (c) Assam (d) Uttarakhand Ans. (a) Sikkim 33. Which one of the following Union Territories has the highest density of population? (a) NCT Delhi (b) Andaman and Nicobar Island (c) Chandigarh (d) Puducherry Ans. (a) NCT Delhi 34. Arrange the following Religious Communities of India, 2011 in a sequential order according to their population % of total. Choose the correct option. 1. Hindu 2. Muslims 3. Christians 4. Sikhs Options: (a) 1 2 3 4 (b) 4 1 3 2

(c) 2 3 1 4 (d) 3 2 4 1 Ans. (a) 1 2 3 4 35. Which of the language families is predominant in India? (a) Sino Tibetan (b) Dravidian (c) Indo-European (d) Austric Ans. (c) Indo-European 36. Which of the following is a category of workers who works for atleast 183 days ( or six months) in a year? (a) Cultivators (b) Agricultural labourers (c) Marginal worker (d) Main worker Ans. (d) Main worker 37. According to 2011 census, which one of the following states has the highest population density? (a) Goa (b) Kerala (c) Maharashtra (d) Tamil Nadu Ans. (b) Kerala 38. In which phase, India improved quality of life, increase in the mean age of marriage and increase in education of female.

(a) Phase I (b) Phase II (c) Phase III (d) Phase IV Ans. (d) Phase IV 39. Which of the following states registered the lowest growth rate of 9.4%? (a) Kerala (b) Karnataka (c) Tamil Nadu (d) Andhra Pradesh Ans. (a) Kerala 40. Which of the following state have very high percentage of rural population? (a) Bihar and Sikkim (b) Goa and Maharashtra (c) Karnataka and Kerala (d) Tamil Nadu and Andhra Ans. (a) Bihar and Sikkim 41. According National Youth Policy-2014, youth as a person in the age group of _________. (a) 14-28 years (b) 15-29 years (c) 15-28 years (d) 15-30 years

Ans. (b) 15-29 years 42. Which one of the following language is not a part of Dravidian family of Modern Indian Languages? (a) North Dravidian (b) South Dravidian (c) Central Dravidian (d) Western Dravidian Ans. (d) Western Dravidian 43. According to census 2011, which of the following percent the proportion of workers (both main and marginal) is correct? (a) 39.8 % (b) 28 % (c) 33.16 % (d) 26.45 % Ans. (a) 39.8 % 44. In India, which sector has highest portion of worker engaged? (a) Primary sector (b) Secondary sector (c) Tertiary sector (d) Quaternary sector Ans. (a) Primary sector 45. The highly urbanised areas like Delhi, Chandigarh and Puducherry have a very large proportion of workers being engaged in other services. Select the correct answer.

(a) This indicates availability of limited farming land. (b) Large scale urbanisation and indus-trialisation requiring more workers in non-farm sectors. (c) Both (a) and (b) (d) This indicates availability of unlimited farming land. Ans. (c) Both (a) and (b) 46. Which of the following statement is true? (a) The first population census in India was conducted in 1872. (b) First complete census was conducted in 1881. (c) Both (a) and (b) (d) The first population in India was conducted in 1891. Ans. (c) Both (a) and (b) 47. The National Youth Policy of Government of India launched in the year _________ . (a) 2000 (b) 2001 (c) 2002 (d) 2014 Ans. (d) 2014 48. Which of the following state has seen the highest fall in the growth rate of population from 2001-2011 compared to 1991-2001? (a) Kerala (b) Tamil Nadu (c) Andhra Pradesh (d) Maharashtra

Ans. (d) Maharashtra 49. Which of the following policy Government of India has undertaken to impart proper education to the adolescent groups so that their talents are better channelised and properly utilised? (a) Rashtriya Kishor Swasthya Karyakram (b) The National Youth Policy (c) The Scheme for Adolescent Girls (d) National Programme for Youth Adolescent Development Scheme Ans. (b) The National Youth Policy 50. Which of the following is a distinct field of study within population geography with a vast coverage of analysis of age and sex, place of residence, ethnic characteristics, tribes, language, religion, marital status, literacy and education, occupational characteristics? (a) Population Composition (b) Rural – Urban Composition (c) Linguistic Composition (d) Religious Composition Ans. (a) Population Composition 51. Which one of the following is the sector where number of female workers is relatively high? (a) Primary (b) Secondary (c) Tertiary (d) Quaternary

Ans. (a) Primary 52. Match the column I with Column II and choose the correct options with the help of given Codes. Column I (Family)

Column II (Speech Area)

I.

Mon-Khmer

1.

Meghalaya, Nicobar Islands

II.

Dravidian

2.

Tamil Nadu, Karnataka, Kerala

III.

Sino-Tibetan

3.

Jammu and Kashmir, Himachal Pradesh,Sikkim

IV.

Indo – Aryan

4.

Uttar Pradesh, Rajasthan, Gujarat

Codes: I II III IV (a) 1 2 3 4 (b) 4 2 1 3 (c) 1 2 4 3 (d) 2 1 4 3 Ans. (a) I – 1, II – 2, III – 3, IV – 4 53. Consider the following features and choose the correct title after associating them. 1. The decades 1921-1951 are referred to as the period of steady population growth. 2. An overall improvement in health and sanitation throughout the country brought down the mortality rate. 3. At the same time better transport and communication

system improved distribution system. Options: (a) Phase I (b) Phase II (c) Phase III (d) Phase IV Ans. (b) Phase II 54. What is life expectancy in India? (a) 55 years (b) 60 years (c) 66 years (d) 70 years Ans. (c) 66 years 55. Density of population is expressed as number of persons ______________ . Choose the correct option. (a) Per unit area (b) Per unit state (c) Per unit agriculture (d) Per unit production Ans. (a) Per unit area 56. Which of the following is a crude measure of human and land relationship? (a) The density of population (b) The density of minerals present in that area.

(c) The density of water in an particular area. (d) The density of flora and fauna. Ans. (a) The density of population 57. Which of the phase referred to as the period of population explosion in India. (a) Phase I (b) Phase II (c) Phase III (d) Phase IV Ans. (c) Phase III 58. Which of the following pair is not correctly matched: INDICATORS

DEFINITION

(a) Physiological density

Total population /net cultivated area

(b) Agricultural density

Total agricultural population / net cultivable area

(c) Agricultural population

Cultivators and agricultural labourers and their family members.

(d) Growth of population

Time taken by any population to double itself

Ans. (d) Growth of population

Time taken by any population to double itself

59. The growth rate of population projected by world development report that population of India will touch 1350 million in year.

(a) 2025 (b) 2030 (c) 2035 (d) 2040 Ans. (a) 2025 60. Which of the following statement is not true regarding the ‘Composition of Working Population’? (a) The occupational composition of India’s population shows a large proportion of primary sector workers compared to secondary and tertiary sectors. (b) The number of female workers is relatively high in primary sector, though in recent years there has been some improvement in work participation of women in secondary and tertiary sectors. (c) It is important to note that the proportion of workers in agricultural sector in India has shown a rise over the last few decades. (d) The highly urbanised areas like Delhi, Chandigarh and Puducherry have a very large proportion of workers being engaged in other services. Ans. (c) It is important to note that the proportion of workers in agricultural sector in India has shown a rise over the last few decades. 61. Which of the following is a source for Decadal Growth Rates in India? (a) Social Geography (b) Occupational Structure (c) Census of India (d) Sectoral Composition of work force

Ans. (c) Census of India 62. Which of the following statement is/are correct? (i) From 1981 till present, the growth rate of country’s population has been slowing down gradually. (ii) A downward trend of crude birth rate is held responsible for such a population growth. Options: (a) Only (i) (b) Only (ii) (c) Both (i) and (ii) (d) Neither (i) nor (ii) Ans. (c) Both (i) and (ii) 63. Which of the following factor is not a Push Factor? (a) Poverty (b) High population (c) Pressure on the land (d) Entertainment Ans. (d) Entertainment 64. Which of the following country is the source of maximum remittance for India? (a) USA (b) UAE (c) Saudi Arabia (d) Russia Ans. (a) USA

65. Which one of the following is the main reason for male migration in India? (a) Marriage (b) Work and employment (c) Business education (d) Education Ans. (b) Work and employment 66. Which one of the following states receives a maximum number of immigrants in India? (a) Madhya Pradesh (b) Maharashtra (c) Uttar Pradesh (d) Tamil Nadu Ans. (b) Maharashtra 67. Name the stream that male migrants in India dominate. (a) Rural-Urban (b) Urban-Rural (c) Urban-Urban (d) Rural-Rural Ans. (a) Rural-Urban 68. Which one of the following Urban Agglomeration (UA) has the highest share in-migrant? (a) Mumbai UA (b) Delhi UA (c) Bangalore UA

(d) Chennai UA Ans. (a) Mumbai UA 69. When migration was recorded in the census in India for the first time? (a) 1881 (b) 1891 (c) 1981 (d) 1991 Ans. (a) 1881 70. Which one of the following is the economic benefit of migration? (a) Remittances (b) Medical facilities (c) Basic facilities (d) None of the above Ans. (a) Remittances 71. Which one is the social consequence of migration in India? (a) Change in sex ratio (b) Remittances (c) Intermixing of people from diverse culture (d) Women empowerment Ans. (c) Intermixing of people from diverse culture 72. Which one of the following is not an environmental consequence of migration? (a) Over-exploitation of natural resources

(b) Depletion of groundwater (c) Pollution (d) Change in sex ratio Ans. (d) Change in sex ratio 73. Which one of the following Indian states has a maximum number of net out-migrants? (a) Uttar Pradesh (b) Madhya Pradesh (c) Bihar (d) Chhattisgarh Ans. (a) Uttar Pradesh 74. Which of the following is a factor is one of the important factors contributing to the population growth of cities? (a) Urban Urban migration (b) Rural Rural migration (c) Urban Rural migration (d) Rural Urban migration Ans. (d) Rural Urban migration 75. Which of the following stream the female migration is highest? (a) Urban to Urban migration (b) Rural to Rural migration (c) Urban to Rural migration (d) Rural to Urban migration Ans. (b) Rural to Rural migration

76. Which among the following was the state which had the largest number of net out-migrants from the state? (a) Kerala (b) Himachal Pradesh (c) Bihar (d) Chhattisgarh Ans. (c) Bihar 77. Which census introduced two major modification (1-place of birth. 2-place of residence)? (a) 1951 (b) 1961 (c) 1971 (d) 1981 Ans. (b) 1961 78. Which of the following is the demographic consequences of migration? (a) High Infrastructure facilities (b) Lesser jobs (c) Uneven distribution of opportunities over space (d) Pressure on existing infra-structure Ans. (c) Uneven distribution of opportunities over space 79. In which sectors do the immigrants mainly work? (a) Agriculture (b) Industry (c) Service

(d) Others Ans. (b) Industry 80. Which of the Indian states receives the highest amount of remittances from migrates? (a) Kerala (b) Punjab (c) Haryana (d) UP Ans. (a) Kerala 81. What work was done by Indian Migrants in West Indies? (a) Mining (b) Industries (c) Plantation farming (d) Horticulture Ans. (c) Plantation farming 82. Which is the major reason for female migration by last residence India, as per 2011 census of India? (a) Business (b) Education (c) Work employment (d) Marriage Ans. (d) Marriage 83. Which one of the following is not a push factor for migration? (a) Drought

(b) Cyclonic storm (c) Better education facilities (d) Earthquake Ans. (c) Better education facilities 84. Which of the following statement is/are correct? (i) Migration evolve composite culture and widens up the mental horizon of the people at large. (ii) Migration has no social negative impact on society. (a) Only (i) (b) Only (ii) (c) Both (i) and (ii) (d) Nither (i) nor (ii) Ans. (a) Only (i) 85. Which is not a push factor? (a) Poverty (b) Pressure of population (c) Entertainment (d) Disaster Ans. (c) Entertainment 86. Which of the statements given below is/are true? I. Marriage is the main reason for male migration in India. II. Work and employment is the main reason for male migration in India. III. Business is the main reason for male migration in India. IV. Education is the main reason for male migration in India.

(a) Only I is true. (b) Only II is true. (c) Both I and II are true. (d) Both III and IV are true. Ans. (b) Only II is true. 87. Match the following. Column I

Column II

I.

Lifetime migrant

(a)

Uttarakhand

II.

Rural-urban

(b)

Place of birth is different from enumeration

(c)

Population growth of cities

migration III. High outmigration (a) I-c, II-a, III-b (b) I-b, II-c, III-a (c) I-a, II-c, III-b (d) I-b, II-a, III-c Ans. (b) I-b, II-c, III-a

88. Greater Mumbai has the highest share of the migrant population. It is due to which of the following reasons: I. More employment opportunities II. Relatively lower wages III. Bad educational facilities IV. Urbanisation

(a) Both I and IV reasons are true. (b) Only II is true. (c) Only I is true. (d) All four are true. Ans. (a) Both I and IV reasons are true. 89. Which of the economic consequences of migration in India is correct? I. Remittance from international migrants. II. Internal migrants play an essential role in the growth of the economy. III. Remittances sent by migrants. (a) Only I is correct. (b) Only II is correct. (c) Both II and III are correct. (d) All three reasons are correct. Ans. (d) All three reasons are correct. 90. Which of the following statement is true? (a) The amount of remittance sent by the internal migrant is very meagre as compased to international migrant. (b) India, the world largest recipient of remittances. (c) Both (a) and (b) (d) India ranked third in the recipient of remittance. Ans. (c) Both (a) and (b) 91. Which of the following factor is pull factor? (a) Better opportunity

(b) Availability of regular wages (c) Relative high wages (d) All of the above Ans. (d) All of the above 92. High out migration from Uttarakhand, Rajasthan, Madhya Pradesh and Eastern Maharashtra have brought serious imbalances in age and sex composition in these states. Which one of the following consequences of migration can be observed here? (a) Economic consequences (b) Cultural consequences (c) Environmental consequences (d) Demographic consequences Ans. (d) Demographic consequences 93. Which statement among the following is not correct regarding Indian Diaspora? Choose the correct option. (a) During colonial period, millions of the indentured labourers were sent abroad (b) Cultural consequences (c) Environmental consequences (d) Demographic consequences Ans. (d) Demographic consequences 94. What per cent of females move out from their parental houses following their marriage? (a) 77% (b) 65% (c) 55%

(d) 80% Ans. (b) 65% 95. Which one of the following year is correct regarding publishing first Human Development Report by the UNDP? (a) 1970 (b) 1980 (c) 1990 (d) 1995 Ans. (c) 1990 96. Which state has highest female literacy rate-2011? (a) Lakshadeep (b) Kerala (c) Goa (d) Maharashtra Ans. (b) Kerala 97. Who among the following was the author of the book ‘Small is beautiful’? (a) Maithus (b) Mahatma Gandhi (c) Schumacher (d) Brundtland Ans. (c) Schumacher 98. Which of the following scholar was the first one to voice his concern about the growing scarcity of resources as compared to the human population? (a) Sir Robert Malthus

(b) Schumacher (c) Brundtland (d) Mahatma Gandhi Ans. (a) Sir Robert Malthus 99. Human development is: (a) Always positive (b) Always negative (c) Both (a) and (b) (d) Partial positive and partial negative Ans. (a) Always positive 100. Which one of the following states of India has the lowest female literacy? (a) Sikkim (b) Assam (c) Jharkhand (d) Bihar Ans. (d) Bihar 101. Which one of the following is the beginning of access to a world of knowledge and freedom? (a) Literacy (b) Equality (c) Power (d) Money Ans. (a) Literacy 102. Which of the option is correct?

(a) GDP of country is only the factor that reflect the quality of life. (b) Factors like housing, access to public transport, air quality and access to drinking water also determine the standard of living. (c) GDP of country reflect only the production of foreign products (d) None of the above Ans. (b) Factors like housing, access to public transport, air quality and access to drinking water also determine the standard of living. 103. Which of the following age group according to findings of 2011 Census of India are very disturbing? (a) 0-6 years (b) 7-14 years (c) 15-19 years (d) 20- 25 years Ans. (a) 0-6 years 104. Which of the following state has the highest percentage of people below the poverty line? (a) UP (b) Odisha (c) Madhya Pradesh (d) Assam Ans. (b) Odisha 105. As compared to other countries, India’s place in terms of human development is (a) High (b) Low

(c) Medium (d) Very Low Ans. (c) Medium 106 What is the composite HDI value of India? (a) 0.802 (b) 0.702 (c) 0.640 (d) 0.502 Ans. (c) 0.645 107. Which agency is responsible for preparation and publishing the list of human development of the world countries? (a) WHO (b) UNESCO (c) UNICEF (d) UNDP Ans. (d) UNDP 108. What is the literacy rate of India on the basis of census 2011? (a) 74.04% (b) 66% (c) 92% (d) 82% Ans. (a) 74.04% 109. According to Neo-Malthusians thinkers, the gap between the resources and population has widened after which

century? (a) 17th (b) 18th (c) 19th (d) 20th Ans. (b) 18th 110. Name the country having highest human development index. (a) Norway (b) India (c) France (d) Japan Ans. (a) Norway 111. Which one of the following best describe development? (a) An increase in size (b) A positive change in quality (c) A constant in size (d) A simple change in quality Ans. (b) A positive change in quality 112. Arrange the state from low to high on the basis of percent of population below poverty line. (i) Arunachal Pradesh (ii) Nagaland (iii) Mizoram

(iv) Meghalaya (a) (ii), (iv), (iii), (i) (b) (i), (iii), (iv), (ii) (c) (ii), (i), (iv), (iii) (d) (iv), (i), (iii), (ii) Ans. (a) (ii), (iv), (iii), (i) 113. Which one of the following states of India has the lowest female child sex ratio in category of 0-6 years? (a) Gujarat (b) Haryana (c) Punjab (d) Himachal Pradesh Ans. (b) Haryana 114. Indian ________ and ________ have been very sensitive to the issues of population, resource and development for a long time. Fill in the blanks. (a) Culture, social behaviour (b) Social behaviour, civilisation (c) Culture, script (d) Culture, civilisation Ans. (d) Culture, civilisation 115. Which of the following states has the highest literacy rate? (a) Himachal Pradesh (b) Haryana

(c) Uttarakhand (d) Rajasthan Ans. (a) Himachal Pradesh 116. Which of the following Union Territories of India has the highest literacy rate? (a) Lakshadweep (b) Chandigarh (c) Daman and Diu (d) Andaman and Nicobar Islands Ans. (a) Lakshadweep 117. What is female literacy rate according to 2011, census of India? (a) 74.04% (b) 71.16% (c) 65.46% (d) 75.84% Ans. (c) 65.46% 118. Arrange the following state UTs from low to high in Human development Index. (i) Himachal Pradesh (ii) Delhi (iii) Goa (iv) Punjab (a) (ii), (iii), (i), (iv) (b) (iv), (iii), (ii), (i)

(c) (iv), (i), (iii), (ii) (d) (ii), (iii), (i), (iv) Ans. (c) (iv), (i), (iii), (ii) 119. Which one of the following state of India has the highest rank in the Human Development Index? (a) Tamil Nadu (b) Punjab (c) Kerala (d) Haryana Ans. (c) Kerala 120. Which one of the following state has the highest literacy rate? (a) Goa (b) Kerala (c) Tamil Nadu (d) Karnataka Ans. (b) Kerala 121. Which of the following programme of Government of India aims at a pollution-free environment? (a) Swachh Bharat Mission (SBM) (b) Construction of Individual Household Latrines (IHHL) (c) Community Toilet (CT) seats (d) Public Toilet (PT) seats Ans. (a) Swachh Bharat Mission (SBM) 122. Which of the following pair is not correctly matched?

REPORTS YEAR (a) Limits to Growth 1972 (b) Small is Beautiful 1974 (c) Our Common Future 1997 (d) Agenda-21 Report of 1993 the Rio Conference Ans. (c) Our Common Future 1997 123. Which statement is correct regarding Swachh Bharat Mission (SBM)? (a) Making India open defection free (b) Providing clean energy fuel LPG to all house hold in rural (c) Providing potable drinking water to every household (d) All of the above Ans. (d) All of the above 124. Kerala is able to record the highest value in the HDI. Select the correct option. (a) Hundred per cent literacy (b) Economic development (c) No Regional distortions and social disparities (d) Social distributive justice Ans. (a) Hundred per cent literacy 125. Which of the following statement is true? (a) Resource are distributed everywhere evenly. (b) Rich country has large basket of resources to access. (c) Rich country has less basket of resources to access.

(d) Least developed country has large basket of resources to access. Ans. (b) Rich country has large basket of resources to access.

1. Find odd one out: (a) Star-shaped (b) Linear (c) Hamleted (d) Circular Ans. (c) Hamleted 2. In which of the following parts of India, dispersed rural settlements are not expected? (a) Kerala (b) Chhattisgarh (c) Meghalaya (d) All of these Ans. (b) Chhattisgarh 3. Which of the following types of rural settlements is found in the fertile alluvial plains and northern and eastern parts of India? (a) Clustered (b) Semi-clustered (c) Hamleted (d) None of the above Ans. (a) Clustered 4. Find odd one out: (a) Clustered (b) Semi-clustered

(c) Hamleted (d) Linear Ans. (d) Linear 5. Which one of the following forms of settlements develops along either side of roads, rivers or canals? (a) Linear (b) Circular (c) Cross-shaped (d) Square Ans. (a) Linear 6. Which one of the following is not a medieval town? (a) Madurai (b) Hyderabad (c) Jaipur (d) Nagpur Ans. (a) Madurai 7. Complete the following figure with an appropriate option.

(a) Star-shaped (b) Linear (c) Fragmented (d) Hamleted Ans. (d) Hamleted

8. Which one of the following is a feature of semi- clustered settlements? (a) Dominance group live in the centre of the village. (b) Availability of water decides the shape of settlements. (c) It is based on farming. (d) They are isolated huts. Ans. (a) Dominance group live in the centre of the village. 9. In which of the following regions are the oldest welldocumented urban settlements found? (a) Mesopotamia (b) Huang-he valley (c) Nile valley (d) Indus valley Ans. (d) Indus valley 10. How many cities in India have a population of million plus in 2011? (a) 42 (b) 48 (c) 53 (d) 56 Ans. (c) 53 11. Which of the following is a functional classification of an urban settlement? (a) Administrative (b) Industrial (c) Mining town

(d) All of the above Ans. (d) All of the above 12. Which of the following statement is/are correct? (i) The British and Other Europeans have developed a number of towns in India. (ii) Bombay, Madras, Calcutta has been built in the French Style. (a) Only (i) (b) Only (ii) (c) Both (i) and (ii) (d) Neither (i) nor (ii) Ans. (a) Only (i) 13. Towns are classified based on evolution as: (a) Ancient towns (b) Modern towns (c) Medieval towns (d) All of the above Ans. (d) All of the above 14. The type of settlement that is locally known as Para, Pani, Nagal, and Dhani is: (a) Clustered (b) Semi-clustered (c) Hamleted (d) Dispersed Ans. (c) Hamleted

15. Which of these settlements is found along the coast? (a) Rectangular (b) Linear (c) Triangular (d) Star-like Ans. (b) Linear 16. The __________ relationship between rural and urban settlements takes place through transport and communication network. (a) Formal (b) Structural (c) Functional (d) Social Ans. (c) Functional 17. Which of the following Statement is correct? (a) Decennial Growth trend in urbanisation shows decline after 1981 (b) Decennial growth trend in urbanisation shows constant after 1981 (c) Decennial growth trends in urbanisation shows rise after 1981 (d) None of the above Ans. (a) Decennial Growth trend in Urbanisation shows decline after 1981 Explanation : Year Decennial Growth% 1981 46.14

1991 36.47 2001 31.13 2011 31.01 18. What do you understand by commercial towns? (a) Emerged as garrison towns (b) Primarily engaged in export and import (c) Specializing in trade and commerce (d) None of the above Ans. (c) Specialising in trade and commerce 19. Which of the following types of economic activities dominated all rural settlements? (a) Primary (b) Secondary (c) Territory (d) Quarternary Ans. (a) Primary 20. Which types of settlements are found in alluvial plains? (a) Clustered (b) Semi-clustered (c) Square (d) Triangle Ans. (a) Clustered 21. Which of the following cities is the largest agglomeration? (a) Delhi (b) Chennai

(c) Kolkata (d) Mumbai Ans. (d) Mumbai 22. In which of the environment does one expect the presence of clustered rural settlements? (a) Alluvial plains of Ganga (b) Arid and semi-arid regions of Rajasthan (c) The lower valley of the Himalayas (d) Forests and hills in northeast Ans. (a) Alluvial plains of Ganga 23. Where do people live in compact village for security or defence reasons? (a) Rajasthan (b) Bihar (c) Tripura (d) Nagaland Ans. (d) Nagaland 24. Srinagar town can be placed in among which of the following specialisation. (a) Administrative town (b) Industrial town (c) Transport town (d) Commercial town Ans. (a) Administrative town 25. Which one of these statements is correct? I. Ancient towns in India had developed as religious and cultural

centres. II. Medieval towns have historical background of over 2000 years. (a) Only I is correct (b) Only II is correct (c) Both are correct (d) Both are incorrect Ans. (a) Only I is correct 26. Consider the following statements: I. Clustered settlements are the result of the fragmentation of a large village. II. The dispersed settlement pattern in India appears in the form of isolated huts or hamlets of a few huts in remote jungles or on small hills with forms or pastures on the near slope. Choose the correct option: (a) Only I is correct (b) Only II is correct (c) Both I and II are correct (d) Both are incorrect Ans. (b) Only II is correct 27. Consider the following statements: I. Urban centres with a population of more than one lakh is called a city. II. The objective of the Smart Cities Mission is to demote cities that provide core infrastructure. Choose the correct option: (a) Only I is correct

(b) Only II is correct (c) Both I and II are correct (d) Both II and I are incorrect Ans. (a) Only I is correct 28. Match the following: Column I

Column II

I.

Settlements

(a) Homestead

II.

Megacity

(b) A cluster of dwellings where people live

III.

Rural settlements

(c)

Population over 5 million

Options : (a) I-b, II-a, III-c (b) I-c, II-a, III-b (c) I-b, II-c, III-a (d) I-a, II-c, III-b Ans. (c) I-b, II-c, III-a 29. Which of the following groups are correct? I.

Hamleted



Para, Palli, Naga, Dhani



Ganga plains, lower valleys of the Himalayas



Depend on processing raw materials and

settlements II.

Dispersed settlements

III.

Urban

settlements

manufacturing goods

Options : (a) I and II are correct (b) Only II is wrong (c) Only I is correct (d) Only III is correct Ans. (b) Only II is wrong 30. Match the following: Particulars

Particulars

I.

Urban settlements

(a)

Collective centres

II.

Meghalaya,

(b)

Hamleted settlements

(c)

Dispersed settlements

(d)

The population resides in city areas

Uttaranchal, and Himachal Pradesh III.

Mining towns

(a) I-d, II-c, III-a (b) I-c, II-d, III-a (c) I-d, II-a, III-b (d) I-b, II-c, III-d Ans. (a) I-d, II-c, III-a 31. Which of the following statements is incorrect? I. Urban settlements depend on the processing of raw materials and manufacturing of finished goods on one hand and a variety of

services on the other. II. The rural settlements derive their life support or basic economic needs from land-based primary economic activities. III. Urban settlements do not have adequate facilities for good living. Modern amenities are almost absent. (a) Both I and II are incorrect (b) Only III is incorrect (c) Both I and III are incorrect (d) All I, II and III are incorrect Ans. (b) Only III is incorrect 32. Which of the following is/are the characteristic(s) of urban settlements? I. These settlements are well-planned. II. These settlements are large. III. These settlements lack infrastructure facilities. (a) Only I is correct (b) Only II is correct (c) Both I and III are correct (d) Both I and II are correct Ans. (d) Both I and II are correct 33. Settlements are classified on the basis of: I. Occupation of the people II. Population size III. Gender and equality Options :

(a) Only II is correct (b) Only III is correct (c) Only I is correct (d) Both I and II are correct Ans. (d) Both I and II are correct 34. Which one of these is not a feature of urban settlements? I. These are smaller in size. II. The density of the population is not very high. III. They have modern facilities. IV. These settlements are well-planned. (a) Both I and II (b) Both III and IV (c) Both I and III (d) Both II and IV Ans. (a) Both I and II 35. Which one of the following is a mining town? (a) Jharia (b) Satna (c) Srinagar (d) Aligarh Ans. (a) Jharia 36. In which valley are Harappa and Mohenjo-Daro towns located? (a) Ganga (b) Narmada

(c) Indus (d) Brahmaputra Ans. (c) Indus 37. Which is an administrative town? (a) Varanasi (b) Surat (c) Gandhinagar (d) Rohtak Ans. (c) Gandhinagar 38. According to census 2011, how many towns are there in India? (a) 4161 (b) 7935 (c) 6161 (d) 7161 Ans. (b) 7935 39. Which one of these towns is not situated along the bank of a river? (a) Agra (b) Bhopal (c) Patna (d) Kolkata Ans. (b) Bhopal 40. Which one of the following is NOT a part of the definition of a town as per the census of India?

(a) Population density of 400 persons per sq. km. (b) Presence of municipality, corporation, etc. (c) More than 75% of the population engaged in the primary sector. (d) The population size of more than 5,000 persons. Ans. (c) More than 75% of the population engaged in the primary sector. 41. Which one of the following groups of cities has been arranged in the sequence of their ranks, i.e., 1, 2, 3, and 4 in size? (a) Greater Mumbai, Bengaluru, Kolkata, Chennai (b) Delhi, Greater Mumbai, Chennai, Kolkata. (c) Bengaluru, Greater Mumbai, Chennai, Kolkata. (d) Greater Mumbai, Kolkata, Delhi, Chennai. Ans. (d) Greater Mumbai, Kolkata, Delhi, Chennai.

1. Which one of the following crop is the second most important cereal crop in India? (a) Rice (b) Wheat (c) Maize (d) Bajra Ans. (b) Wheat 2. Which is known as golden fibre? (a) Cotton (b) Silk (c) Jute (d) Wool Ans. (c) Jute 3. In which season among the following the cultivation of watermelons, cucumbers, vegetables and fodder crops is done on irrigated lands? (a) Rabi (b) Kharif (c) Zaid (d) Rainy Ans. (c) Zaid 4. Which of the following months of the year are included in Kharif Cropping Season? (a) June-September

(b) October – March (c) April–June (d) July – October Ans. (a) June-September 5. The objective of this type irrigation is to protect the crops from adverse effects of soil moisture deficiency which often means that irrigation acts as a supplementary source of water over and above the rainfall. These are: (a) Rainfed farming (b) Dryland farming (c) Protective irrigation (d) Barani Ans. (c) Protective irrigation 6. Which one of the following crop India is the second-largest producer in the world? (a) Wheat (b) Coffee (c) Rice (d) Cotton Ans. (c) Rice 7. Which one of the following is the main form of degradation in irrigated areas? (a) Gully erosion (b) Wind erosion (c) Salinisation of soil (d) None of the above Ans. (c) Salinisation of soil

8. Which of the following is not a correct statement about tea? (a) Tea is a plantation crop used as beverage. (b) Black tea leaves are fermented whereas green tea leaves are unfermented. (c) Tea leaves have rich content of caffeine and tannin. (d) It is an indigenous crop of hills in northern Sri Lanka. Ans. (d) It is an indigenous crop of hills in northern Sri Lanka. 9. Which one of the following price is announced by the government in support of a crop? (a) Influential support price (b) Moderate support price (c) Minimum support price (d) Maximum support price Ans. (c) Minimum support price 10. Cotton need a frost-free period of: (a) 100 days (b) 150 days (c) 200 days (d) 210 days Ans. (d) 210 days 11. Which one of the following is true about India’s ranks in the world in the production of cotton? (a) First (b) Second (c) Third

(d) Fourth Ans. (b) Second 12. Which of the following statement is true for regarding National Mission for sustainable agriculture? (a) To make crop climate resistant (b) To conserve natural resources (c) Both (a) and (b) (d) Promotion of Organic farming should be least Ans. (c) Both (a) and (b) 13. Which among the following is a fibre crops that provide us fibre for preparing cloth, bags, sacks and a number of other items? (a) Cotton and coffee (b) Sugarcane and jute (c) Cotton and Sugarcane (d) Cotton and jute Ans. (d) Cotton and jute 14. Which two states of the following states together produce about 90 percent of total output of soyabean in the country? (a) Andhra Pradesh and Karnataka (b) Madhya Pradesh and Karnataka (c) Andhra Pradesh and Maharashtra (d) Madhya Pradesh and Maharashtra Ans. (d) Madhya Pradesh and Maharashtra 15. Which of the following is the important oilseeds grown in India? (a) Soyabean

(b) Sugarcane (c) Tea (d) Coffee Ans. (a) Soyabean 16. Which among the following is not a variety coffee? (a) Arabica (b) Robusta (c) Liberica (d) Tannin Ans. (d) Tannin 17. India grows both short staple (Indian) cotton as well as long staple (American) cotton. These cotton also known as? (a) Narma (b) Gauze (c) Gingham (d) Herringbone Ans. (a) Narma 18. How much time did chemical consumption increased in mid 1960s’? (a) 5 time (b) 10 time (c) 15 time (d) 20 time Ans. (c) 15 time 19. Tur is the second important pulse crop in the country. It also known as?

(a) Red gram (b) Pigeon (c) White gram (d) Both (a) and (b) Ans. (d) Both (a) and (b) 20. Which state is the largest wheat producing state? (a) Punjab (b) Haryana (c) Uttar Pradesh (d) Rajasthan Ans. (c) Uttar Pradesh 21. Which one of the following is NOT a land use category? (a) Follow land (b) Marginal land (c) Net area sown (d) Culturable wasteland Ans. (b) Marginal land 22. Current follow land is land which left uncultivated for period of: (a) More than 5 years (b) 3 to 5 years (c) 2 to 5 years (d) 1 or less than 1 year Ans. (d) 1 or less than 1 year 23. What is the name given to the physical extent of land on which crops are sown and harvested?

(a) Current Fallow (b) Culturable Waste-Land (c) Barren and Wastelands (d) Net Area Sown Ans. (d) Net Area Sown 24. Name the two countries where HYVs of wheat and rice were developed. (a) Maxico and Philippines (b) USA and Maxico (c) Maxico and Japan (d) Philippines and USA Ans. (a) Maxico and Philippines 25. Which of the following is not a Kharif crop? (a) Rice (b) Maize (c) Rapeseeds (d) Cotton Ans. (c) Rapeseeds 26. Which one of the following is the main reason due to which share of the forest has shown an increase in the last forty years? (a) Extensive and efficient efforts of afforestation (b) Increase in community forest land (c) Increase in the notified area allocated for forest growth (d) Better peoples participation in managing forest area Ans. (c) Increase in the notified area allocated for forest growth

27. Which one of the following is correct full form of CPR ? (a) Common Property Resources (b) Common Public Resources (c) Civil Property Resources (d) Common Property Results Ans. (a) Common Property Resources 28. Which state is the largest producer of Jowar in India? (a) Punjab (b) Maharashtra (c) Karnataka (d) Rajasthan Ans. (b) Maharashtra 29. Which of the following is NOT a Rabi crops? (a) Wheat (b) Gram (c) Mustards (d) Jowar Ans. (d) Jowar 30. Which of the following crop that does not grow under dryland farming? (a) Sugarcane (b) Wheat (c) Rice (d) Ragi Ans. (a) Sugarcane

31. Which department is meant for geographical area of administrative units?

measuring

the

(a) Agriculture Department (b) Forest Department (c) Revenue Department (d) Survey of India Ans. (d) Survey of India 32. Which of the following is a cash crop in West Bengal and adjoining eastern parts of the country? (a) Jute (b) Cotton (c) Ragi (d) Bajra Ans. (a) Jute 33. Which one of the following is a leguminous crop? (a) Pulses (b Millets (c) Jowar (d) Sesamum Ans. (a) Pulses 34. How much part of the earth surface is covered with water? (a) 51% (b) 61% (c) 71% (d) 81%

Ans. (c) 71% 35. How much freshwater constituents of the total water? (a) 0.5% (b) 1.0% (c) 2.5% (d) 3.0% Ans. (d) 3.0% 36. What is the share of India in the world’s water resources? (a) 1% (b) 2% (c) 3% (d) 4% Ans. (d) 4% 37. Nagarjunasagar Dam is located on which river? (a) River Narmada (b) River Tapi (c) River Krishna (d) River Damodar Ans. (c) River Krishna 38. How much percent of surface water in India can be used? (a) 22% (b) 25% (c) 32% (d) 35%

Ans. (c) 32% 39. Which chemical’s concentration has increased in water in Bihar? (a) Salt (b) Nitrates (c) Fluoride (d) Arsenic Ans. (d) Arsenic 40. How much percent groundwater is used in agriculture? (a) 72% (b) 82% (c) 92% (d) 85% Ans. (c) 92% 41. How much percent of the net sown area is irrigated in Punjab? (a) 65% (b) 75% (c) 80% (d) 85% Ans. (d) 85% 42. Consider the following and choose the correct answer from the given options: I. The groundwater utilisation is very high in the states of Punjab and Haryana. II. These states have more than 85 percent of their net sown area

under irrigation. Options: (a) Only statement I is correct. (b) Both the statements are correct and statement II correctly explains statement I. (c) Only statement II is correct. (d) Both the statements are incorrect. Ans. (b) Both the statements are correct and statement II correctly explains statement I. 43. Which one of the following types describes water as a resource? (a) Abiotic resources (b) Biotic resources (c) Non-renewable resources (d) Cyclic resources Ans. (d) Cyclic resources 44. In which of the following sectors water is used in the highest proportion in the country? (a) Irrigation (b) Domestic use (c) Industries (d) None of the above Ans. (a) Irrigation 45. Which of the following is a programme under watershed management? (a) Haryali

(b) Neeru-Meeru (c) Arvary Pani Sansad (d) All of the above Ans. (d) All of the above 46. In which year, the Government of India has launched “Jal Kranti Abhiyan”? (a) 2011-12 (b) 2013-14 (c) 2015-16 (d) 2017-18 Ans. (c) 2015-16 47. How did rainwater harvesting help in the development of certain areas in India? (a) Cheap and environment-friendly technique (b) Recharges groundwater (c) Prevent soil erosion (d) All of the above Ans. (d) All of the above 48. Consider the following and choose the correct answer from the given options: 1. India has a number of lagoons and lakes. 2. India has a vast coastline and the coast is very indented in some states. Options: (a) Only 1 is correct. (b) Only 2 is correct.

(c) Both the statements are incorrect. (d) Both the statements are correct and statement 2 correctly explains the statement 1. Ans. (d) Both the statements are correct and statement 2 correctly explains the statements 1. 49. Tick the correct reason for the shrinking of per capita availability of water. (a) Decreasing population (b) Increasing population (c) Illiteracy (d) Depleting water resources Ans. (b) Increasing population 50. To ensure ______________ we need to be such a citizen who is responsible for sensitizing people with positive cooperation about the necessity of water conservation. (a) Cultural development (b) Economic development (c) Sustainable development (d) All of the above Ans. (c) Sustainable development 51. Which state has made the water harvesting structure in the houses compulsory? (a) Maharashtra (b) Tamil Nadu (c) Haryana (d) U.P.

Ans. (b) Tamil Nadu 52. Which state government has launched the Neeru-Meeru programme? (a) Andhra Pradesh (b) Haryana (c) Bihar (d) U.P. Ans. (a) Andhra Pradesh 53. Where was Arvary Pani Sansad has established? (a) Bihar (b) Jammu and Kashmir (c) Alwar, Rajasthan (d) U.P. Ans. (c) Alwar, Rajasthan 54. Tick the correct cause of water deterioration in India. (a) Toxic substances (b) Foreign matters, such as micro organism, chemical (c) Both (a) and (b) (d) Water recycling and reuse Ans. (c) Both (a) and (b) 55. When was the Water (Prevention and Control of Pollution) Act passed? (a) 1974 (b) 1947 (c) 7419

(d) 1497 Ans. (a) 1974 56. Tick from the following the proper water conservation techniques adopted in India? (a) Rainwater harvesting (b) Watershed management (c) Water meters (d) Both (a) and (b) Ans. (d) Both (a) and (b) 57. Which one is the positive effect of irrigation? (a) Multiple cropping possible (b) Wide yielding varieties of crop (c) Green revolution strategy (d) All of the above Ans. (d) All of the above (Map Based Questions) Study the map carefully and answer the questions that follow.

58. Which state of India has the highest utilisation of groundwater? (a) Goa (b) Kerala (c) Punjab (d) Chhattisgarh Ans. (c) Punjab 59. Which state of India utilises a small proportion of groundwater? (a) Madhya Pradesh (b) Chhattisgarh (c) Uttar Pradesh (d) Himachal Pradesh Ans. (b) Chhattisgarh 60. Which state of India utilises groundwater at a moderate rate? (a) Gujarat (b) Uttar Pradesh (c) Punjab (d) Haryana Ans. (a) Gujarat 61. Which state of India has vast surface water resources in the lagoons and lakes? (a) Tamil Nadu (b) Jammu and Kashmir (c) Kerala

(d) Assam Ans. (c) Kerala 62. In which state of India green revolution strategy of agriculture was successful? (a) Haryana (b) Bihar (c) West Bengal (d) Jammu and Kashmir Ans. (a) Haryana 63. Which of the statements given below is true? I. Rainwater harvesting increases ground water recharge by capturing and storing rainwater locally in subsurface water reservoirs to meet household needs. II. Rainwater harvesting is a technique of decreasing the recharge of tank water by capturing and storing rainwater locally in subsurface water reservoirs to meet industrial needs. (a) Only I is true (b) Only II is true (c) Both I and II are true (d) Both I and II are false Ans. (a) Only I is true 64. Which one of the following pairs is correct? I.

Rainwater



harvesting II.

Conservation of

Recharge of groundwater



Preventing pollution

water III.

of water

Sustaining water



Short time

supply (a) Both I and II are true (b) Only I is true (c) Only II is true (d) Only III is true Ans. (b) Only I is true 65. Match the following: Column I I.

Water quality

Column II (a)

Sustainable development

II.

Watershed

(b)

management III. Water conservation

Unwanted foreign substances

(c)

To control damaging runoff and degradation

(a) I-c, II-a, III-b (b) I-b, II-a, III-c (c) I-b, II-c, III-a (d) I-a, II-c, III-b Ans. (c) I-b, II-c, III-a 66. Which of the following are not the causes of water scarcity? I. Growing population

II. Water harvesting technique III. Expansion of irrigation IV. Rooftop water harvesting (a) Both I and III (b) Both II and IV (c) Only II (d) Only IV Ans. (b) Both II and IV 67. Which resources?

among

the

following

is/are

surface

water

(a) Rivers (b) Lakes (c) Ponds (d) All of these Ans. (d) All of these 68. Which of the following statement(s) given below is/are correct? I. Water scarcity is the practice of using water efficiently to reduce unnecessary usage or wastage of water. II. Water conservation is the practice of using water efficiently to reduce unnecessary usage or wastage of water. (a) Only I (b) Only II (c) Both I and II (d) Neither I nor II Ans. (b) Only II

69. Which objective of watershed management given below is correct? I. To create a balance among natural elements as well as in society. II. To enable the villagers to conserve water for various uses such as drinking, irrigation, fisheries and afforestation. (a) Only I (b) Only II (c) Both I and II (d) Neither I nor II Ans. (c) Both I and II 70. “The assessment, efficient use and conservation of water are necessary to ensure development”. Identify the values enshrined in the statement. I. Sustainable development II. Social awareness III. Environmental destruction (a) Only I (b) Only II (c) Only III (d) Both I and II Ans. (d) Both I and II 71. Which of the following values lead to water scarcity? I. Better utilisation of resources II. Overutilisation of resources III. Environmental upgradation

IV. Social pressure (a) Both I and III (b) Both II and IV (c) Both I and II (d) Both III and IV Ans. (b) Both II and IV 72. Match the following: Column I (a)

Rainwater

Column II I.

Agricultural sector

harvesting (b)

Water scarcity

II.

Recharge well

(c)

The exploitation of groundwater resources

III.

Lack of freshwater resources

(a) (a)-II, (b)-III, (c)-I (b) (a)-III, (b)-I, (c)-II (c) (a)-I, (b)-III, (c)-II (d) (a)-II, (b)-I, (c)-III Ans. (a) (a)-II, (b)-III, (c)-I 73. What does CPCB stand for? (a) The Commandant Pollution Control Board (b) The Central Pollution Control Board (c) The Central Polythene Control (d) The Centre Passage Control Ans. (b) The Central Pollution Control Board

74. Which of the statements given below is correct? I. Watershed development is related to Haryali Programme. II. Van Mahotsava is related to the watershed programme. III. Neeru-Meeru is not related to the watershed programme. (a) Only I is correct (b) Only II is correct (c) Only III is correct (d) None of these Ans. (a) Only I is correct 75. Which of the pairs given below is/are true? I. Sources of water – rainfall, surface water and groundwater II. Need of irrigation – Unlimited rainfall III. Traditional rainwater harvesting – Tanka or Kund (a) Only I (b) Only II (c) Only III (d) Both I and III Ans. (d) Both I and III 76. What are the measures of water conservation? (a) Dams on rivers (b) Recycling of polluted water (c) Growing selected crops (d) All of the above Ans. (d) All of the above

77. What are the factors responsible for the depletion of water resources? (a) Industrial (b) Agricultural (c) Both (a) and (b) (d) None of these Ans. (c) Both (a) and (b) 78. Which one of the rainwater harvesting objectives given below is true? I. To meet the ever-decreasing demand for water. II. To avoid flooding of roads. III. To improve the quality of groundwater IV. To increase underground water pollution (a) Only I (b) Only IV (c) Both I and III (d) Both II and IV Ans. (c) Both I and III 79. Which state uses the highest amount of groundwater? (a) Punjab (b) Tamil Nadu (c) Haryana (d) All of these Ans. (d) All of these 80. Name the low-cost techniques to recharge groundwater?

(a) Roof-water harvesting (b) Recharging of hand pumps (c) Both (a) and (b) (d) Locate more tube well where water tabel is high Ans. (c) Both (a) and (b) 81. Which of the statements given below is true? I. India accounts for about 2.45 percent of the world’s surface area, 4 per cent of the world’s water resources and about 16 percent of the world’s population. II. The total water available from precipitation in the country in a year is about 10,000 cubic km. (a) Only II is correct (b) Only I is correct (c) Both I and II are correct (d) None of the above is correct Ans. (b) Only I is correct 82. Identify the non-metal from the following list. (a) Iron ore (b) Limestone (c) Gold (d) Copper Ans. (b) Limestone 83. Identify the metal from the following list. (a) Coal (b) Petroleum

(c) Bauxite (d) Mica Ans. (c) Bauxite 84. Which of the following is not a characteristic of minerals? (a) There is an inverse relationship in quality and quantity of minerals. (b) Minerals are unevenly distributed over space. (c) All minerals are exhaustible over time. (d) All minerals are inorganic in origin. Ans. (d) All minerals are inorganic in origin. 85. Among the following places where do we find petroleum reserves? (a) Sedimentary basins of Assam (b) Damodar (c) Sone (d) Mahanadi Ans. (a) Sedimentary basins of Assam 86. Which of the following regions does not fall under the North-Eastern Plateau belt? (a) West Bengal (b) Chota Nagpur (c) Odisha Plateau (d) Assam Ans. (d) Assam 87. Which of the following belts lacks in coal deposits?

(a) The North-Eastern Plateau Region (b) The North-Western Region (c) The South-Western Plateau Region (d) The South-Eastern Region Ans. (c) The South-Western Plateau Region 88. Which of the following belts extends along Aravali in Rajasthan? (a) The North-Eastern Plateau Region (b) The North-Western Region (c) The South-Western Plateau Region (d) The South-Eastern Region Ans. (b) The North-Western Region 89. Which one of the following resources is abundantly available in Gujarat? (a) Mineral oil deposits (b) Petroleum deposits (c) Gypsum (d) Fuller’s earth deposits Ans. (b) Petroleum deposits 90. Which of the following minerals serves as a raw material for smelting of iron ore? (a) Manganese (b) Iron Ore (c) Bauxite (d) Copper

Ans. (a) Manganese 91. Goa has become an important producer of _____. (a) Manganese (b) Iron Ore (c) Bauxite (d) Copper Ans. (b) Iron Ore 92. Which among the following manufacturing ferro alloys?

metals

is

used

for

(a) Manganese (b) Iron Ore (c) Bauxite (d) Copper Ans. (a) Manganese 93. _____ is the leading producer of Manganese. (a) West Bengal (b) Rajasthan (c) Bihar (d) Odisha Ans. (d) Odisha 94. Which of the following is considered to be the most important non-metallic mineral in India? (a) Mica (b) Limestone

(c) Dolomite (d) Phosphate Ans. (a) Mica 95. Which of the following is not a conventional source of energy? (a) Coal (b) Petroleum (c) Geo-thermal energy (d) Nuclear energy minerals Ans. (c) Geo-thermal energy 96. Which of the following is a conventional source of energy? (a) Wind energy (b ) Nuclear energy minerals (c) Geo-thermal energy (d) Bio-energy Ans. (b) Nuclear energy minerals 97. In which of the following industry, Mica is mainly used? (a) Agricultural industry (b) Iron and steel industry (c) Thermal power industry (d) Electrical and electronic industries Ans. (d) Electrical and electronic industries 98. Where are Gondwana coal fields of India located?

(a) Singareni (b) Korba (c) Rampur (d) Damodar Valley Ans. (d) Damodar Valley 99. When was the Gas Authority of India established? (a) 1984 (b) 1977 (c) 1965 (d) 1956 Ans. (a) 1984 100. When was the Oil and Natural Gas Commission set up in India? (a) 1956 (b) 1984 (c) 1953 (d) 1982 Ans. (a) 1956 101. Petroleum is also referred as _____. (a) liquid silver (b) liquid gold (c) liquid copper (d) liquid uranium Ans. (b) liquid gold 102. How many types of oil refineries exist in India?

(a) Four (b) Three (c) Two (d) One Ans. (c) Two 103. Crude petroleum occurs in _____ of the tertiary period. Choose the correct option: (a) igneous rocks (b) metamorphic rocks (c) sedimentary rocks (d) granite Ans. (c) sedimentary rocks 104. Read the following information and identify the substance : (I) It consists of hydrocarbons of liquid and gaseous states varying in chemical composition. (II) It is an essential source of energy for all internal combustion engines in automobiles, railways and aircraft. (III) Its numerous by-products are processed in petrochemical industries. Choose the correct option: (a) Crude Oil (b) Tertiary coals (c) Natural Gas (d) Nuclear energy

Ans. (a) Crude Oil 105. Which among the following is the largest coal field followed by Raniganj? (a) Giridih (b) Bokaro (c) Jharia (d) Karanpura Ans. (c) Jharia 106. What is the other name of brown coal? (a) Bituminous coal (b) Anthracite (c) Cannel coal (d) Lignite Ans. (d) Lignite 107. Which of the following mineral provides raw materials for the cement industry? I. Dolomite II. Limestone III. Granite IV. Marble Options: (a) I and II (b) I, II and III (c) III and IV (d) II, III and IV Ans. (a) I and II 108. Which of the following information about energy resources is incorrect? (a) Mineral fuels are essential for the generation of power, required by agriculture, industry and transport. (b) Mineral fuels are conventional sources of energy.

(c) Mineral fuels include coal, petroleum and natural gas. (d) These conventional sources of energy are in exhaustible sources. Ans. (d) These conventional sources of energy are in exhaustible sources. 109. Consider the following statements. I. Singareni collieries is the premier coal production company. II. It uses canaries to detect the presence of deadly carbon monoxide in underground mines. III. According to a coal miner, cage of birds is a bad indicator in air containing more than 0.15 per cent CO. Which of the above statement(s) is/are correct about Singareni Collieries? (a) II and III (b) I and III (c) I and II (d) Only I Ans. (c) I and II 110. Identify the substance with the help of the following information. (I) It is an ore which is used in manufacturing of aluminium. (II) It is mainly found in tertiary deposits. (III) Odisha happens to be its largest producer. Choose the correct option: (a) Iron Ore (b) Bauxite

(c) Mica (d) Copper Ans. (c) Bauxite 111. Which of the following district of Andhra Pradesh is famous for producing best quality Mica? (a) Guntur (b) Srikakulam (c) Chittoor (d) Nellore Ans. (d) Nellore 112. Match the following column and choose the right option: Column A

Column B

(A)

Atomic Energy Commission

i.

1967

(B)

Atomic Energy Institute at Trombay

ii.

1948

(C)

Bhabha Atomic Research Centre

iii.

1956

(D)

Oil and Natural Gas Commission

iv.

1954

Options : A B C D (a) (i) (iii) (iv) (ii) (b) (iv) (ii) (iii) (i) (c) (ii) (iv) (i) (iii) (d) (iii) (i) (ii) (iv) Ans. (c) (ii) (iv) (i) (iii)

113. Which of the following states have rich sources of salt? I. Jammu II. Rajasthan III. Odisha IV. Gujarat Options : (a) Only I (b) II and IV (c) II, III, and IV (d) I, III and IV Ans. (b) II and IV 114. In which one of the following States are the major oil fields located? [NCERT] (a) Assam (b) Bihar (c) Rajasthan (d) Tamil Nadu Ans. (a) Assam 115. Read the following information given below carefully. (I) It processes the waste and garbage to produce energy. (II) It is a potential source of energy conversion. (III) It can be converted into electrical energy, heat energy or gas for cooking. Identify the form of energy from the following options: Choose the correct option: (a) Solar energy

(b) Tidal and wave energy (c) Geothermal energy (d) Bio-energy Ans. (d) Bio-energy 116. At which one of the following places was the first atomic power station started? [NCERT] (a) Kalpakkam (b) Narora (c) Rana Pratap Sagar (d) Tarapur Ans. (d) Tarapur 117. At which country was the first successful attempt to tap the underground heat held? (a) USA (b) Germany (c) China (d) French Ans. (a) USA 118. In which kind of rocks does Uranium deposits occur? (a) Vindhyan rock (b) Archaean rock (c) Dharwar rock (d) Gondwana rock

Ans. (c) Dharwar rock 119. Which one of the following minerals is known as brown diamond? [NCERT] (a) Iron (b) Lignite (c) Manganese (d) Mica Ans. (b) Lignite 120. Read the following statements on non-conventional energy sources and choose the correct option. I. Non-conventional energy sources use exhaustible raw materials. II. These energy sources are more equitably distributed and environmentally friendly. III. This energy provides more sustained, eco-friendly and cheaper energy. IV. Fossil fuel sources such as coal, petroleum, natural gas and nuclear energy are non-conventional energy sources. Choose the correct option: (a) Only I and II are true. (b) Only II and III are true. (c) Only II, III, and IV are true. (d) All of the above statements are true. Ans. (d) All of the above statements are true. 121. Which one of the following is non-renewable source of energy?

[NCERT] (a) Hydel (b) Solar (c) Thermal (d) Wind power Ans. (c) Thermal 122. Which sector among the following industries of strategic and national importance are classified? (a) Public sector (b) Private sector (c) Joint sector (d) Cooperative sector Ans. (a) Public sector 123. Where was the first port-based plant, which started operating in 1992, set up? (a) Andhra Pradesh (b) Madhya Pradesh (c) Maharashtra (d) Uttar Pradesh Ans. (a) Andhra Pradesh 124. In which of the following states is Rourkela steel plant located? (a) Jharkhand (b) Bihar (c) West Bengal

(d) Odisha Ans. (d) Odisha 125. The concentration of jute industry in West Bengal is in this region: (a) Kolkata and Durgapur (b) Kolkata and Mid nagpur (c) Haora and Bhatapara (d) None of the above Ans. (c) Haora and Bhatapara 126. Location of industries is influenced by several factors. There is strong relationship between two of these. These factors are: (a) Raw material and type of industry (b) Raw materials and power (c) Market and capital (d) Transport and labour Ans. (a) Raw material and type of industry 127. Which one of the following industry uses a non-weightlosing raw material and is generally located in large urban centre, like - Mumbai, Ahmedabad, Surat, etc.? (a) Petroleum refineries (b) Cotton Textile (c) Food industries (d) Iron ore industries Ans. (b) Cotton Textile 128. Which one of the following industry is basic to the

industrial development of any country? (a) The iron and steel industry (b) The sugar Industry (c) The petrol-chemical industry (d) The IT industry Ans. (a) The iron and steel industry 129. When was the New Industrial Policy announced? (a) 1900 (b) 1989 (c) 1991 (d) 2000 Ans. (c) 1991 130. _______ is a “pure” raw material which does not lose weight in the manufacturing process. (a) Iron (b) Copper (c) Aluminium (d) Cotton Ans. (d) Cotton 131. Which of the following was the initial name of Visvesvaraya Iron and Steel Works Ltd. (VISL)? (a) Mysore Iron and Steel Works (b) Tata Iron and Steel Company (c) Visvesvaraya Iron and Steel Works Ltd. (d) Rourkela Steel Plant

Ans. (a) Mysore Iron and Steel Works 132. Which one of the following state has the largest number of mills and most of them produce yarn rather than cloth? (a) Maharashtra (b) Gujarat (c) Karnataka (d) Tamil Nadu Ans. (d) Tamil Nadu 133. Which city is known as Manchester of North India? (a) Kanpur (b) Patna (c) Meerut (d) Agra Ans. (a) Kanpur 134. Which among the following industry is the second most important agro-based industry in the country? (a) Sugar industry (b) Cotton industry (c) Iron and steel industry (d) Petrochemical Industry Ans. (a) Sugar industry 135. Which of the following statement is/are true? (a) Foreign direct investment act as supplement to the domestic investment (b) FDI benefits the domestic industry

(c) Both (a) and (b) (d) FDI demolish technological upgradation Ans. (d) Both (a) and (b) 136. Which of the following is not a major centre of the cotton textile industry, while all other centres are the traditional centres and are located close to the cotton producing regions? (a) Ahmedabad (b) Solapur (c) Kolhapur (d) Agra Ans. (d) Agra 137. The Rourkela steel plant set up with the help of which country? (a) Germany (b) Russia (c) USA (c) England Ans. (a) Germany 138. Which is an example of foot loose industry? (a) Iron and steel (b) Cotton textile (c) Cement (d) Electronic Ans. (d) Electronic 139. Which of the following state has emerged as a leading

sugar producer in the country and produces more than onethird of the total production of the sugar in the country? (a) Punjab (b) Haryana (c) Madhya Pradesh (d) Maharashtra Ans. (d) Maharashtra 140. Which is the largest producer of sugar in India? (a) Uttar Pradesh (b) Maharashtra (c) Andhra Pradesh (d) Tamil Nadu Ans. (a) Uttar Pradesh 141. The ________ revolution opened up new possibilities of economic and social transformation. Fill in the blanks. (a) ITES (b) BPO (c) Chemical (d) Information Technology Ans. (d) Information Technology 142. A city also refer as silicon city: (a) Mumbai (b) Jaipur (c) Agra (d) Bengaluru

Ans. (d) Bengaluru 143. Which is an example of Pure Raw Material? (a) Cotton (b) Iron ore (c) Bauxite (d) Crude oil Ans. (a) Cotton 144. In which of the following cities of Gujarat has the largest petroleum refinery been set up? (a) Jamnagar (b) Ahmedabad (c) Surat (d) Rajkot Ans. (a) Jamnagar 145. The establishment of first jute mill at......... (a) Pune in 1855 (b) Kanpur in 1856 (c) Midnapur in 1855 (d) Rishra in 1855 Ans. (d) Rishra in 1855 146. Which one of the following is not true about Globalisation in the Indian context ? (a) It provides facilities to foreign companies to invest in different fields of economies activity in India. (b) It removes restrictions and obstacles to the entry of multinational companies in India

(c) It allows Indian companies to enter into foreign collaboration. (d) It do not liberalised industrial location programme. Ans. (d) It do not liberalised industrial location programme. 147. In which year Suez Canal was opened that provided impetus to the growth of Mumbai port? (a) 1869 (b) 1969 (c) 1870 (d) 1970 Ans. (a) 1869 148. In which state Jamshedpur iron and steel industry situated? (a) West Bengal (b) Jharkhand (c) Bihar (d) Odisha Ans. (b) Jharkhand 149. Which of the following industry in Gurugram-DelhiMeerut Region is a recent addition? (a) Synthetic fabrics industry (b) Cement industry (c) Vanaspati industry (d) Software industry Ans. (d) Software industry 150. The National Organic Chemicals Industries Limited (NOCIL), established in private sector in 1961, started the first naphtha based chemical industry at which place.

Choose the correct option: (a) Pimpri (b) Mettur (c) Barauni (d) Mumbai Ans. (d) Mumbai 151. This industry has achieved a remarkable distinction for providing quality products. A large number of Indian companies have acquired international quality certification. A majority of the multinational companies operating have either software development centres or research development centres in India. Above mentioned line is an important characteristic of which of the following industry ? (a) Sugar industry (b) Iron and steel industry (c) Cotton industry (d) Software industry Ans. (d) Software industry 152. Manufacturing is an activity of which type: (a) Primary (b) Secondary (c) Tertiary (d) Quarternary Ans. (ii) Secondary 153. First cotton textile industry was founded in ___________ .

(a) Ahmedabad (b) Fort Glaster (c) Mumbai (d) Kanpur Ans. (b) Fort Glaster 154. When was NITI Aayog formed and it replaced Planning Commission? (a) 1st January, 2015 (b) 1st January 2016 (c) 11th January 2016 (d) 11th January 2015 Ans. (b) 1st January 2016 155. When was the Third Five Year Plan launched? (a) 1960 (b) 1961 (c) 1964 (d) 1965 Ans. (b) 1961 156. When was the Fifth Five Year Plan terminated by the government? (a) 1974 (b) 1970 (c) 1978 (d) 1975 Ans. (c) 1978

157. When was the Twelfth Five Year Plan initiated? (a) 2012 (b) 2007 (c) 2002 (d) 1997 Ans. (a) 2012 158. What does CADP stands for? (a) Colonial Area Development Programme (b) Common Area Development Process (c) Command Area Development Programme (d) Consolidated Area Development Programme Ans. (c) Command Area Development Programme 159. Which of the following are the tributaries of river Ravi? (A) Budhil (B) Tundahen (C) Jhelum (D) Panjnad Options : (a) A, C, and D (b) A and B (c) C and D (d) A and D Ans. (b) A and B 160. What is the full form of SFDA? (a) Small Farmers Development Area

(b) Small Farmers Development Administration (c) Small Farmers Development Authority (d) Small Farmers Development Agency Ans. (d) Small Farmers Development Agency 161. Which of the following committees reviewed the Drought Prone Area Programme? (a) National Committee on Development of Backward Areas (b) National Committee of Agricultural Development (c) National Committee of Rural Development (d) National Committee of Transportation Network Ans. (a) National Committee on Development of Backward Areas 162. Which of the following villages are located along the river Ravi do not have sufficient infrastructure? (A) Holi (B) Khani (C) Tundah (D) Kugti Options : (a) A, C and D (b) A and B (c) C and D (d) A and D Ans. (c) C and D 163. The Planning Commission of India was established in the year _______.

(a) 1956 (b) 1950 (c) 1978 (d) 1984 Ans. (b) 1950 164. In which of the following regions did the Gaddis migrate to? (a) Bharmaur (b) Tundah (c) Kangra (d) Kugti Ans. (c) Kangra 165. ITDP refers to which one of the following ? [NCERT] (a) Integrated Travel Development Projects (b) Integrated Tourism Development Projects (c) Integrated Tribal Development Projects (d) Integrated Transport Development Projects Ans. (c) Integrated Tribal Development Projects 166. Who among the following wrote ‘The Population Bomb’? (a) Ehrlich (b) Rudolf Virchow (c) Alexander Fleming (d) Meadows

Ans. (a) Ehrlich 167. Who among the following wrote ‘The Limits to Growth’? (a) Ehrlich (b) Rudolf Virchow (c) Alexander Fleming (d) Meadows Ans. (d) Meadows 168. When was ‘Our Common Future’ published? (a) 1965 (b) 1987 (c) 1967 (d) 1978 Ans. (b) 1987 169. Identify the personality with the help of the following information: I. She is a Norwegian Prime Minister. II. She is the chairperson of WCED. III. She submitted a report which was titled as ‘Our Common Future’. Options : (a) Margaret Chan (b) Lee Jong Wook (c) Kare Willoch (d) Gro Harlem Brundtland Ans. (d) Gro Harlem Brundtland

170. Regional Planning relates: [NCERT] (a) Development of various sectors of economy (b) Area differences in transportation network (c) Area specific approach of development (d) Development of rural sectors Ans. (c) Area specific approach of development 171. Which one of the following is the most crucial factor for sustainable development in Indira Gandhi Command Area? [NCERT] (a) Eco-development (b) Agricultural Development (c) Transport development (d) Colonisation of land Ans. (a) Eco-development 172. What was the Indira Gandhi canal previously known as? (a) Bikaner canal (b) Rajasthan canal (c) Jaisalmer canal (d) Jodhpur canal Ans. (b) Rajasthan canal 173. In which of the following fields Canal irrigation has helped to bring a change ? (a) Agricultural economy (b) Transport facility

(c) Eco-development (d) Small scale industries Ans. (a) Agricultural economy 174. What percentage of the command area is to be irrigated by flow system? Choose the correct option. (a) 60 percent (b) 75 percent (c) 70 percent (d) 80 percent Ans. (a) 70 percent

1. Which one of the following roadways is the longest? (a) National Highway (b) State Highway (c) Major District Road (d) Rural Road Ans. (d) Rural Road 2. Arrange the following Waterways in a sequential order according to their stretches and select the answer from the given options : 1. NW-1 2. NW-2 3. NW-3 4. NW-4 Options: (a) 1 4 2 3 (b) 4 1 3 2 (c) 2 3 1 4 (d) 3 2 4 1 Ans. (a) 1 4 2 3 3. Radio club of Bombay, Indian Broadcasting System and All India Radio are the examples of which of the following communication system? (a) Mass communication

(b) Personal communication (c) Satellite communication (d) Television communication Ans. (a) Mass communication 4. Which of the following years is associated with air transport being nationalised? (a) 1947 (b) 1951 (c) 1911 (d) 1953 Ans. (d) 1953 5. The road built by Sher Shah Suri to strengthened and consolidate his empire from the Indus Valley to the Sonar Valley in Bengal is : (a) NH-4 (b) GT road/Shahi road (c) NH-1 (d) NH-2 Ans. (a) GT road/Shahi road 6. Who among the following entrusted with the function of development, maintenance and operation of National Highways? (a) PRY (b) DRDO (c) BRO

(d) NHAI Ans. (d) NHAI 7. Which one of the following is not a gauge of the Indian Railways? (a) Very Broad (b) Broad (c) Metre (d) Narrow Ans. (a) Very Broad 8. Which one of the following Railways is considered an engineering marvel? (a) Konkan Railway (b) South Western Railway (c) Central Railway (d) North Eastern Railway Ans. (a) Konkan Railway 9. Which one of the following is a correctly pair of Railway Zone and its headquarter? (a) Central- Mumbai CST (b) Eastern-Bhubaneswar (c) East Central - Kolkata (d) East Coast- Hajipur Ans. (a) Central- Mumbai CST 10. Which one of the following is not included in Bharatmala, which is a proposed umbrella scheme for the development of

roads and highways ? (a) State Roads (b) Backward areas, religious and tourist places (c) Longest road of the world (d) District headquarter connectivity Ans. (c) Longest road of the world 11. Which one of the following is not included in the NorthSouth and the East West corridors? (a) Srinagar (b) Kanyakumari (c) Silcher (d) Delhi Ans. (d) Delhi 12. In how many zones has the Indian Railways system been divided? [NCERT] (a) 9 (b) 12 (c) 16 (d) 14 Ans. (c) 16 13. According to the Annual Report 2017 which of the following depicts the percentage of District Roads of the total length of the roads in the country? (a) 15 per cent

(b) 15.5 per cent (c) 14 per cent (d) 14.5 per cent Ans. (c) 14 per cent 14. The Border Road organization was established in the year : (a) 1950 (b) 1960 (c) 1970 (d) 1980 Ans. (b) 1960 15. Which one of the following is the longest highway of India? [NCERT] (a) NH-l (b) NH-6 (c) NH-7 (d) NH-8 Ans. (c) NH-7 16. Which one of the following is the link between Srinagar and Muzaffarabad? (a) Aman Setu (b) Atal Tunnel (c) Khardung La Pass (d) Setu Bharatam Pariyojana Ans. (a) Aman Setu

17. The Urban transport system in Kolkata and Delhi is revolutionised by : (a) Metro Rail (b) National Highways (c) District Road (d) CNG Buses Ans. (a) Metro Rail 18. In which of the following year Indian Railways was introduced ? (a) 1853 (b) 1854 (c) 1855 (d) 1856 Ans. (a) 1853 19. In which of the following year the Inland waterways Authority was set up for the development, maintenance and regulation of the National Waterways in India? (a) 1984 (b) 1985 (c) 1986 (d) 1987 Ans. (c) 1986 20. On which river and between which two places does the National Water Way No. 1 lie? [NCERT]

(a) The Brahmaputra, Sadiya-Dhubri (b) The Ganga, Haldia-Allahabad (c) West Coast Canal, Kottapuram to Kollam (d) None of the above Ans. (b) The Ganga, Haldia-Allahabad 21. Which of the following corporations manage Air Transport in India? (a) Air India (b) Indian Airlines (c) Pawan Hans (d) All of these Ans. (d) All of these 22. Which one of the following is a good example of transporting liquid and gases over long distances? (a) Highways (b) Inland Waterways (c) Pipelines (d) Railways Ans. (c) Pipelines 23. Which one of the following is like a huge Central warehouse of data with detailed information on various items? (a) Internet (b) E-mail (c) Newspaper

(d) E-commerce Ans. (a) Internet 24. Which one of the following is the apex body to improve the quality of roads designated as National Highways? (a) NHAI (b) DRDO (c) HAI (d) BRDO Ans. (a) NHAI 25. Which one of the following count is correct about major and minor and intermediate ports? (a) 11 Major and 185 Minor intermediate ports (b) 12 Major and 185 Minor intermediate ports (c) 13 Major and 205 Minor intermediate ports (d) 14 Major and 201 Minor intermediate ports Ans. (b) 12 Major and 185 Minor intermediate ports 26. Which among the following is correct about the first Asia’s Cross country pipeline? (a) Assam to Baruni in Bihar (b) Assam to Kanpur (c) Baruni to Kanpur (d) Ankleshwar to Koyali Ans. (a) Assam to Baruni in Bihar 27. Which among the following attributes is correct about Radio broadcasting?

(a) It was started in 1924 (b) It was started by Radio Club of Bombay (c) Government brought this under its control in 1931 (d) It was changed to All India Radio in 1957 Ans. (b) It was started by Radio Club of Bombay 28. Considered the following statements and choose the correct answer from the given options: 1. Human beings have communication over times.

evolved

different

methods

of

2. Initially, the means of communication were also the means of transportation. Options : (a) Only 1 is correct (b) Only 2 is correct (c) Both the statements are incorrect (d) Both the statements are correct Ans. (d) Both the statements are correct 29. Which is the longest National Highway in India? (a) NH-1 (b) NH-5 (c) NH-44 (d) NH-7 Ans. (d) NH-7 30. “Indian railways brought people of diverse cultures together to contribute to India’s freedom struggle.” Above mentioned line is a statement of which of the following

persons ? (a) Jawaharlal Nehru (b) Rajendra Prasad (c) B. Rao Ambedkar (d) Mahatma Gandhi Ans. (d) Mahatma Gandhi 31. Which of the following statements is not true regarding the Konkan Railways? (a) Areas around towns raw material producing areas and of plantations and other commercial crops hill stations and cantonment towns were well connected by railways from the British colonial era. (b) These were mostly developed for the exploitation of resources. (c) After the independence of the country railway routes have been extended to other areas too. (d) The most significant development has been the development of Konkan railway along the eastern coast providing a direct link between Mumbai and Mangaluru. Ans. (d) The most significant development has been the development of Konkan railway along the eastern coast providing a direct link between Mumbai and Mangaluru. 32. Which of the following is not true about Goldenquadrilateral? (a) It comprises construction of 5,806 Km long 4/6 lane. (b) It is high density traffic corridor connect India’s four big metro cities of-Delhi- Mumbai-Chennai-Kolkata. (c) With the construction of golden quadrilateral, the time, distance and cost of moment among the mega cities of India will be

considerably minimised. (d) It also connects Bihar and Tamil Nadu. Ans. (d) It also connects Bihar and Tamil Nadu. 33. In which of the following year, the first radio programme was broadcast? [NCERT] (a) 1911 (b) 1936 (c) 1927 (d) 1923 Ans. (d) 1923 34. State highways join the state capitals with which of the following places? (a) District headquarters (b) Other important towns of state (c) (a) and (b) are correct (d) Ports of the state Ans. (c) (a) and (b) are correct 35. Match the column 1 with column 2 and choose the correct options with the help of given codes: Railway Zone

Headquarters

I.

North-Eastern

1.

Maligaon (Guwahati)

II.

North East Frontier

2.

Gorakhpur

III.

North Western

3.

Chennai

IV.

Southern

4.

Jaipur

V.

South Central

5.

Kolkata

VI.

North Western

6.

Secunderabad

Codes: I II III IV V VI (a) 1 2 3 4 5 6 (b) 4 2 1 3 6 5 (c) 6 5 4 3 2 1 (d) 2 1 4 3 6 5 Ans. (d) I-2, II-1, III-4, IV-3, V-6, VI-5 36. Which of the following option is correct about NH-1? (a) It connect Delhi to Amritsar (b) It connect Delhi to Kolkata (c) It connect Delhi to Bhopal (d) It connect Delhi to Imphal Ans. (a) It connect Delhi to Amritsar 37. Which NH is know as Sher Shah Suri Marg? (a) National Highway-4 (b) National Highway-3 (c) National Highway-2 (d) National Highway-1 Ans. (d) National Highway-1 38. Which of the following column is not matched correctly?

Short form Expanded form (a) DD. Doordarshan (b) CNP. Common National Programmes (c) INSAT-IA National Television Forum (d) AIR. All India Radio Ans. (c) INSAT-IA National Television Forum 39. Consider the following statements and choose the correct option from them : I. The use of transport and communication depends upon our need to move things from place of their availability to the place of their use. II. Human beings use various methods to move goods, commodities, ideas from one place to another. Options : (a) Only statement I is correct (b) Both I and II are correct (c) Only II is correct (d) Both are incorrect Ans. (b) Both I and II are correct 40. When was the first pipeline constructed in India? (a) 1957 (b) 1958 (c) 1959 (d) 1960 Ans. (c) 1959

41. Which stations among the following are joined by the North South corridor? (a) Srinagar- Kanyakumari (b) Delhi- Kanyakumari (c) Delhi - Chennai (d) Srinagar -Chennai Ans. (a) Srinagar- Kanyakumari 42. Which of the following pairs is not matched correctly? Ways of transport Medium of transport (a) Ropeways Waterways (b) Highways. Landways (c) Cableways. Landways (d) Pipeline. Landways Ans. (a) Ropeways – Waterways 43. Which of the following pairs is not correctly matched ? COLUMN (I)

COLUMN (II)

I.

Mumbai

1.

Major Port

II.

Mundra

2.

Minor Port

III.

Delhi-Goa

3.

NH-1

IV.

Delhi to Kolkata

4.

NH-2

V.

Delhi-Mumbai- ChennaiKolkata

5.

Golden quadrilateral

(a) I Mumbai Major Port

(b) II Mundra Minor Port (c) III Delhi-Goa NH-1 (d) IV Delhi-Mumbai-Chennai-Kolkata Golden quadrilateral Ans. (c) III Delhi-Goa NH1 44. Consider the following statements and choose the correct options for them : 1. State Highways are constructed and maintained by state governments. They join the state capitals with district headquarters and other important towns. 2. State roads are connected to the National Highways. These constitute 4 per cent of total road length in the country. Options : (a) Only statement I is correct (b) Both I and II are correct (c) Only II is correct (d) Both statements are incorrect Ans. (b) Both I and II are correct 45. Match the column I with the Column II and select the correct answer from the given options: Water Ways

Stretch

I.

NW-5

1.

891 Km

II.

NW-4

2.

205 Km

III.

NW-3

3.

1078 Km

IV.

NW-2

4.

588 Km

Options:

(a) I-(1), II-(4), III-(2), IV-(3) (b) I-(4), II-(1), III-(3), IV-(2) (c) I-(2), II-(3), III-(4), IV-(1) (d) I-(4), II-(3), III-(2), IV-(1) Ans. (d) I-(4), II-(3), III-(2), IV-(1) 46. What is India’s share in world trade? (a) 1% (b) 5% (c) 7% (d) 9% Ans. (a) 1% 47. Which element’s of India’s international trade has changed? (a) Amount (b) Composition (c) Direction (d) All of these Ans. (d) All of these 48. Which country is the largest trading partner of India? (a) Britain (b) China (c) The USA (d) Pakistan Ans. (c) The USA 49. Which among the following commodity has highest

percentage share in Exports Composition of India in 20092017? (a) Agriculture and allied products (b) Ore and Minerals (c) Manufactured goods (d) Crude and petroleum products Ans. (c) Manufactured goods 50. Which one of the following commodity of India’s export has declined during 2009-17? (a) Agriculture and allied products (b) Petroleum and crude products (c) Ore and Minerals (d) Manufactured goods Ans. (c) Ore and Minerals 51. In which state is Nhava Sheva port located? (a) Gujarat (b) Goa (c) Maharashtra (d) Karnataka Ans. (c) Maharashtra 52. Which of the following port gained significance after 1961 to handle Iron-ore export to Japan. (a) Jawahar Lal Nehru port (b) Marmagao port (c) New Mangalore port (d) Vishakhapatnam

Ans. (b) Marmagao port 53. After which of the following year foodgrain import was discontinued due to the success of Green revolution? (a) 1960s (b) 1970s (c) 1980s (d) 1990s Ans. (b) 1970s 54. Which one of the following factor in 1973 has pushed the prices and import of petroleum hence, budget was also pushed up? (a) Energy crisis (b) Famine (c) Drought (d) War Ans. (a) Energy crisis 55. What is the major export from New Mangalore Ports? (a) Iron ore (b) Coal (c) Manganese (d) Mica Ans. (a) Iron ore 56. Which of the following statement is/are correct? (i) Food grain import was replaced by fertilisers and petroleum in India. (ii) Machine and equipment, special steel, edible oil and chemical largely make the input basket of India

(a) Only (i) (b) Only (ii) (c) Both (i) and (ii) (d) Neither (i) nor (ii) Ans. (c) Both (i) and (ii) 57. Which one of the following is the main factor of the emergence of ports as gateways of international trade? (a) After the coming of the European traders. (b) Since ancient times (c) After independence (d) After the coming of the Mughals Ans. (a) After the coming of the European traders. 58. India has _____major ports and _____ minor or intermediate ports. Choose the correct answer. (a) 10,200 (b) 10,150 (c) 12,200 (d) 10,200 Ans. (c) 12,200 59. Which authority in case of the major ports decides the policy and plays regulatory functions. (a) Port Authority (b) Local Authority (c) State Authority (d) Central Authority

Ans. (d) Central Authority 60. Which of the following port is called the ‘Queen of Arabian Sea’? (a) Mangalore (b) Kochchi port (c) Mumbai (d) Kandla Ans. (b) Kochchi port 61. Match the column I with Column II and choose the correct options with the help of given Codes. Column I (Port)

Column II (Location)

I.

Kandla Port

1.

Tamil Nadu

II.

Jawaharlal Nehru Port

2.

Kerala

III.

Marmagao Port

3.

Karnataka

IV.

New Mangalore Port

4.

Goa

V.

Kochchi

5.

Maharashtra

VI.

Ennore

6.

Gulf of Kachchh

Codes: I II III IV V VI (a) 1 2 3 4 5 6 (b) 4 2 1 3 6 5 (c) 6 5 4 3 2 1 (d) 2 1 4 3 5 6

Ans. (b) I – 6, II – 5, III – 4, IV – 3, V – 2, VI – 1 62. Which of the following port went to Bangladesh after the partition of the country? (a) Karachi (b) Marmagao (c) Kandala (d) Chittagong Ans. (d) Chittagong 63. Which of the international trade)?

following

statement

is

true

(w.r.t.

(a) The share of agriculture and allied product increased in recent year (b) The share of petroleum and crude product increased in recent year (c) Both (a) and (b) (d) International trade is not mutually beneficial because every country is self sufficient Ans. (b) The share of petroleum and crude product increased in recent year 64. Which of the following is close to the suez-colombo rout? (a) Kochi port (b) Chennai port (c) Vishakhapatnam port (d) Kolkata port Ans. (a) Kochi port 65. Which one of the following is the major export from New Mangalore ports?

(a) Coal (b) Iron ore (c) Copper (d) Mica Ans. (b) Iron ore 66. Most of India’s foreign trade is carried through: (a) Land (b) Sea (c) Air (d) Air and Sea Ans. (d) Air and Sea 67. Which of the following is NOT true about Marmagao Port? (a) It is an artificial harbour in Goa. (b) Construction of Konkan railway has considerably extended the hinterland of this port. (c) Karnataka, hinterland.

Goa,

Southern

Maharashtra

constitute

its

(d) It gained significance after its remodelling in 1961 to handle iron-ore exports to Japan. Ans. (a) It is a artificial harbour in Goa. 68. Which among the following Indian Sea Ports provides facilities to the land-locked neighbouring countries? (a) Nhava Sheva (b) Kandla (c) Kolkata

(d) Vishakhapatnam Ans. (c) Kolkata 69. Which of the following option has major contribution in India’s import trade (a) Europe (b) Africa (c) North America (d) Latin America Ans. (a) Europe 70. Which of the following problem is faced by Kolkata port? (a) The diversion of exports to the other ports (b) Silt accumulation in the Hugli river (c) It handles bulk cargo. (d) It also extends ports facilities to our neighbouring land-locked countries Ans. (b) Silt accumulation in the Hugli river. 71. Which of the following distance is correct about distance of Haldia Port from Kolkata ? (a) 101 Km (b) 107 Km (c) 103 Km (d) 105 Km Ans. (d) 105 Km 72. Trade between two countries is termed as: (a) Internal trade

(b) External trade (c) International trade (d) Local trade Ans. (c) International trade 73. Name the Newport of India which developed to revive Mumbai port: (a) Kandla (b) Jawaharlal Nehru (c) Chennai (d) Kolkata Ans. (b) Jawaharlal Nehru 74. Which one of the following are the main hinterland for Visakhapatnam Port? (a) Andhra Pradesh and Telangana (b) Odisha and Telangana (c) Chhattisgarh and Telangana (d) Jharkhand and Telangana Ans. (a) Andhra Pradesh and Telangana 75. Which one of the following port caters to the needs of Kerala, southern-Karnataka and south western Tamil Nadu? (a) Kochchi (b) Ennore (c) Chennai (d) Tuticorin Ans. (a) Kochchi 76. The port is situated closer to the general routes from the

countries of Middle East, Mediterranean countries, North Africa, North America and Europe where the major share of country’s overseas trade is carried out. The port is 20 km long and 6-10 km wide with 54 berths and has the country’s largest oil terminal. The port is : (a) Kochchi (b) Ennore (c) Mumbai (d) Tuticorin Ans. (c) Mumbai 77. Which of the following port is situated at Mahanadi Delta? (a) Paradeep port (b) Vishakhapatnam (c) Chennai port (d) Kochchi port Ans. (a) Paradeep port 78. In which year Chennai port was built? (a) 1839 (b) 1849 (c) 1859 (d) 1869 Ans. (c) 1859 79. Which one of the following is a landlocked harbour? (a) Vishakhapatnam (b) Mumbai (c) Ennore

(d) Haldia Ans. (a) Vishakhapatnam

1. Which one of the following cultural activity is not a cause of water pollution? (a) Pilgrimage (b) Religious fairs (c) Tourism (d) Use of pesticides Ans. (d) Use of pesticides 2. Which one of the following is not an objective of Namami Gange Programmes ? (a) Developing sewerage treatment systems in towns (b) Monitoring of industrial effluents (c) The Ganga aarti (d) Cleaning of the river surface Ans. (c) The Ganga aarti 3. Erosion, landslides, decay and decomposition of plants and animals are the example of which of the following pollution ? (a) Air Pollution (b) Water Pollution (c) Land Pollution (d) Noise Pollution Ans. (b) Water Pollution 4. Which one of the following is a polluted stretch in the river Ganga?

(a) Farrakka Barrage (b) Extraction of water by Haryana and Uttar Pradesh for irrigation (c) Agricultural run off resulting in high levels of micro-pollutants in the Yamuna (d) Domestic and industrial waste of Delhi flowing into the river Ans. (a) Farraka Barrage 5. Air pollution is taken as addition of containments like: (a) Dust (b) Fumes (c) Odour (d) All of these Ans. (d) All of these 6. Which one of the following is a source of noise pollution at harbour ? (a) Loading and unloading activities being carried at harbour (b) Noise of aircraft etc. passing over harbour (c) Heavy traffic at harbour (d) All of the above Ans. (d) All of the above 7. Which one of the following is not a source of land pollution? (a) Combustion of coal (b) Improper human activities, (c) Disposal of untreated industrial waste, (d) Use of pesticides and fertilisers.

Ans. (a) Combustion of coal 8. Which one of the following is residential areas of the least choice, dilapidated houses, poor hygienic conditions, poor ventilation, lack of basic amenities, like drinking water, light and toilet facilities, etc.? (a) Urban (b) Rural (c) Semi-Urban (d) Slums Ans. (c) Slums 9. Which one of the following is not a type of wasteland? (a) Gullies (b) Barren rocky areas (c) Marshy areas (d) Oceans Ans. (d) Oceans 10. Which one of the following river is highly polluted? [NCERT] (a) Brahmaputra (b) Sutlej (c) Yamuna (d) Godavari Ans. (c) Yamuna 11. Which of the following state bhils tribes are habitat? (a) Madhya Pradesh

(b) Uttar Pradesh (c) Assam (d) Andhra Pradesh Ans. (a) Madhya Pradesh 12. According to Seabrook ‘Dharavi Asia’s largest slum’. When was this book published? (a) 1995 (b) 1996 (c) 1997 (d) 1998 Ans. (b) 1996 13. Which state Dharavi stream is located? (a) Kerala (b) Maharashtra (c) Gujarat (d) Uttar Pradesh Ans. (c) Gujarat 14. Which one of the following diseases is caused by water pollution? [NCERT] (a) Conjunctivitis (b) Diarrhoea (c) Respiratory infections (d) Bronchitis

Ans. (b) Diarrhoea 15. What is the Nature of pollution in river Yamuna? (a) Extraction of water by Haryana and Uttar Pradesh for irrigation (b) Domestic and industrial waste of Delhi flowing into the river (c) Agriculture run off resulting in high level of micro pollutant in the Yamuna. (d) All of the above Ans. (d) All of the above 16. The level of steady noise is measured by sound level expressed in term of: (a) decibels (dB) (b) mcg (c) gm (d) kg Ans. (a) decibels (dB) 17. Which among the following programmes is a part of the Namami Gange Programme? (a) Development of Ganga Grams (b) Development of Ganga Paani (c) Development of Ganga Ghats (d) Development of Ganga Waste Ans. (a) Development of Ganga Grams 18. According to world health Organisation..........of the communication diseases in India are water-borne. (a) One-third (b) One-half

(c) One-fourth (d) None of these Ans. (c) One-fourth 19. Which one of the following is the cause of acid rain? [NCERT] (a) Water pollution (b) Land pollution (c) Noise pollution (d) Air pollution Ans. (d) Air pollution 20. Which one of the following is a main reason of Air Pollution? (a) Combustion of fossil fuels, mining and industries (b) Increasing use of varieties of fuels as the source of energy (c) Emission of toxic gases into the atmosphere (d) Release of oxides of sulphur and nitrogen in the atmosphere. Ans. (a) Combustion of fossil fuels, mining and industries 21. Which of the following is are the polluted stretches of the river Yamuna? (a) Mathura and Agra (b) Delhi to confluence with Chambal (c) Downstream of Kanpur (d) Both (a) and (b) Ans. (d) Both (a) and (b) 22. Which among the following is the main polluter of river

Yamuna? (a) Kanpur (b) Varanasi (c) Allahabad (d) Delhi Ans. (d) Delhi 23. Which among the following city is the major example of environmental pollution in Urban areas in recent years? (a) Solid wastes (b) Congestion (c) Overcrowding (d) Inadequate facilities to support the fast-growing population Ans. (a) Solid wastes 24. Considered the following statements and choose the correct option from the given options: 1. Noise pollution is location specific and its intensity declines with increase in distance from the source of pollution, i.e. industrial areas, arteries of transportation, airport, etc. 2. Noise pollution is hazardous in many metropolitan and big cities in India. Options : (a) Only 1 is correct (b) Only 2 is correct (c) Both the statements are incorrect (d) Both the statements are correct Ans. (d) Both the statements are correct

25. Push and pull factors are responsible for: [NCERT] (a) Migration (b) Land degradation (c) Slums (d) Air pollution Ans. (a) Migration 26. What is/are the objective of Namami Gange programme? (a) Developing sewerage treatment system in town (b) Monitoring of Industrial efficients (c) Development of river front (d) All of the above Ans. (d) All of the above 27. What is the full form of NRSC? (a) Natioanl Region satellite centre (b) National Resource and Source centre (c) National Recovery source centre (d) National Remote Sensing centre Ans. (d) National Remote Sensing centre 28. Which one of the following is the correct per cent of the population living in urban areas as per the data of 2020 ? (a) 45.8 % (b) 46.1% (c) 34.93%

(d) 35.02% Ans. (c) 34.93% 29. Which one of the following statements is not true regarding Daurala ? (a) Jhabua district is located in the westernmost agro-climatic Zone in M.P. (b) Rajiv Gandhi Mission for watershed Management was implemented area (c) The People of the district are very rich (d) Bhils are the main tribe of the region Ans. (c) The people of the district are very rich 30. Which among the following is not major factor of population flow from rural to urban areas? (a) Higher demand of labour in urban areas (b) Low job opportunities in rural areas (c) Unbalanced pattern of development between urban and rural areas (d) Higher education in urban areas Ans. (d) Higher education in urban areas 31. Solid wastes cause health hazards through : (a) creation of obnoxious smell, (b) harbouring of flies and rodents (c) carriers of diseases like typhoid, diphtheria, diarrhoea, malaria and cholera (d) all of the above Ans. (d) all of the above

32. Which one of the following is the estimate of urban population to the total population by 2050? (a) Two-third (b) One-third (c) Three-fourth (d) One-fourth Ans. (a) Two-third 33. Through which human activities human beings pollute the water? (a) Industrial (b) Agricultural (c) Domestic (d) All of these Ans. (d) All of these 34. Which of the following column is not matched correctly? Pollution Pollution Involved (a) Air Oxides of Sulphur (b) Land Nitrates and Nitrites (c) Water Sulphates and Sulphides (d) Noise Noise above tolerance levels Ans. (b) Land – Nitrates and Nitrites 35. Consider the following features and choose the correct title after associating them. (I) It causes various diseases related to respiratory, nervous and circulatory systems.

(II) Smoky fog over cities called as urban smog is caused by atmospheric pollution. (III) It proves very harmful to human health. (IV) It can also cause acid rains. Options: (a) Air Pollution (b) Water Pollution (c) Noise Pollution (d) Land Pollution Ans. (a) Air Pollution 36. Consider the following statements and choose the correct option from the same I. Urban centres exist as undifferentiated homogeneous entities in opposition to the rural areas. II. Urban centres in India are more differentiated in terms of the socio-economic, politico-cultural and other indicators of development than any other areas. Options: (a) Only statement I is correct (b) Both I and II are correct (c) Only II is correct (d) Both are incorrect Ans. (b) Both I and II are correct 37. Which one of the following is incorrectly matched abbreviation ? (a) Swachh Bharat Mission : SBM

(b) National Remote Sensing Centre : NRSC (c) Decibels : de (d) Namami Gange Programme : NGP Ans. (c) Decibels : de 38. Which one of the following pairs is not matched correctly regarding land degradation? Activities Agents of degradation (a) Glaciers Natural Agent (b) Industrial Wastelands Human Agents (c) Land with scrub Both natural and human Agents (d) Degraded forests Natural Agents Ans. (d) degraded forests – Natural Agents 39. Consider and evaluate the following statements and choose them correct. 1. Solid wastes cause health hazards through creation of obnoxious smell, and harbouring of flies and rodents, which act as carriers of diseases like typhoid, diphtheria, diarrhoea, malaria and cholera, etc. 2. These wastes cause frequent nuisance as and when these are carelessly handled, spread by wind and spluttered through rain water. Options: (a) Only statement I is correct (b) Both I and II are correct (c) Only II is correct (d) Both statements are incorrect

Ans. (b) Both I and II are correct